220964005-Deja-Review-Pharmacology-2nd-Ed.pdf

July 20, 2017 | Author: Asif Rahman | Category: Agonist, Drugs, Pharmacology, Medicine, Wellness
Share Embed Donate


Short Description

Download 220964005-Deja-Review-Pharmacology-2nd-Ed.pdf...

Description

DEJA REVIEW ™ Pharmacology

NOTICE

Medicine is an ever-changing science. As new research and clinical experience broaden our knowledge, changes in treatment and drug therapy are required. T he authors and the publisher of this work have checked with sources believed to be reliable in their efforts to provide information that is complete and generally in accord with the standards accepted at the time of publication. However, in view of the possibility of human error or changes in medical sciences, neither the authors nor the publisher nor any other party who has been involved in the preparation or publication of this work warrants that the information contained herein is in every respect accurate or complete, and they disclaim all responsibility for any errors or omissions or for the results obtained from use of the information contained in this work. Readers are encouraged to confirm the information contained herein with other sources. For example and in particular, readers are advised to check the product information sheet included in the package of each drug they plan to administer to be certain that the information contained in this work is accurate and that changes have not been made in the recommended dose or in the contraindications for administration. T his recommendation is of particular importance in connection with new or infrequently used drugs.

DEJA REVIEW™ Pharmacology Se cond Edition Je ssica A. Gle ason, M.D. Stony Brook University B.S. Pharmacology with Departmental Honors Stony Brook, New York School of Medicine Stony Brook University Medical Center Doctor of Medicine Stony Brook, New York Class of 2010 Beth Israel Deaconess Medical Center Department of Anesthesiology Harvard Medical School Boston, MA St. Vincent Hospital Department of Medicine Worcester, MA

Copyright © 2010, 2007 by the McGraw-Hill Companies, Inc. All rights reserved. Except as permitted under the United States Copyright Act of 1976, no part of this publication may be reproduced or distributed in any form or by any means, or stored in a database or retrieval system, without the prior written permission of the publisher. ISBN: 978-0-07-173678-7 MHID: 0-07-173678-6 T he material in this eBook also appears in the print version of this title: ISBN: 978-0-07-162729-0, MHID: 0-07-162729-4. All trademarks are trademarks of their respective owners. Rather than put a trademark symbol after every occurrence of a trademarked name, we use names in an editorial fashion only, and to the benefit of the trademark owner, with no intention of infringement of the trademark. Where such designations appear in this book, they have been printed with initial caps. McGraw-Hill eBooks are available at special quantity discounts to use as premiums and sales promotions, or for use in corporate training programs. To contact a representative please e-mail us at [email protected]. TERMS O F USE T his is a copyrighted work and T he McGraw-Hill Companies, Inc. (“ McGraw-Hill”) and its licensors reserve all rights in and to the work. Use of this work is subject to these terms. Except as permitted under the Copyright Act of 1976 and the right to store and retrieve one copy of the work, you may not decompile, disassemble, reverse engineer, reproduce, modify, create derivative works based upon, transmit, distribute, disseminate, sell, publish or sublicense the work or any part of it without McGraw-Hill’s prior consent. You may use the work for your own noncommercial and personal use; any other use of the work is strictly prohibited. Your right to use the work may be terminated if you fail to comply with these terms. T HE WORK IS PROVIDED “ AS IS.” McGRAW-HILL AND IT S LICENSORS MAKE NO GUARANT EES OR WARRANT IES AS T O T HE ACCURACY, ADEQUACY OR COMPLET ENESS OF OR RESULT S T O BE OBTAINED FROM USING T HE WORK, INCLUDING ANY INFORMAT ION T HAT CAN BE ACCESSED T HROUGH T HE WORK VIA HYPERLINK OR OT HERWISE, AND EXPRESSLY DISCLAIM ANY WARRANT Y, EXPRESS OR IMPLIED, INCLUDING BUT NOT LIMIT ED T O IMPLIED WARRANT IES OF MERCHANTABILIT Y OR FIT NESS FOR A PART ICULAR PURPOSE. McGraw-Hill and its licensors do not warrant or guarantee that the functions contained in the work will meet your requirements or that its operation will be uninterrupted or error free. Neither McGraw-Hill nor its licensors shall be liable to you or anyone else for any inaccuracy, error or omission, regardless of cause, in the work or for any damages resulting there from. McGrawHill has no responsibility for the content of any information accessed through the work. Under no circumstances shall McGraw-Hill and/or its licensors be liable for any indirect, incidental, special, punitive, consequential or similar damages that result from the use of or inability to use the work, even if any of them has been advised of the possibility of such damages. T his limitation of liability shall apply to any claim or cause whatsoever whether such claim or cause arises in contract, tort or otherwise.

This book is dedicated to Adam. Your confidence in me has enriched my career, and your love for me has enriched my life. —Jessica A. Gleason

Contents Faculty Re vie we r/Stude nt Re vie we rs Pre face Acknowle dgme nts Chapter 1 BASIC PRINCIPLES Pharmacokinetics Pharmacodynamics Clinical Vignettes Chapter 2 ANTIMICRO BIAL AGENTS Antibacterial Agents Antifungal Agents Antiviral Agents Antiprotozoal Agents Antihelminthic Agents Clinical Vignettes Chapter 3 CANCER CHEMO THERAPEUTIC AGENTS Basic Pharmacology of Cancer Chemotherapeutics Chapter 4 AUTO NO MIC AGENTS Cholinergic Agents Adrenergic Agents Clinical Vignettes Chapter 5 CNS AGENTS General and Local Anesthetics Opioid Analgesics and Antagonists Anxiolytic and Sedative-Hypnotic Agents Antidepressant Agents Agents for Manic-Depression Antipsychotic Agents Agents for Parkinson Disease Anticonvulsant Agents Agents for Migraine Clinical Vignettes Chapter 6 CARDIO VASCULAR/RENAL AGENTS Antiarrhythmic (Antidysrrhythmic) Agents Congestive Heart Failure Agents Antianginal Agents Antihypertensive Agents Antihyperlipidemic Agents Anticoagulation Agents Clinical Vignettes Chapter 7 PULMO NARY AGENTS Drugs for Asthma Chronic Obstructive Pulmonary Disease Agents Antitussive Agents Agents for Allergic Rhinitis Agents for Respiratory Distress Syndrome Agents for Cystic Fibrosis Clinical Vignettes Chapter 8 GASTRO INTESTINAL AGENTS Agents for Gastroesophageal Reflux Disease Agents for Peptic Ulcer Disease Agents for Inflammatory Bowel Disease Agents for Nausea and Vomiting Agents for Diarrhea and Constipation Clinical Vignettes Chapter 9 ENDO CRINE AGENTS Agents for Diabetes Mellitus Agents for Diabetes Insipidus Agents for T hyroid Disorders Adrenal Steroids Androgens and Anti-Androgens Estrogens and Anti-Estrogens—Selective Estrogen Receptor Modulators Progestins and Antiprogestins Clinical Vignettes Chapter 10 ANTI-INFLAMMATO RY AGENTS AND p-AMINO PHENO L DERIVATIVES (ACETAMINO PHEN) Clinical Vignettes Chapter 11 MISCELLANEO US TO PICS IN PHARMACO LO GY Immunosuppressive Agents

Agents for Obesity Agents for Osteoporosis Agents for Erectile Dysfunction Agents for Rheumatoid Arthritis Agents for Gout Retinoids Herbal Medications Substance Abuse and T olerance T oxicology Clinical Vignettes Appe ndix Sugge ste d Re adings Inde x

Faculty Reviewer Migue l Be rrios, Ph.D. Department of Pharmacology School of Medicine State University of New York Stony Brook, New York

Student Reviewers Joe Bart Lake Erie College of Osteopathic Medicine Bradenton Division Class of 2008 Rosalyn Pham University of Washington School of Medicine Class of 2008 Ale xis M. Dallara SUNY Downstate College of Medicine Class of 2009

Preface T he Deja Review series is a unique resource that has been designed to allow you to review the essential facts and determine your level of knowledge on the subjects tested on Step 1 of the United States Medical Licensing Examination (USMLE). T he format of this book is highly beneficial to aid one’s understanding of the subject of pharmacology for several reasons. T he breadth of the subject material could quickly become overwhelming without some sort of organizational framework, provided concisely and coherently in this book. T he question/answer format allows for quick reviews of familiar concepts, and moreover is conducive to repetition, which, above all else, will be necessary to master this subject. T he new, expanded clinical vignettes section now follow each chapter in this edition to allow you to test that you are able to integrate the knowledge you have obtained and ensure your grasp on the material extends beyond rote memorization. A great effort has been made in this edition to expand emphasis on clinical application and to make necessary updates to this ever evolving realm of medicine.

ORGANIZATION All concepts are presented in a question and answer format that covers key facts on commonly tested topics in medical pharmacology. T he first chapter introduces the basic principles of the subject. T hen the chapters move through defined subtopics within pharmacology. A series of vignettes follows most chapters. T hese vignettes are meant to be representative of the types of questions tested on national licensing exams to help you further evaluate your understanding of the material presented in the chapter. T he compact, condensed design of the book is conducive to studying on the go, especially during any downtime throughout your day. T his question and answer format has several important advantages: It provides a rapid, straightforward way for you to assess your strengths and weaknesses. It allows you to efficiently review and commit to memory a large body of information. It serves as a quick, last-minute review of high-yield facts.

At the end of each chapter, you will find clinical vignettes that expose you to the pro-totypic presentation of diseases classically tested on the USMLE Step 1. T hese board-style questions put the basic science into a clinical context, allowing you apply the facts you have just reviewed in a clinical scenario.

HOW TO US E THIS BOOK Remember, this text is not intended to replace comprehensive textbooks, course packs, or lectures. It is simply intended to serve as a supplement to your studies during your medical pharmacology course and Step 1 preparation. T his text was contributed to by a number of medical students to represent the core topics tested on course examinations and Step 1. You may use the book to quiz yourself or classmates on topics covered in recent lectures and clinical case discussions. A bookmark is included so that you can easily cover up the answers as you work through each chapter. However you choose to study, I hope you find this resource helpful throughout your preparation for course examinations, the USMLE Step 1, and other national licensing exams. Jessica Gleason

Acknowledgments T he author would like to thank Miguel Berrios, Ph.D., Kirsten Funk, Editor, and Somya Rustagi, Project Manager for their invaluable contributions to this text and their efforts in making this a useful resource for students.

CHAPTER 1 Basic Principles Basic principles are often perceived as the most challenging aspect of learning pharmacology. While possibly conceptually difficult, this subject is absolutely key to gain understanding how medications exert their effects and side effects. While licensing exams may not stress basic principles in their pure form, this topic is at the core of many pharmacology questions that may appear in these examinations. T herefore, a strong knowledge of basic principles will help you with your study of pharmacology, and throughout your medical career.

PHARMACOKINETICS De fine the following te rms: Pharmacokine tics Field of study that deals with time required for drug absorption, distribution in the body, metabolism, and method of excretion. In short, it is the body’s effect on the drug. Volume of distribution(Vd) T he apparent volume in the body available to contain the drug. Formula: Vd = Dose/Plasma Drug Concentration Is Vd a physiologic value ? No Is Vdan absolute value for any give n drug? No Cle arance (CI) Volume of blood cleared of the drug per unit time; Cl = Rate of elimination of drug/plasma drug concentration; Total Body Cl = Cl hepatic + Clrenal + Clpulmonary + Clother Half-life (t1/2) T ime required for plasma concentration of drug to decrease by one-half after absorption and distribution are complete; t1/2 = (0.693 × Vd)/(Cl) Bioavailability (F) T he fraction of (active) drug that reaches the systemic circulation/site of action after administration by any route; F = (AUCpo)/(AUCiv), where AUC po and AUCiv are the extravascular and intravenous areas under the plasma concentration versus time curves, respectively Ste ady state (Css) Steady state is reached when the rate of drug influx into the body = the rate of drug elimination out of the body; Css = plasma concentration of drug at steady state How much drug is le ft afte r two half-live s? 25% How much drug is le ft afte r thre e half-live s? 12.5% During constant infusion, what pe rce nt of ste ady state is re ache d afte r one half-life ? 50% During constant infusion, what pe rce nt of ste ady state is re ache d afte r two half-live s? 75% During constant infusion, what pe rce nt of ste ady state is re ache d afte r thre e half-live s? 87.5% During constant infusion, what pe rce nt of ste ady state is re ache d afte r four half-live s? 94% Give the e quation for the following te rms: Infusion rate (k 0) k 0= Cl × Css Loading dose (LD) LD = (Vd × CSS)/(F); for examination purposes, F is usually 1 Mainte nance dose (MD) (Cl × Css × τ)/(F), where τ (tao) is the dosing interval Cle arance (Cl) Cl = K × Vd, where K is the elimination constant Volume of distribution (Vd) Vd= = (LD)/(CSS) Half-life (t1/2) tl/2 = (0.693)/(K) or (0.693 × Vd)/(Cl) What happe ns to the ste ady state conce ntration of a drug if the infusion rate is double d? Steady state concentration is also doubled; remember that dose and concentration are directly proportional (linear kinetics); Css × k0/Cl If the re is no active se cre tion or re absorption, the n re nal cle arance (Cl renal) is e qual to what? Glomerular filtration rate (GFR) If a drug is prote in bound, the n Cl renal is e qual to what? GFR × free fraction (of drug) What happe ns to the LD in patie nts with impaire d re nal or he patic function? Stays the same What happe ns to the MD in patie nts with impaire d re nal or he patic function? Decreases For e ach of the following state me nts, state whe the r it re fe rs to z e ro-orde r e limination or first-orde r e limination? Rate of e limination is constant, re gardle ss of conce ntration Zero-order elimination Plasma conce ntration de cre ase s e xpone ntially with time First-order elimination Rate of e limination is proportional to the drug conce ntration First-order elimination Plasma conce ntration de cre ase s line arly with time Zero-order elimination

Rate of e limination is inde pe nde nt of conce ntration Zero-order elimination Rate of e limination is de pe nde nt on conce ntration First-order elimination What are some e xample s of drugs/substance s that unde rgo z e ro-orde r e limination? Acetylsalicylic acid (Aspirin, ASA) at high/toxic concentrations; phenytoin; ethanol De scribe the following type s of me tabolism: Phase I me tabolism Metabolism that generally yields more polar, water-soluble metabolites (may still be active); enzyme activity decreases with patient’s age Phase II me tabolism Metabolism that generally yields very polar, inactive metabolites (renally excreted); enzyme activity does not decrease with patient’s age Phase I (microsomal) me tabolism involve s what type s of re actions? Oxidation; reduction; hydrolysis (carried out by cytochrome P-450 enzymes) Phase II (nonmicrosomal) me tabolism involve s what type s of re actions? Glucuronidation; acetylation; sulfation; amidation; glutathione conjugation Give e xample s of drugs that unde rgo phase II me tabolism: Isoniazid (INH), morphine, 6-mercaptopurine, acetaminophen What are the pote ntial conse que nce s of phase I oxidation re actions with re gard to drug activity and e limination? Drug activity may or may not change (no rule, ie, potentially dangerous outcome). Drug elimination is usually increased due to greater water solubility. What are the pote ntial conse que nce s of phase II re actions with re gard to drug activity and e limination? Drug products of phase II reactions are usually inactive and their renal elimination is enhanced. Whe re are cytochrome P-450 e nz yme s found? Smooth endoplasmic reticulum of cells in mainly the liver, but also found in the gastrointestinal (GI) tract, kidney, and lungs Explain what e ach type of the following clinical phase s in drug de ve lopme nt is trying to accomplish? Phase I Safety in healthy individuals; drug pharmacokinetics Phase II Efficacy in diseased individuals (small scale trials, single- double-blind) Phase III Efficacy in diseased individuals (small scale trials, single- or double-blind) Phase IV Postmarketing surveillance (monitored release) At what point during drug de ve lopme nt is an inve stigational ne w drug (IND) application file d? Before phase I At what point during drug de ve lopme nt is a ne w drug application (NDA) file d? After phase III (and before phase IV) What doe s the te rm bioe quivale nce me an? When comparing two formulations of the same compound, they are said to be bioequivalent to each other if they have the same bioavailability and the same rate of absorption. What is the first-pass e ffe ct? After oral administration, many drugs are absorbed intact from the small intestine and transported first via the portal system to the liver, where they undergo extensive metabolism, therefore usually decreasing the bioavailability of certain oral medications. How many lite rs are in e ach of the following compartme nts of an ave rage adult human? Blood 5L Plasma 3L Total body wate r (TBW) 42 L (avg. 70 kg man × 60%. For women 50% of mass [body weight] in kg is body water due to lower lean muscle mass and higher fat content [adipose tissue]). What is the most common plasma prote in that drugs bind to? Albumin Displacing a drug that is bound to plasma prote in(s), for e xample , albumin, will incre ase its what? Its free fraction (therefore may possibly increase the risk of toxicity because the plasma concentration of active drug has been increased, yet depending on the drug, an increase in free fraction may actually increase its metabolism because more drug is available to metabolizing enzymes) For e ach of the following me chanisms of me mbrane transport, state if e ne rgy is re quire d, if a carrie r is re quire d, and if the syste m is saturable ? Passive diffusion No energy required; no carrier; not saturable (proportional to concentration gradient) Facilitate d diffusion No energy required; carrier required; saturable Active transport Against concentration/electrical gradient, therefore energy required; carrier required; saturable The pe rme ation of drugs across ce llular me mbrane s is de pe nde nt on what drug prope rtie s and (local) circumstance s? Drug solubility; drug concentration gradient; drug ionization; surface area; vascularity Acidification of urine will incre ase re nal e limination of what type s of drugs? Weak bases (ionized form of drug, BH+ ) will be trapped in the renal tubules and thus excreted in the urine. Acidification of urine will de cre ase re nal e limination of what type s of drugs? Weak acids (nonionized, HA, form of a drug) can cross membranes. Alkaliniz ation of urine will incre ase re nal e limination of what type s of drugs? Weak acids (ionized, A− , form of a drug) will be trapped in the renal tubules and thus excreted in the urine. Alkaliniz ation of urine will de cre ase re nal e limination of what type s of drugs? Weak bases (nonionized, B, form of a drug) can cross membranes. What age nts are use d to acidify urine ? NH4Cl; high dose of vitamin C What age nts are use d to alkaliniz e urine ? NaHCO3; acetazolamide Give an e xample of a we akly acidic drug: Acetylsalicylic acid (ASA); barbiturates Give an e xample of a we akly basic drug: Amphetamines Give e xample s of drugs me taboliz e d by e ach of the following cytochrome P-450 e nz yme s me taboliz e s: CYP 1A2

Caffeine; ciprofloxacin; theophylline; R-warfarin CYP 2C9 Ibuprofen; naproxen; phenytoin; S-warfarin CYP 2C19 Diazepam; omeprazole CYP 2D6 Codeine; dextromethorphan; fluoxetine; haloperidol; loratadine; metoprolol; paroxetine; risperidone; thioridazine; venlafaxine CYP 2E1 Ethanol; INH; acetaminophen (at high doses) CYP 3A4 (50%-60% of all the rape utically use d drugs are me taboliz e d via CYP 3A4) Alprazolam; carbamazepine; cyclosporine; diltiazem; erythromycin; fluconazole; itraconazole; ketoconazole; lidocaine; lovastatin; midazolam; nifedipine; quinidine; simvastatin; tacrolimus; verapamil Give e xample s of drugs and he rbal e xtracts that ge ne rally induce cytochrome P-450 e nz yme s: Phenobarbital; nicotine; rifampin; phenytoin; carbamazepine; St. John’s wort; chronic ethanol consumption Give e xample s of drugs and foods that ge ne rally inhibit cytochrome P-450 e nz yme s: Erythromycin; ketoconazole; ciprofloxacin; quinidine; cimetidine; omeprazole; ritonavir; chloramphenicol; acute alcohol intoxication, grapefruit juice The following graph de picts what type of e limination?

The following graph de picts what type of e limination?

Intrace llular volume (ICV) make s up what fraction of TBW? 2/3 (ICV = 2/3 T BW) Extrace llular volume (ECV) make s up what fraction of TBW? 1/3 (ECV = 1/3 T BW) Inte rstitial volume make s up what fraction of ECV? 2/3 (interstitial volume = 2/3 ECV) Plasma volume make s up what fraction 1/3 (plasma volume = 1/3 ECV) of ECV? What type of e limination involve s a constant fraction of drug e liminate d pe r unit time ? First-order elimination What type of e limination involve s a constant amount of drug e liminate d pe r unit time ? Zero-order elimination

PHARMACODYNAMICS

De fine the following te rms: Pharmacodynamics (PD) Field of study that deals with the relationship between plasma concentration of a drug and the body response obtained to that drug; in short, the drug’s effect on the body Affinity Ability of a drug to bind to a receptor; on a graded dose-response curve, the nearer the curve is to the y axis, the greater the affinity (affinity deals with drugs acting on the same receptor) Efficacy How well a drug produces a pharmacological response; on a graded dose-response curve, the height of the curve represents the efficacy Pote ncy T he amount of drug required to obtain a desired effect; on a graded dose-response curve, the nearer the curve is to the y axis, the greater the potency Will the curve s for two drugs acting on the same re ce ptor be paralle l or nonparalle l to e ach othe r on a grade d dose -re sponse curve ? Parallel Will the curve s for two drugs acting on diffe re nt re ce ptors be paralle l or nonparalle l to e ach othe r on a grade d dose -re sponse curve ? Nonparallel For the graph be low, which drug has gre ate r pote ncy?

For the graph be low, which drug has gre ate r e fficacy?

De fine the following te rms: Agonist A drug that binds to a receptor, alters its conformation, and activates that receptor’s function Full agonist An agonist capable of producing a maximal response Partial agonist An agonist incapable of producing a maximal response; less efficacious than a full agonist Antagonist A drug that binds to a receptor and prevents activation of a receptor Compe titive antagonist An antagonist that competes with an agonist for the same receptor site; graded dose-response curve will be shifted to the right (parallel) thus decreasing the agonist’s potency (and affinity); maximal response will not be affected Noncompe titive antagonist An antagonist that acts at a different site from the agonist, yet still prevents the agonist from activating its receptor; potency not affected; maximal response will be decreased; causes a nonparallel shift to the right on a graded dose-response curve What happe ns whe n a partial agonist is adde d in the pre se nce of a full agonist? T he response of the full agonist is reduced; partial agonist is acting as an antagonist. How doe s a compe titive antagonist affe ct the following? Affinity (1/Km) of agonist Decreased (thus Km will increase as it is inversely proportional to affinity) Maximal re sponse (Vmax) of agonist

No change How doe s a noncompe titive antagonist affe ct the following? Affinity (1/Km) of agonist No change Maximal re sponse (Vmax) of agonist Decreased What type of antagonism doe s the following graph re pre se nt?

What type of antagonism doe s the following graph re pre se nt?

What type of antagonism doe s the following graph re pre se nt?

What type of antagonism doe s the following graph re pre se nt?

Will incre asing the dose of an agonist Ye s comple te ly re ve rse the e ffe ct of a compe titive antagonist? Yes Will incre asing the dose of an agonist No comple te ly re ve rse the e ffe ct of a noncompe titive antagonist? No De fine the following te rms: Pharmacologic antagonism Antagonist and agonist compete for the same receptor site. Physiologic antagonism Two different types of agonists acting at different receptors causing opposite responses, therefore, antagonizing each other (acetylcholine [ACh] activating an M receptor to cause bradycardia is antagonized by norepinephrine [NE] acting at a (β-receptor to cause tachycardia) Che mical antagonism Response to a drug is antagonized by another compound that binds directly to the effector drug (digoxin is antagonized by digoxin immune Fab[Digibind] which binds directly to digoxin and not its receptor) Pote ntiation When one agonist enhances the action of another compound (benzodiazepines and barbiturates potentiate the effect of gamma-aminobutyric acid (GABA) on its receptor); graded dose-response curve is shifted to the left What is a quantal (cumulative ) dose -re sponse curve ? Curve showing the percentage of a population responding to a given drug effect versus dose of drug given (or log of dose); allows you to visualize intersubject variability regarding drug response in graph form What is ED50 Estimation of the effective dose in 50% of a population; the dose at which 50% of the population will respond to the drug Can you obtain the ED50 from a grade d (quantitative ) dose -re sponse curve ? No (this curve does not represent a population of individuals) Can you obtain the ED50 from a quantal (cumulative ) dose -re sponse curve ? Yes (this curve represents a population of individuals) What is TD50? Estimation of the toxic dose in 50% of a population; the dose at which 50% of the population will have toxic effects from the drug What is LD50? Estimation of the lethal dose in 50% of a population; the dose at which 50% of the population will die from the drug What is the rape utic inde x (TI)? T he relative safety of a drug by comparing the ED50 to either the T D50 or LD50; the safer the drug, the “ wider” the T I, meaning the T D50 or LD50 is much greater than the ED50; drugs with a “ narrow” T I have their T D50 or LD50close to the ED50 How do you calculate TI?

In the following graph, what is the ED50?

In the following graph, what is the TD50?

In the following graph, what is the TI?

What is a narrow the rape utic inde x (NTI)? Drugs with an NT I usually have a T I less than 2. List some e xample s of drugs with an NTI: Carbamazepine; digoxin; levothyroxine; lithium; phenytoin; theophylline; valproic acid; warfarin

CLINICAL VIGNETTES If the ED50 of a particular drug is 100 mg and the LD50 is 1 g, the n what is the drug’s the rape utic inde x (TI)? T I = 10; T I = LD50/ED50 If 100% of a drug is cle are d re nally, the n what would happe n to the drug’s half-life if the le ft kidne y was to comple te ly fail? Increase twofold; tl/2- (0.7 × Vd)/(Cl); if only one kidney is “ working” then the clearance has been cut in half thereby doubling the half-life (half-life is inversely proportional to clearance) A 57-ye ar-old man, that re ce ntly unde rwe nt a live r transplant 2 ye ars ago, is curre ntly be ing immunosuppre sse d with cyclosporin A. The patie nt asks his primary care physician if he can take ove r-the -counte r St. John’s wort for his “de pre sse d mood.” How should the patie nt’s physician re spond to this que stion? T he physician should advise against the use of over-the-counter medications. St. John’s wort is an inducer of the cytochrome P-450 enzymes that metabolize cyclosporin A, thereby leading to subtherapeutic serum levels and potentially inducing a graft rejection secondary to inadequate immunosuppression. A woman who is a long-time Floridian re tire e has e njoye d having re d grape fruits for bre akfast e ach morning be fore taking a be ach stroll with he r frie nds. De spite he r good physical condition she was re ce ntly diagnose d with ge ne ral anxie ty disorde r and was pre scribe d triaz olam be fore be dtime . Doe s the physician ne e d to le arn this patie nt’s me al habits be fore pre scribing this me dication? Yes, consumption of fresh grapefruits and grapefruit juice will inhibit the metabolism of triazolam leading to overdosing and increased sedation in an elderly patient.

CHAPTER 2 Antimicrobial Agents ANTIBACTERIAL AGENTS What is a bacte riostatic antibiotic? An antibiotic that causes reversible inhibition of growth. T he bacteria are still present and able to replicate should the bacteriostatic antibiotic be removed. A bacteriostatic antibiotic therefore prevents the exponential growth of bacteria, allowing the host immune system better chances of clearing the bacteria that is present in the body. What is a bacte ricidal antibiotic? An antibiotic that causes irreversible inhibition of growth, therefore directly killing the bacteria Why should bacte riostatic and bacte ricidal antibiotics not be give n toge the r? Bacteriostatic drugs will antagonize the effects of bacteriocidal drugs which rely on the active replication and utilization of environmental resources by bacteria. Which antibiotic inhibits bacte rial ce ll wall synthe sis by blocking glycope ptide polyme riz ation through binding tightly to the D-alanyl-D-alanine portion of the ce ll wall (pe ptidoglycan) pre cursor? Vancomycin Vancomycin-re sistant bacte ria change the ir D-ala-D-ala te rminus of the pe ptide side chain to what? To D-ala-D-lactate; this change prevents the cross linking reaction necessary for elongation of the peptide side chain, weakening the cell wall and making the bacteria susceptible to lysis Vancomycin is most commonly use d to tre at what type s of infe ctions? Gram-positive infections. Vancomycin is only effective against gram-positive bacteria, and is particularly useful for infections due to methicillin resistant Staphylococcus aureus (MRSA) infections. Doe s vancomycin have good oral bioavailability? No. Vancomycin can be given orally to treat Clostridium difficile enterocolitis because the drug stays in the gastrointestinal tract (it is poorly absorbed from the GI tract). What are the adve rse e ffe cts of vancomycin? Hyperemia, or “ red man” syndrome (see following questions); ototoxicity (rare, but it must be used with caution when coadministered with other drugs having ototoxicity, such as aminoglycosides); nephrotoxicity (similar situation as described for ototoxicity); phlebitis at site of injection Re le ase of what substance is re sponsible for “re d man” syndrome ? Histamine How can vancomycin-induce d “re d man” syndrome be pre ve nte d? By slowing the infusion rate. Infusion over 1 to 2 hours is normally sufficient. Additionally, antihistamines may be coadministered. Which antibiotic inhibits the phosphorylation/de phosphorylation cycling of the lipid carrie r re quire d in the transfe r of pe ptidoglycan to the ce ll wall? Bacitracin (used topically only due to severe nephrotoxicity if given systemically) Sulfonamide antibiotics antagoniz e what compound? Para-aminobenzoic acid (PABA) (see next answer) What is the me chanism of action of sulfonamide antibiotics? Sulfonamides are structural analogs of PABA. T his class of antibiotics is effective against bacteria that must use PABA to synthesize folate de novo. Sulfonamides work by inhibiting dihydropteroic acid synthase, the enzyme that catalyzes the condensation reaction between PABA and dihydropteridine to form dihydropteroic acid, the first step in the synthesis of tetrahydrofolic acid. Do humans posse ss dihydropte roic acid synthase ? No. T herefore sulfonamides are selectively toxic to bacteria and other microorganisms. Against which microorganisms are sulfonamide s e ffe ctive ? Gram-positive and gram-negative bacteria, Nocardia, Chlamydia trachomatis, some protozoa (malaria), Escherichia Coli, Klebsiella, Salmonella, Shigella, Enterobacter Are sulfonamide antibiotics bacte ricidal or bacte riostatic? Primarily bacteriostatic What are the adve rse e ffe cts of sulfonamide antibiotics? Nausea; vomiting; diarrhea; phototoxicity; hemolysis (in individuals having glucose-6-phosphate dehydrogenase (G6PD) deficiency); hypersensitivity; StevensJohnson syndrome (incidence and severity of adverse effects greatly increase immunocompromised in AIDS patients) Why are sulfonamide antibiotics contraindicate d in ne onate s? T hey displace bilirubin from albumin thereby causing kernicterus in neonates. Give e xample s of sulfonamide antibiotics: Sulfamethoxazole; sulfacetamide; sulfisoxazole; sulfadiazine (only available in the United States in combination with pyrimethamine); sulfadoxine in combination with pyrimethamine (the antimalarial “ Fansidar”) Name antibiotics that works syne rgistically with sulfonamide s by pre ve nting the ne xt re action in folate synthe sis: T rimethoprim or pyrimethamine What is the me chanism of action of trime thoprim? Competitive inhibitor of dihydrofolic acid reductase (DHFR), the enzyme that converts dihydrofolic acid to tetrahydrofolic acid What are the adve rse e ffe cts of trime thoprim? Leukopenia; granulocytopenia; thrombocytopenia; megaloblastic anemia The trime thoprim-sulfame thoxaz ole combination is most commonly use d to tre at what type of infe ctions? Urinary tract infections (UT I) primarily caused by E. coli as well as many upper respiratory infections (URI) such as Pneumocystis jiroveci pneumonia, some nontuberculous mycobacterial infections, most S. aureus strains, Pneumococcus, Haemophilus sp, Moraxella catarrhalis, and Klebsiella pneumoniae infections. However, up to 30% of UT I and URI pathogenic strains are resistant to this antibiotic combination. Give e xample s of how bacte ria may be come re sistant to sulfonamide antibiotics: Increased concentrations of PABA; decreased binding affinity of target enzymes; uptake and use of exogenous sources of folic acid What is the me chanism of action of fluoroquinolone antibiotics? Inhibition of two DNA gyrases (bacterial DNA topoisomerase II and IV) which in turn prevents relaxation of supercoiled DNA inhibiting DNA replication and transcription Give e xample s of fluoroquinolone antibiotics: Ciprofloxacin, moxifloxacin, gemifloxacin, levofloxacin, lomefloxacin, norfloxacin, ofloxacin, gatifloxacin. All are fluorinated derivatives of nalidixic acid giving them the ability to be active systemically. What type s of bacte ria are susce ptible to quinolone antibiotics? Many gram-positive and gram-negative bacteria. Examples include Shigella, Salmonella, toxigenic E. Coli, Campylobacter, Pseudomonas, Enterobacter, chlamydial urethritis or cervicitis, and atypical mycobacterial infections What are the adve rse e ffe cts of fluoroquinolone antibiotics?

Generally are well tolerated, but nausea, vomiting, and diarrhea are the most common adverse effects. Dizziness, insomnia, headache, photosensitivity, QT interval prolongation also occur with certain quinolones. Gatifloxacin causes hyperglycemia in diabetics and hypoglycemia when used in combination with oral hypoglycemics, and therefore is only available for ophthalmic use in the United States. Why are fluoroquinolone antibiotics contraindicate d in childre n? T hey have deleterious effects on cartilage development, thereby causing tendonitis and possible tendon rupture. Are fluoroquinolone antibiotics bacte ricidal or bacte riostatic? Bactericidal Give e xample s of how bacte ria may be come re sistant to fluoroquinolone antibiotics: Reduced drug penetration (drug efflux pumps); mutations in DNA gyrases result in decreased binding affinity of bacterial target enzymes for fluoroquinolones What is the me chanism of action of β-lactam antibiotics? T hey weaken the cell wall by inactivating transpeptidases (penicillin-binding protein [PBPs]), thereby inhibiting transpeptidation reactions necessary in the cross-linking of peptidoglycan subunits in the bacterial cell wall. Name the four main classe s of β-lactam antibiotics: 1. 2. 3. 4.

Penicillins Cephalosporins Carbapenems Monobactams

Give e xample s of how bacte ria may be come re sistant to β-lactam antibiotics? Production of β-lactamases (cleaves the β-lactam ring—the most common mechanism); alteration of PBPs; inhibition of drugs to reach PBPs; downregulation of porin structure (only in gram-negative organisms since gram-positive organisms lack the outer cell wall where porins are located); development of efflux pumps in gram-negatives Which class of β-lactam antibiotics is re sistant to β-lactamase s? Monobactams Give an e xample of a monobactam antibiotic: Aztreonam. T his is the only monobactam available in the United States. What organisms are monobactams active against? Aerobic gram-negative rods, including pseudomonas Can a monobactam be use d in a pe nicillin-alle rgic patie nt? Yes. T his makes monobactams a good choice for patients with a penicillin allergy and a serious gram-negative infection. Give e xample s of antibiotics that be long to e ach of the following pe nicillin classe s: β-lactamase susce ptible ; narrow spe ctrum Penicillin G (intravenous); penicillin V (oral) β-lactamase susce ptible ; broad spe ctrum Amoxicillin; ampicillin; piperacillin; ticarcillin β-lactamase re sistant; narrow spe ctrum Methicillin; nafcillin; dicloxacillin; oxacillin Pe nicillins are syne rgistic with what othe r antibiotic drug class in the tre atme nt of e nte rococcal and pse udomonal infe ctions? Aminoglycosides. Aminoglycosides are very polar molecules that cannot easily cross the cell wall. With inhibition of cell wall formation by penicillins, aminoglycosides are then able to enter cells and exert their effects. Name thre e β-lactamase inhibitors that can be use d in combination with pe nicillins: 1. Clavulanate 2. Sulbactam 3. T azobactam What are the adve rse e ffe cts of the pe nicillin antibiotics? Hypersensitivity; acute interstitial nephritis (common with methicillin); nausea; vomiting; diarrhea; hepatitis (oxacillin); hemolytic anemia; pseudomembranous colitis (ampicillin) What is the me chanism of me thicillin re sistance by S. aureusl? Production of an alternative PBP 2a Doe s Streptococcus make β-lactamase ? No (mechanism of resistance is via altered PBPs) Are β-lactam antibiotics e ffe ctive in tre ating Mycoplasma infe ctions? No (Mycoplasma have no cell walls) Give e xample s of me dications in e ach of the following ce phalosporin classe s: Note: It is unnecessary to memorize every drug in each generation. Usually there are two to three cephalosporins on formulary, so their use will vary depending on the particular hospital. Licensing exams will not ask you to choose between cephalosporins in the same generation. First ge ne ration Cefazolin; cephalexin; cefadroxil; cephapirin; cephradine Se cond ge ne ration Cefuroxime; cefotetan; cefaclor; cefoxitin; cefprozil; cefpodoxime; cefamandole; cefmetazole; loracarbef; cefonicid Third ge ne ration Cefotaxime; Ceftazidime; Ceftriaxone; Cefpodoxime; cefdinir; cefditoren; ceftibuten; cefixime; cefoperazone; ceftizoxime; moxalactam Fourth ge ne ration Cefepime (only representative drug of this generation) Which ce phalosporin has the broade st spe ctrum of activity and is re sistant to β-lactamase s? Cefepime How doe s the antibiotic spe ctrum of activity of ce phalosporins vary by ge ne ration, that is, how doe s the cove rage of se cond-ge ne ration drugs compare to first ge ne ration and so on? Second generation: increased gram-negative coverage as compared to first generation. T hird generation: continued increase in gram-negative coverage and greater ability to cross blood-brain barrier as compared to second generation drugs. Fourth generation: increased β-lactamase resistance as compared to thirdgeneration drugs Give e xample s of carbape ne m β- lactams: Imipenem; meropenem; ertapenem; doripenem Why is cilastatin give n concomitantly with imipe ne m? Cilastatin (a dipeptidase inhibitor) inhibits renal dihydropeptidases in the renal tubules which inactivate imipenem, thereby allowing imipenem to exert its effects.

What is the diffe re nce in microbial cove rage be twe e n imipe ne m and e rtape ne m? Ertapenem does not cover Acinetobacter species and pseudomonal species. What is the me chanism of action of aminoglycoside antibiotics? Binds to 3OS ribosomal subunit to prevent formation of initiation complex, thereby inhibiting bacterial protein synthesis; incorporation of incorrect amino acids in the growing peptide chain Are aminoglycoside antibiotics bacte ricidal or bacte riostatic? Bactericidal Give e xample s of aminoglycoside antibiotics: Gentamicin; tobramycin; streptomycin; amikacin; neomycin What is stre ptomycin commonly use d to tre at? T uberculosis infections Why might the e fficacy of an aminoglycoside be incre ase d whe n give n as a single large dose as oppose d to multiple smalle r dose s (two re asons)? 1. Concentration-dependent killing: increasing concentrations of aminoglycosides kill a greater proportion of bacteria more quickly. 2. Postantibiotic effect: the antibacterial activity of aminoglycosides lasts longer than detectable levels of the drug are found in the bloodstream. Against which organisms are aminoglycoside s e ffe ctive ? Gram-negative aerobes. Aminoglycosides passively diffuse across porins in the outer membrane of gram-negatives and their entry across the inner membrane is oxygen dependent. Use with a (β-lactam increases gram-positive coverage. Although syne rgistic in the ir clinical e ffe cts, why can pe nicillins and aminoglycoside s not be give n in the same vial? T he penicillins would directly inactivate the aminoglycosides. What are the adve rse e ffe cts of the aminoglycoside antibiotics? Nephrotoxicity (acute tubular necrosis); ototoxicity; hypersensitivity; neuromuscular blockade. Adverse effects are also dose dependent, just like efficacy, so close monitoring of drug levels is necessary. Ototoxicity may be irreversible; therefore it is imperative to act before a clinical change in hearing has occurred. In re gards to aminoglycoside ototoxicity, are high-fre que ncy or low-fre que ncy sounds affe cte d first? High frequency Give e xample s of how bacte ria may be come re sistant to aminoglycoside antibiotics? Inactivation of drug via conjugation reactions (acetylation; adenylation; phosphorylation); inactivation driven by plasmid-encoded enzymes Which aminoglycoside antibiotic is the most toxic? Neomycin (used primarily for topical application) What is the me chanism of action of clindamycin? Binds to 5OS ribosomal subunit to inhibit translocation of peptidyl-tRNA from acceptor to donor site, thereby inhibiting bacterial protein synthesis Is clindamycin bacte ricidal or bacte riostatic? Bacteriostatic What is the spe ctrum of activity of clindamycin? Gram-positives (eg, penicillin-resistant Staphylococcus); anaerobes (eg, Bacteroides sp) What major adve rse e ffe ct is clindamycin associate d with? Pseudomembranous colitis (due to C. difficile) What is the me chanism of action of macrolide antibiotics? Binds to 5OS ribosomal subunit to inhibit translocation of peptidyl-tRNA from acceptor to donor site, thereby inhibiting bacterial protein synthesis Give e xample s of macrolide antibiotics: Erythromycin; clarithromycin; azithromycin; telithromycin (a ketolide that is structurally related to the macrolides) Are macrolide antibiotics bacte ricidal or bacte riostatic? Primarily bacteriostatic Why is a single dose of az ithromycin as e ffe ctive as a 7-day course of doxycycline for chlamydial infe ctions? Azithromycin has a very long half-life of 68 hours. Which of the macrolide antibiotics is re lative ly fre e of drug-drug inte ractions? Azithromycin Which macrolide re late d antibiotic can cause he patotoxicity and blurre d vision? T elithromycin What are the clinical use s of te lithromycin? Respiratory tract infections Give e xample s of how bacte ria may be come re sistant to macrolide antibiotics: Alteration of binding sites on the 5OS ribosomal subunit; reduced permeability of cell membrane; active efflux; production of esterases by bacteria that hydrolyze the drug What are the adve rse e ffe cts of e rythromycin? Nausea; vomiting; diarrhea; anorexia; hepatitis; drug-drug interactions (cytochrome P-450 inhibitor) What organisms doe s clarithromycin cove r that e rythromycin doe s not? Mycobacterium avium complex (MAC), M. leprae, Toxoplasma gondii Which macrolide antibiotic is safe in pre gnancy? Azithromycin What adve rse e ffe ct is cause d by e rythromycin give n to infants le ss than 6 we e ks of age for pe rtussis? Hypertrophic pyloric stenosis What is the me chanism of action of te tracycline antibiotics? Binds to 3OS ribosomal subunit to inhibit the attachment of aminoacyl-tRNA to its acceptor site, thereby inhibiting bacterial protein synthesis Give e xample s of te tracycline antibiotics: T etracycline; minocycline; doxycycline; demeclocycline; methacycline What is de me clocycline use d for? Syndrome of inappropriate antidiuretic hormone (SIADH) via inhibition of antidiuretic hormone (ADH) receptors in the renal collecting ducts Are te tracycline antibiotics bacte ricidal or bacte riostatic? Primarily bacteriostatic What are the adve rse e ffe cts of te tracycline antibiotics? Nausea; vomiting; diarrhea; Fanconi syndrome (outdated tetracyclines); phototoxicity; hepatotoxicity; vestibular toxicity; superinfection Why are te tracycline antibiotics contraindicate d in childre n? T ooth enamel dysplasia; permanent discoloration of teeth; decreased bone growth via chelation with calcium salts O ral absorption of te tracycline antibiotics may be de cre ase d by which multivale nt cations? Iron; calcium; magnesium; aluminum Give e xample s of how bacte ria may be come re sistant to te tracycline antibiotics: Efflux pumps or impaired influx; bacterial production of proteins that decrease binding of tetracyclines to ribosome; enzymatic inactivation Give an e xample of a glycylcycline antibiotic (de rivative of te tracycline s):

T igecycline Is tige cycline e ffe ctive against MRSA? Yes Is tige cycline a substrate for the e fflux pump me chanism of re sistance to te tracycline s? No What is the me chanism of action of chloramphe nicol? Binds to 5OS ribosomal subunit to inhibit peptidyltransferase, thereby inhibiting bacterial protein synthesis What are the adve rse e ffe cts of chloramphe nicol? Gray baby syndrome in neonates (hypotension, ashen discoloration, vomiting, flaccidity); nausea, vomiting, diarrhea in adults, aplastic anemia; drug-drug interactions (CYP450 inhibitor) Give e xample s of stre ptogramin antibiotics: Quinupristin; dalfopristin What is the me chanism of action of stre ptogramin antibiotics? Binds 5OS ribosomal subunit to inhibit the attachment of aminoacyl-tRNA to its acceptor site, thereby inhibiting bacterial protein synthesis. Specifically, with the fixed dose combination of dalfopristin/quinupristin, dalfopristin distorts the ribosome promoting quinupristin binding. T his blocks the aminoacyl-rRNAs from binding to the ribosome and therefore the transpeptidase reaction. What is the spe ctrum of action of stre ptogramin antibiotics? MRSA; vancomycin-resistant S. aureus (VRSA); vancomycin-resistant Enterococcus faecium (not Enterococcus fecalis) What are the adve rse e ffe cts of the stre ptogramin antibiotics? Arthralgias; myalgias; drug-drug interactions (CYP450 inhibitor) Give an e xample of an oxaz olidinone antibiotic: Linezolid What is the me chanism of action of line z olid? Binds to 5OS ribosomal subunit to prevent formation of initiation complex, thereby inhibiting bacterial protein synthesis What are the adve rse e ffe cts of line z olid? Nausea; vomiting; diarrhea; headache; bone marrow suppression (primarily thrombocytopenia) after 2 weeks of use; weak reversible inhibitor of (monoamine oxidase) MAOA and MAOB; lactic acidosis; peripheral neuropathy; optic neuritis What is the spe ctrum of action of line z olid? Gram-positive organisms such as MRSA; VRSA; vancomycin-resistant E. faecium and E. fecalis Give an e xample of a cyclic lipope ptide antibiotic: Daptomycin What is the me chanism of action of daptomycin? Binds to components of the bacterial cell membrane and causes rapid intracellular depolarization, thereby inhibiting DNA, RNA, and protein synthesis What is the spe ctrum of action of daptomycin? MRSA; VRSA; vancomycin-resistant E. faecium and E.fecalis, therefore daptomycin is an effective alternative to vancomycin Which antibiotic, that works by inhibiting prote in synthe sis, can also be use d in patie nts to incre ase GI motility? Erythromycin (activates motilin receptors) Give the me chanism of action for e ach of the following first-line antitube rculosis me dications: Rifampin Inhibition of DNA-dependent RNA polymerase Isoniaz id Inhibition of mycolic acid synthesis Pyraz inamide Unknown; activated by susceptible bacterial strains which in turn lowers pH of the surrounding environment Ethambutol Inhibition of RNA synthesis Ethambutol Which of the pre vious four antitube rculosis me dications is bacte riostatic? Give the adve rse e ffe cts for e ach of the following antitube rculosis me dications: Rifampin Flu-like syndrome; hepatitis; elevated liver function tests (LFTs); drug-drug interactions (cytochrome P-450 inducer); proteinuria; thrombocytopenia; redorange discoloration of tears, sweat, urine Isoniaz id Drug-induced systemic lupus erythematosus (SLE); hepatitis; peripheral neuropathy; hemolytic anemia in G6PD deficiency; seizures Pyraz inamide Phototoxicity; increased porphyrin synthesis; hepatitis; arthralgias; myalgias; hyperuricemia Ethambutol Optic (retrobulbar) neuritis; decreased visual acuity; red-green color blindness; hyperuricemia How can isoniaz id-induce d pe riphe ral ne uropathy be pre ve nte d? Supplementation of vitamin B6 (pyridoxine)

ANTIFUNGAL AGENTS Name two me dications in the polye ne antifungal drug class: Amphotericin B; nystatin What is the me chanism of action of the polye ne antifungals? Forms artificial pores by binding to ergosterol in fungal membranes, thereby disrupting membrane permeability How do fungi be come re sistant to polye ne antifungals? Reduction in the amount of membrane ergosterol What type s of fungi are affe cte d by amphote ricin B? Candida; Aspergillus; Histoplasma; Cryptococcus; Rhizopus; Sporothrix Amphote ricin B is syne rgistic with what othe r antifungal drug in the tre atme nt of candidal and cryptococcal infe ctions? Flucytosine Flucytosine is conve rte d by fungal cytosine de aminase to what active compound? 5-Fluorouracil which is subsequently converted selectively in fungal cells into two other compounds which inhibit DNA and RNA synthesis Doe s amphote ricin B have good ce ntral ne rvous syste m (CNS) pe ne tration? No, amphotericin B must be given via intrathecal route if adequate cerebrospinal fluid (CSF) levels are warranted Which polye ne antifungal is said to cause a “shake and bake ” adve rse re action? IV infusion of amphotericin B can cause fevers, chills, rigors, and hypotension, the so called “ shake and bake” adverse reaction. How can the “shake and bake ” adve rse re action cause d by amphote ricin B be pre ve nte d? Test dose prior to initiation of intravenous therapy; pretreatment with antihistamines, nonsteroidal anti-inflammatory drugs (NSAIDs), meperidine, and glucocorticoids

Pre tre atme nt with me pe ridine prior to amphote ricin B infusion is use d to pre ve nt what spe cific adve rse re action? Rigors What is the major dose -limiting adve rse e ffe ct of amphote ricin B? Nephrotoxicity (also causes anemia via decreased erythropoietin, hypokalemia, hypomagnesemia, decreased glomerular filtration rate [GFR], renal tubular acidosis) How can the ne phrotoxicity cause d by amphote ricin B be minimiz e d? Load with normal saline solution; use of amphotericin B in combination with another medication so that the dose of amphotericin B can be decreased; use of liposomal amphotericin B formulations Give e xample s of the az ole antifungal drug class: Fluconazole; itraconazole; ketoconazole; voriconazole; miconazole; clotrimazole; posaconazole; ravuconazole What is the me chanism of action of the az ole antifungals? Prevents the synthesis of ergosterol from lanosterol by inhibiting cytochrome P-450-dependent 14-α-demethylation Fluconaz ole is the drug of choice for what type s of fungal infe ctions? Mucocutaneous candidiasis; coccidioidomycosis; prevention and treatment of cryptococcal meningitis Itraconaz ole is the drug of choice for what type s of fungal infe ctions? Sporotrichoses; blastomycoses Which antifungal is the drug of choice for paracoccidioide s infe ctions? Ketoconazole What antifungal can be formulate d into a topical shampoo ge l to tre at de rmatophytosis of the scalp? Ketoconazole What is anothe r name for de rmatophytosis of the scalp? T inea capitis What adve rse e ffe ct of ke toconaz ole is also an adve rse e ffe ct of spironolactone ? Gynecomastia (via inhibition of androgen synthesis) Voriconaz ole can be use d to tre at what type s of fungal infe ctions? Invasive aspergillosis; invasive candidiasis; candidemia Is absorption of ke toconaz ole incre ase d or de cre ase d by alkaliniz ation of gastric pH? It is decreased. Do not use antacids in combination with ketoconazole. Some physicians may te ll patie nts to drink what in orde r to e nhance the oral absorption of ke toconaz ole ? Coca-Cola, Pepsi-Cola, etc. Carbonated beverages that contain phosphoric and/or citric acid are acidic and therefore enhance oral absorption. The Inte rnational Normaliz e d Ratio (INR) of a patie nt stabiliz e d on warfarin the rapy will be incre ase d or de cre ase d whe n an az ole antifungal me dication is initiate d? Increased (azole antifungals inhibit hepatic cytochrome P-450 enzymes thereby inhibiting the metabolism and increasing the blood levels of warfarin) Which laboratory te sts may be come e le vate d in patie nts be ing tre ate d with az ole antifungals? Liver function tests (LFT s) used to monitor for hepatotoxicity Which antifungal me dications act by inhibiting the synthe sis of β-(l-3)-d-glucan? Caspofungin; anidulafungin; micafungin β-(1-3)-d-Glucan is an inte gral part of the fungal ce ll me mbrane or fungal ce ll wall? Fungal cell wall Caspofungin can be use d to tre at what type s of fungal infe ctions? Invasive aspergillosis; invasive candidiasis; candidemia Which antifungal, active only against de rmatophyte s, acts by de positing in ne wly forme d ke ratin and disrupting microtubule structure ? Griseofulvin Grise ofulvin is active against de rmatophyte s whe n use d orally or topically? When used orally What is the major dose -limiting adve rse re action of grise ofulvin? Hepatotoxicity Grise ofulvin is contraindicate d in patie nts with which dise ase ? Acute intermittent porphyria Name the thre e major de rmatophyte s: 1. Epidermophyton 2. T richophyton 3. Microsporum Which antifungal inhibits e rgoste rol synthe sis by inhibiting squale ne e poxidase ? T erbinafine Te rbinafine is use d to tre at what type s of fungal infe ctions? Dermatophytic infections O ral te rbinafine is use d to tre at what spe cific type s of de rmatophytic infe ctions? Onychomycosis of the toenail; onychomycosis of the fingernail O ral te rbinafine can cause what major dose -limiting adve rse re action? Hepatotoxicity

ANTIVIRAL AGENTS What e nz yme adds the first phosphate to acyclovir? Viral thymidine kinase True or False ? Monophosphorylate d acyclovir is conve rte d to the triphosphate form by viral e nz yme s. False (host cell kinases are responsible for these reactions) How doe s acyclovir triphosphate work as an antiviral age nt? Inhibits viral DNA replication by competing with deoxyguanosine triphosphate for viral DNA polymerase; incorporated into the viral DNA molecule and acts as a chain terminator How doe s acyclovir triphosphate work as a chain te rminator? Lacks the ribosyl 3’ hydroxyl group How do viruse s be come re sistant to acyclovir? Downregulation of viral thymidine kinase; lacking thymidine kinase altogether; altered specificity of viral thymidine kinase; altered specificity of viral DNA polymerase Acyclovir is e ffe ctive in tre ating which virus type s? Herpes simplex virus (HSV) 1 and 2; varicella-zoster virus (VZV). Acyclovir is 10 × more potent against HSV than VZV

Is acyclovir e ffe ctive in tre ating posthe rpe tic ne uralgia? No (only effective against acute neuritis) What is the oral bioavailability of acyclovir? 15%-30%. T here is minimal systemic distribution after topical application. What is the half-life of acyclovir in adults? 2.5-3 hours Why is it ne ce ssary to maintain ade quate hydration in patie nts re ce iving IV acyclovir the rapy? T o prevent crystalluria or interstitial nephritis. Slow infusion additionally helps to avoid these adverse reactions. What is the name of the prodrug that is conve rte d to acyclovir and L-valine by first-pass me tabolism? Valacyclovir What is the advantage of valacyclovir ove r acyclovir? Higher oral bioavailability of 54%-70% Famciclovir is a prodrug that is me taboliz e d to what active me tabolite ? Penciclovir What is the bioavailability of pe nciclovir afte r oral administration of famciclovir? 70% Is famciclovir e ffe ctive in viral strains re sistant to acyclovir se condary to mutate d DNA polyme rase ? Yes Is famciclovir e ffe ctive in viral strains re sistant to acyclovir se condary to lack of thymidine kinase ? No What is the me chanism of action of ganciclovir? Phosphorylated to a substrate which competitively inhibits binding of deoxyguanosine triphosphate to DNA polymerase, thereby inhibiting viral DNA synthesis Doe s ganciclovir have chain-te rminating ability? No Ganciclovir is e ffe ctive in tre ating which virus type s? HSV; VZV; human herpes virus (HHV)-6 and 8; cytomegalovirus (CMV). Activity against CMV is 100 × greater than acyclovir. It may be used intraocularly for CMV retinitis. What is the advantage of valganciclovir ove r its pare nt drug ganciclovir? Valganciclovir (the valine ester) has up to 60% better oral availability than ganciclovir. What is ganciclovir’s dose -limiting adve rse e ffe ct? Myelosuppression; thrombocytopenia; anemia; leukopenia What are the adve rse e ffe cts of ganciclovir? Crystalluria; mucositis; rash; fever; hepatotoxicity; seizures; diarrhea; nausea; hematotoxicity What is cidofovir use d for? CMV retinitis most commonly. It also has activity against HSV-1 and 2, varicella zoster virus (VZV), Epstein-Barr virus (EBV), HHV-6 and 8, adenovirus, poxviruses, polyomaviruses, and human papilloma virus (HPV). What antiviral age nt is a pyrophosphate analogue that acts as an inhibitor of viral RNA and DNA polyme rase and HIV re ve rse transcriptase ? Foscarnet Doe s foscarne t re quire activation by thymidine kinase ? No Foscarne t is e ffe ctive in tre ating which virus type s? Acyclovir-resistant HSV and VZV; ganciclovir-resistant CMV Doe s foscarne t cause he matotoxicity? Yes What are the major adve rse e ffe cts of foscarne t? Hematotoxicity; fever; seizures; electrolyte abnormalities; nausea; vomiting; diarrhea What type s of e le ctrolyte abnormalitie s can foscarne t cause ? Hyper- or hypocalcemia; hyper- or hypomagnesemia; hyper- or hypophosphatemia; hypokalemia True of False ? Amantadine is e ffe ctive in tre ating both influe nz a A and B. False (it is effective against the influenza A virus only) What is the antiviral me chanism of action of amantadine ? Blocks the uncoating of influenza A virus, thereby preventing penetration of the virus into host cells What othe r noninf e ctious dise ase proce sse s is amantadine use d for? Parkinson disease; drug-induced extrapyramidal symptoms. It also increases dopamine levels in the synaptic cleft by either inhibiting reuptake into presynaptic neurons or by increasing release from presynaptic neurons. It may have anticholinergic effects. What are the adve rse e ffe cts of amantadine ? Seizures; insomnia; nervousness; livedo reticularis; orthostatic hypotension; peripheral edema; dry nose; xerostomia; nausea; anorexia What is live do re ticularis? A purplish discoloration of the skin caused by dilation of capillaries and venules secondary to stasis or changes in underlying blood vessels Name two drugs that inhibit ne uraminidase of both influe nz a A and B, the re by de cre asing the like lihood of viral pe ne tration into host ce lls: Oseltamivir; zanamivir Which ne uraminidase inhibitor has an oral inhalational route of administration? Zanamivir Ribavirin is e ffe ctive in tre ating which virus type s? Respiratory syncytial virus (RSV); influenza A and B; hepatitis C virus (HCV) Ribavirin is use d in conjunction with what othe r drug to tre at HCV? Interferon-alpha (IFN-α) What are the adve rse e ffe cts of ribavirin? Anemia; neutropenia; thrombocytopenia; anorexia; headache; conjunctivitis; nausea; pharyngitis; lacrimation; alopecia; rash; flu-like syndrome; teratogenicity (pregnancy category X) Name the major adve rse e ffe cts of IFN-α: Flu-like symptoms; depression; alopecia; insomnia; nausea What is the name of the only available nucle otide re ve rse transcriptase inhibitor? T enofovir Give e xample s of nucle oside re ve rse transcriptase inhibitors (NRTIs): Zidovudine (AZT ); stavudine (d4T ); lamivudine (3T C); didanosine (ddl); abacavir (ABC); emtricitabine (FT C) What adve rse e ffe ct(s) are associate d with all NRTIs? Lactic acidosis with hepatic steatosis What is the ge ne ral me chanism of action of NRTIs? Interference with HIV viral RNA-dependent DNA polymerase resulting in inhibition of HIV viral replication What two NRTIs are thymidine analogs? Zidovudine; stavudine

What NRTI is an ade nosine analog? Didanosine What NRTI is a guanosine analog? Abacavir What two NRTIs are cytosine analogs? 1. Emtricitabine 2. Lamivudine Which NRTI should not be re challe nge d if hype rse nsitivity is e xpe cte d? Abacavir (symptoms include fever, rash, nausea, vomiting, malaise, fatigue, and respiratory dysfunction) Which NRTI can cause hype rurice mia? Didanosine Which two NRTIs can cause pancre atitis? 1. Didanosine 2. Stavudine (dose-limiting effect) Which two NRTIs can cause pe riphe ral ne uropathy? 1. Didanosine 2. Stavudine (dose-limiting effect) What is didanosine ’s dose -limiting adve rse e ffe ct? Pancreatitis What is stavudine ’s dose -limiting adve rse e ffe ct? Peripheral neuropathy What are the main adve rse e ffe cts of AZT? Anemia and neutropenia (dose-limiting effect/potentiated by vitamin B12); headache; nausea; insomnia; body aches; lactic acidosis What is the dose -limiting adve rse e ffe ct of AZT? Hematotoxicity What antire troviral age nt can cause Fanconi syndrome ? T enofovir. Fanconi syndrome is impairment of the proximal tubule resulting in increased phosphate and calcium losses. What NRTI can cause alte re d LFTs, lipoatrophy, hype rlipide mia, and asce nding pare sis? Stavudine Name thre e nonnucle oside re ve rse transcriptase inhibitors (NNRTIs): 1. Delavirdine 2. Efavirenz 3. Nevirapine What is the me chanism of action of e favire nz ? Binds directly to reverse transcriptase and blocks the RNA-and DNA-dependent DNA polymerase activity of reverse transcriptase What is the class adve rse e ffe ct(s) of the NNRTIs? Rash Which NNRTI can cause he patitis and he patic ne crosis? Nevirapine Which NNRTI can cause abnormal dre ams, impaire d conce ntration, diz z ine ss, and alte re d LFTs? Efavirenz Which NNRT can produce a false -positive urine te st for Cannabis? Efavirenz (in about 50% of patients) Do NNRTIs re quire me tabolic activation? No Do NRTIs re quire me tabolic activation? Yes What is the name of the drug that inhibits fusion of the HIV-1 virus with CD4 ce lls by binding to and blocking the conformational change in gp41 re quire d for me mbrane fusion and e ntry into CD4 ce lls? Enfuvirtide (fusion inhibitor) What are the adve rse e ffe cts of e nfuvirtide ? Pain, induration, erythema, and nodules at the injection site; nausea; vomiting; diarrhea; fatigue Give e xample s of prote ase inhibitors: Atazanavir; indinavir; lopinavir; fosamprenavir; nelfinavir; ritonavir; saquinavir; tipranavir; amprenavir; darunavir What adve rse e ffe ct(s) are associate d with the prote ase inhibitors? Hepatotoxicity; fat maldistribution; insulin resistance; osteonecrosis; increased bleeding in hemophiliac patients Are prote ase inhibitors me taboliz e d by P-450 e nz yme s? Yes Do prote ase inhibitors inhibit or induce P-450 e nz yme s? T hey inhibit P-450 enzymes. What is HIV prote ase re sponsible for? Cleaves the Gag-Pol polyprotein of HIV (the gag region of the gene codes for structural proteins whereas the Pol region of the gene codes for protease, reverse transcriptase, and integrase) The combination of ataz anavir and indinavir can cause what possible adve rse e ffe ct? Hyperbilirubinemia Patie nts with sulfonamide alle rgy should use caution whe n taking which two prote ase inhibitors? 1. T ipranavir

2. Fosamprenavir Which prote ase inhibitor can cause an alte re d taste se nsation? Ritonavir Which two prote ase inhibitors can cause asthe nia (lack of stre ngth)? 1. Lopinavir 2. Ritonavir Which prote ase inhibitor can cause kidne y stone formation? Indinavir Which prote ase inhibitor can cause numbne ss around the mouth? Ritonavir What type of prophylaxis is give n to a pe rson stuck with a pote ntially HIV contaminate d ne e dle ? Zidovudine and lamivudine for 1 month (protease inhibitor should be added for high-risk exposures) How is mate rnal-fe tal HIV transmission pre ve nte d in mothe rs? Zidovudine beginning at 14-34 weeks’ gestation and continued until start of labor; during labor and delivery, zidovudine until the umbilical cord is clamped How is mate rnal-fe tal HIV transmission pre ve nte d in ne onate s? Zidovudine started 8-12 hours after birth and continued for 6 weeks

ANTIPROTOZOAL AGENTS Ame biasis is ge ne rally tre ate d with what drug combination? Metronidazole and diloxanide What is the me chanism of action of me tronidaz ole ? Mixed amebicide (effective against both luminal and systemic forms of disease); nitro group of metronidazole acts as an electron acceptor, thereby forming reduced cytotoxic compounds that lead to inhibition of protein synthesis and DNA strand breakage What are the adve rse e ffe cts of me tronidaz ole ? Nausea; vomiting; metallic taste sensation; disulfiram-like reaction Give e xample s of me dications that can cause a disulfiram-like re action: Metronidazole; chlorpropamide; cefotetan; cefamandole; cefoperazone What is the antimicrobial spe ctrum of me tronidaz ole ? Entamoeba histolytica; Giardia lamblia; Trichomonas vaginalis; bacterial anaerobes; C. difficile Me tronidaz ole is contraindicate d in which trime ste r of pre gnancy? First trimester during organogenesis since teratogenicity has not been effectively ruled out Can me tronidaz ole cross the blood-brain barrie r (BBB)? Yes What is the me chanism of action of diloxanide ? Luminal amebicide (effective against luminal forms of disease) used in the treatment of asymptomatic amoebic cyst passers What are the adve rse e ffe cts of diloxanide ? Contraindicated in pregnancy and children less than 2 years old; dry mouth; pruritus; flatulence What are the four spe cie s of Plasmodium 1. 2. 3. 4.

Malariae Falciparum Vivax Ovale

What is the most dange rous/life -thre ate ning Plasmodium spe cie s? Falciparum What are the thre e stage s of the malarial parasite life cycle primarily targe te d by antimalarial drugs? 1. Erythrocytic stage 2. Exoerythrocytic stage 3. Gametocytic stage What is the olde st antimalarial drug still in use ? Quinine (“ Jesuit Bark”) Q uinine has be e n re place d by which antimalarial drug? Chloroquine (more potent and less toxic than quinine) What drugs are e ffe ctive against the e xoe rythrocytic forms of malaria? Primaquine; atovaquone + proguanil (Malarone) What drug is e ffe ctive against the game tocytic (he patic) forms of malaria? Primaquine Is primaquine e ffe ctive against e rythrocytic forms of malaria? No Give e xample s of antimalarial drugs that are e ffe ctive against the e rythrocytic forms of malaria: Hydroxychloroquine; chloroquine; mefloquine; pyrimethamine; quinine; atovaquone + proguanil; artemisinin What drug is e ffe ctive against re lapsing forms of P. vivax and P. ovale malarias? Primaquine. T o prevent recurrence of infection, hepatic forms of these parasites must be eliminated. What are the adve rse e ffe cts of primaquine ? Hemolytic anemia in patients with G6PD deficiency; methemoglobinemia; agranulocytosis What is the drug of choice for acute attacks of malaria cause d by chloroquine -se nsitive strains of P. falciparum and P. vivax? Chloroquine Doe s chloroquine have a large or small Vd? Large

What is the me chanism of action of chloroquine ? It concentrates within parasite food vacuoles and raises pH leading to inhibition of growth; inhibits hemoglobin metabolism and utilization by parasites; concentrates within parasite vacuoles and raises pH leading to inhibition of growth; binds to ferriprotoporphyrin IX leading to membrane damage; inhibits DNA and RNA polymerase What contine nts contain the large st re positorie s of chloroquine -re sistant P. falciparuml Africa, Asia What are the adve rse e ffe cts of chloroquine ? ECG changes (quinidine-like effects); headaches; pruritus; mucosal pigmentary changes (blue-black); photosensitivity; nausea; vomiting; diarrhea; aplastic anemia; agranulocytosis; neutropenia; thrombocytopenia; retinopathy; tinnitus; reduced hearing Chloroquine is contraindicate d in patie nts with what dise ase state s? Porphyria; psoriasis What are the major adve rse e ffe cts of quinine ? Cinchonism (nausea, vomiting, diarrhea, tinnitus, vertigo); hemolytic anemia; digoxin toxicity What othe r me dication can cause cinchonism? Quinidine Is quinine acidic or basic? Basic How can the urinary e xcre tion of quinine (or chloroquine ) be e nhance d? Acidification of the urine Name two ne we r antimalarials that are che mically re late d to quinine : 1. Halofantrine 2. Lumefantrine What can be use d to acidify the urine ? Ammonium chloride What is the me chanism of action of pyrime thamine ? Inhibits nucleic acid and protein metabolism in the parasites; plasmodial dihydrofolate reductase (DHFR) inhibitor The antimalarial e ffe cts of pyrime thamine can be pote ntiate d by combining it with which drugs? Sulfonamides (synergistic blockade of folic acid synthesis). How is folate -de ficie nt me galoblastic ane mia re ve rse d in patie nts taking pyrime thamine ? Leucovorin What is the me chanism of action of le ucovorin? As a reduced form of folic acid, leucovorin supplies human cells with the necessary cofactor blocked by DHFR inhibitors. What kind of compounds are the antimalarial drug arte misinin and its de rivative s? Sesquiterpene lactones (the active ingredient in a 2000-year-old Chinese herb—Qing Hao). What is the antimalarial me chanism of action of arte misinin? It is thought that it is activated by heme to irreversibly decompose generating free radicals that form adducts mostly with proteins and lipids. Give e xample s of antimalarial drugs use d in arte misinin combination the rapie s (ACT): Pyrimethamine/sulfadoxine (Fansidar), mefloquine, amodiaquine What is the name of the most promising antimalarial vaccine ? RT S-S/AS02A or Mosquirix Which ge ne tic dise ase s/conditions may he lp prote ct against malarial infe ctions? Sickle cell trait; G6PD deficiency Trypanosoma cruzi is re sponsible for causing what dise ase ? American trypanosomiasis (Chagas disease) T. brucei gambiense and T. brucei rhodesiense are re sponsible for causing what dise ase ? African trypanosomiasis (sleeping sickness) What drug is use d as a suppre ssive age nt in patie nts with acute T. cruzi infe ctions? Nifurtimox What is the me chanism of action of nifurtimox? Forms intracellular oxygen-free radicals which are toxic to the parasite because of its lack of catalase (oxygen radical scavenger) What drug is use d to tre at African sle e ping sickne ss with CNS involve me nt? Eflornithine for West African trypanosomiasis; melarsoprol for East African trypanosomiasis What are the adve rse e ve nts of me larsoprol? Hypersensitivity; abdominal pain; vomiting; hemolytic anemia in patients with G6PD deficiency; encephalopathy What two drugs are use d in the e arly stage s of African sle e ping sickne ss? 1. Pentamidine (first choice for West African sleeping sickness) 2. Suramin (first choice for East African sleeping sickness) Doe s pe ntamidine cross the BBB? No. T herefore it cannot be used for late trypanosomiasis with CNS involvement. What are the two route s of administration of pe ntamidine ? 1. IV 2. Aerosol What fungus is pe ntamidine commonly use d to tre at? Pneumocystis carinii What drug combination is use d for prophylaxis against P. carinii? T rimethoprim-sulfamethoxazole What is the tre atme nt of choice for T. gondii? Pyrimethamine + sulfadiazine How do humans be come infe cte d with T. gondii? Ingestion of undercooked, infected meat; contact with infected cats Can pre gnant mothe rs transmit T. gondii to the fe tus?

Yes (remember T ORCH syndromes) What are the thre e type s of le ishmaniasis infe ctions? 1. Visceral 2. Cutaneous 3. Mucocutaneous What is the drug of choice for tre ating le ishmaniasis? Stibogluconate (pentavalent antimony compound)

ANTIHELMINTHIC AGENTS What is anothe r name for the ne matode s? Roundworms What is anothe r name for the tre matode s? Flukes What is anothe r name for the ce stode s? T apeworms What drug is commonly use d to tre at tre matode infe ctions? Praziquantel What is the me chanism of action of praz iquante l? Increases cell permeability to calcium, thereby increasing contractions with subsequent paralysis of musculature What is the me chanism of action of me be ndaz ole ? It irreversibly blocks glucose uptake; inhibits microtubule polymerization What are the adve rse e ffe cts of me be ndaz ole ? Diarrhea; abdominal pain; contraindicated during pregnancy What is the me chanism of action of albe ndaz ole ? It interferes with microtubule polymerization; inhibits adenosine triphosphate (AT P) production thereby depleting energy availability What is the me chanism of action of thiabe ndaz ole ? Inhibits helminth-specific mitochondrial fumarate reductase What type s of cutane ous adve rse e ffe cts are cause d by thiabe ndaz ole ? Stevens-Johnson syndrome; erythema multiforme What is the me chanism of action of pyrante l? Depolarizing neuromuscular blocker thereby causing paralysis of musculature What type s of he lminths are affe cte d by praz iquante l? T rematodes; cestodes What type s of he lminths are affe cte d by me be ndaz ole ? Nematodes What type s of he lminths are affe cte d by pyrante l? Nematodes What is the drug of choice for tre ating Enterobius vermicularis? Mebendazole What is the common name for E. vermicularis? Pinworm What is the drug of choice for tre ating Onchocerca volvulus (onchoce rciasis or rive r blindne ss)? Ivermectin What is the me chanism of action of ive rme ctin? Acts at helminthic gamma-aminobutyric acid (GABA) receptors, thereby enhancing influx of chloride and causing hyperpolarization and paralysis Why doe s onchoce rciasis pote ntially le ad to blindne ss? A bacteria (Wolbachia sp.) that colonizes many parasitic worms, including the nematode that causes onchocerciasis, is an important factor in the inflammatory response that leads to blindness. What is anothe r name for onchoce rciasis? River blindness What drug is commonly use d to tre at ce stode infe ctions? Niclosamide What is the me chanism of action of niclosamide ? Inhibits mitochondrial phosphorylation of ADP to AT P, thereby depleting energy availability Is niclosamide active against the ova of ce stode s? No; only active against cestode’s scolex and segments

CLINICAL VIGNETTES A 48-ye ar-old fe male marine animal traine r de ve lops a re ddish granuloma on he r hand. He r past me dical history is significant only for gastroe sophage al re flux dise ase (GERD) which is we ll controlle d with a PPI (ome praz ole ). More ove r, she is ve ry conscious as to maintain fitne ss in ke e ping with he r re quire d scuba lice nsing; thus, she re gularly take s calcium table ts with he r me als. She is diagnose d with a Mycobacterium marinum infe ction and be gins tre atme nt with minocycline . Howe ve r, 6 we e ks afte r tre atme nt he r condition has not re solve d and she has de ve lope d ne w granulomas. What is the most like ly re ason for tre atme nt failure in this patie nt? T his vignette underscores the importance of taking a good patient’s history. Here the patient’s occupation, gender, age, fitness requirements, and medication history are crucial. Her unique occupation gives the patient exposure to microorganisms that are uncommonly causes of infection in the lay human population. In terms of her medications, even supplements have significant side effects and drug interactions, and must be asked about as part of your history. Calcium salts such as those used to increase calcium intake in menopausal women will chelate tetracyclines like minocycline, decreasing their oral bioavailability. T herefore, this patient should have been instructed not to take her calcium supplement tablets 2 hours before or after a dose of her antibiotic. A 76-ye ar-old man is brought to the inte nsive care unit (ICU) afte r be ing found unre sponsive on the floor of his home . It is unknown how long the patie nt had re maine d immobile on the floor. Re spiratory e ffort was diminishe d and the patie nt was intubate d and place d on a ve ntilator. The inte rn on call place d the patie nt on a tobramycin ne buliz e r to suppre ss ve ntilator-associate d pne umonia. The ne xt morning, the atte nding physician se e s this and imme diate ly te rminate s this tre atme nt. Why is tobramycin contraindicate d in this patie nt? Prolonged immobility leads to dehydration and muscle breakdown with release of creatine phosphokinase (CPK), both of which can cause acute renal failure. Aminoglycosides, such as tobramycin, are nephrotoxic and will cause further damage to the kidneys. All aminoglycosides should be used with caution in the elderly. Another consideration is ototoxicity. Since this patient is unresponsive, we are unaware of his baseline hearing status. A small amount of damage in an individual

who is already hard of hearing could have dramatic consequences. A 24-ye ar-old fe male me dical stude nt participate s in an e xchange program in rural South Kore a. De spite che moprophylaxis (me floquine ), she de ve lops cyclic shaking chills and fe ve r and is diagnose d with malaria. Tre atme nt is be gun with chloroquine , but the stude nt fails to re spond. Re sistance to chloroquine is suspe cte d, and the tre atme nt is switche d to quinidine plus a te tracycline with comple te re solution of he r symptoms. Two months afte r he r re turn to the Unite d State s, she has a re occurre nce of the malarial symptoms. Why has this the rapy faile d to re solve he r illne ss? T he student has most likely contracted P. vivax, an endemic strain of malaria on the Korean peninsula. Chloroquine (and mefloquine) resistance is an increasing problem worldwide. A quinidine plus tetracycline (eg, doxycycline) combination is an effective treatment regimen for eradication of chloroquine-resistant malaria. However, it is important to remember that P. vivax has liver repositories which the above antimalarial will not treat effectively. Reactivation of these hypnozoites can lead to recurrence of infection months to years later. Once the original acute attack P. vivax malaria is resolved, primaquine is the drug of choice to eradicate the liver forms and must be added to the drug regimen in cases of P. vivax or P. ovale. Note: Licensing exams will not expect you to determine whether a specific geographic area is endemic with chloroquine resistant strains of malaria. You should know treatment alternatives for drug-resistant strains, as well as when addition of primaquine is necessary to eradicate hepatic repositories of the malaria parasite (ie, therapy is specie-specific). During the se cond we e k of a trip to Be liz e , a trave le r e xpe rie nce d some diarrhe a, which was sufficie nt to re mind him of pre vious admonitions about consuming food dispe nse d by stre e t ve ndors. Fortunate ly, his symptoms se e m to subside and he re cove re d in a fe w days. About a month afte r his re turn, the trave le r de ve lope d se ve re pain in the right uppe r quadrant of his abdome n. Whe n the abdominal pain pe rsiste d, he we nt to a gastroe nte rologist. The gastroe nte rologist pe rforme d an x-ray study of the inte stine afte r a barium e ne ma, a CT scan, and a se rological te st for E. histolytica. The re sults of the se te sts re ve ale d pse udopolyps consiste nt with inflammatory bowe l dise ase , the CT scan showe d absce sse s in the live r and a he magglutination tite r of 1:2000 for E. histolytica. What antiprotoz oal drug should be give n to this patie nt? T he drug of choice for this active amebic infection picture is metronidazole. It is absorbed rapidly from oral doses with a half-life in serum of about 8 hours. It has potent activity against E. histolytica. T he drug is well tolerated and adverse effects are not common, but nausea, headaches, and dry mouth can occur.

CHAPTER 3 Cancer Chemotherapeutic Agents BAS IC PHARMACOLOGY OF CANCER CHEMOTHERAPEUTICS According to the log-kill hypothe sis, doe s the cytotoxic action of anticance r drugs follow first-orde r or se cond-orde r kine tics? First-order kinetics With first-orde r kine tics, is it a fixe d amount or fixe d pe rce ntage of tumor ce lls that are kille d by cance r che mothe rape utic age nts? Fixed percentage If a che mothe rapy tre atme nt le ads to a 4 log-kill re duction, the n how many tumor ce lls would re main if the re we re 10 10 tumor ce lls to be gin with? 10 6 (or 10 10/10 4) Give a brie f summary of what happe ns during e ach of the following phase s of the ce ll cycle : G0 Cells are not actively dividing (resting state). G1 Enzymes and proteins required for DNA replication are synthesized. S Replication of DNA G2 Enzymes and proteins required for mitosis are synthesized. M Mitosis occurs. The ratio of prolife rating (malignant) ce lls to nonprolifcrating (G 0) ce lls is also known as what? Growth fraction Are tumor ce lls more susce ptible to cance r che mothe rape utic age nts whe n the y are active ly dividing or whe n the y are dormant? Actively dividing. T hus, tumor cells which are dormant may not be sufficiently susceptible to the effects of cancer chemotherapeutic agents. What is the de finition of a ce ll-cycle spe cific (CCS) cance r che mothe rape utic age nt? An agent that kills actively dividing cells (cells currently going through the cell cycle) Give e xample s of normal/nonmalignant ce lls in the body that normally are unde rgoing rapid prolife ration: Bone marrow cells; GI mucosal cells; hair cells. T hus, the common side effects of chemotherapy include myelosuppression, GI disturbances, and alopecia. P-glycoprote in is an ATP-de pe nde nt me mbrane (e fflux) transporte r that is re sponsible for what? Pumping drugs out of cells (responsible for multidrug resistance of chemotherapeutic agents) Give e xample s of cance r che mothe rape utic age nts that are commonly associate d with e ach of the following adve rse e ffe cts: Cardiotoxicity; dilate d cardiomyopathy Doxorubicin Pulmonary fibrosis; pne umonitis Bleomycin Stomatitis; e sophagitis Methotrexate; 5-fluorouracil; dactinomycin He morrhagic cystitis Cyclophosphamide; ifosfamide He morrhagic diathe sis Pe riphe ral ne uropathy; ne urotoxicity Plicamycin Ne phrotoxicity Vincristine Alle rgic re actions Cisplatin He patotoxicity Etoposide; L-asparaginase Pancre atitis 6-Mercaptopurine; busulfan; Cyclophosphamide Cutane ous toxicity (hand-foot syndrome ) L-Asparaginase 5-Fluorouracil Disulfiram-type re actions Procarbazine What is the name of the antidote that binds to and inactivate s the toxic me tabolite s re sponsible for cisplatin-induce d ne phrotoxicity? Amifostine What is the name of the cyclophosphamide and ifosfamide urotoxic me tabolite that is re sponsible for causing he morrhagic cystitis? Acrolein What is the name of the antidote that binds to and inactivate s acrole in, the re by pre ve nting he morrhagic cystitis in patie nts re ce iving cyclophosphamide or ifosfamide che mothe rapy? Mesna Which iron che lating age nt is use d to de cre ase the incide nce and se ve rity of doxorubicin-induce d cardiomyopathy in patie nts with me tastatic bre ast cance r who have re ce ive d a life time cumulative doxorubicin dose (300 mg/m 2)? Dexrazoxane Give e xample s of antime tabolite cance r che mothe rape utic age nts: Methotrexate; 5-fluorouracil; cytarabine; fludarabine; 6-thioguanine; 6-mercaptopurine Are the antime tabolite cance r che mothe rape utic age nts CCS? Yes (S phase) What is the me chanism of action of me thotre xate ? Inhibits dihydrofolate reductase (DHFR) What re action doe s DHFR catalyz e ? Conversion of folic acid to tetrahydrofolic acid (active form) What drug is use d as a “re scue me dication” in patie nts taking me thotre xate ? Leucovorin, which acts as an active form of folic acid (replenishing the folate pool) that has bypassed the inhibited DHFR and is more readily taken up by normal cells than by malignant cells

What are the adve rse e ffe cts of me thotre xate ? Stomatitis; bone marrow suppression (BMS); urticaria; alopecia; nausea; vomiting; diarrhea; nephrotoxicity; hepatotoxicity; pulmonary toxicity; neurotoxicity What is the me chanism of action of 5-fluorouracil? Pyrimidine analog that is converted to active 5-FdUMP which inhibits thymidylate synthetase, thereby decreasing the amount of cellular thymidine and subsequent DNA What is the me chanism of action of cytarabine ? Pyrimidine antagonist What is the me chanism of action of both 6-me rcaptopurine and 6-thioguanine ? Purine antagonists What immunosuppre ssive drug be come s active only afte r be ing conve rte d to 6-me rcaptopurine ? Azathioprine Be cause 6-me rcaptopurine is me taboliz e d by xanthine oxidase , its se rum le ve ls may be significantly incre ase d whe n give n concomitantly with what othe r me dication? Allopurinol (xanthine oxidase inhibitor) What e nz yme activate s 6-me rcaptopurine to its corre sponding nucle otide form by adding a ribose phosphate to its structure ? Hypoxanthine-guanine phosphoribosyl transferase (HGPRT ) What are the major adve rse e ffe cts of 6-me rcaptopurine ? Nausea; vomiting; diarrhea; hepatotoxicity; BMS Give e xample s of antitumor antibiotics: Doxorubicin; daunorubicin; dactinomycin; plicamycin; bleomycin; idarubicin Are the antitumor antibiotics CCS? Yes (S-phase) Name thre e anthracycline antitumor antibiotics: 1. Doxorubicin 2. Daunorubicin 3. Idarubicin What is the me chanism of action of the anthracycline antibiotics? Inhibition of DNA topoisomerase II; formation of free radicals (leading to DNA strand scission); DNA intercalation; inhibition of DNA and RNA synthesis Name thre e non-anthracycline antitumor antibiotics: 1. Dactinomycin 2. Bleomycin 3. Mitomycin What is the me chanism of action of ble omycin? Complexes with iron and reacts with oxygen which in turn leads to DNA strand scission Which phase of the ce ll cycle is ble omycin spe cific for? G2 Give e xample s of anticance r alkylating age nts: Cyclophosphamide; ifosfamide; mechlorethamine; nitrosoureas (carmustine, lomustine, streptozotocin); cisplatin; carboplatin Are the anticance r alkylating che mothe rape utic age nts CCS? No What is the me chanism of action of anticance r alkylating age nts? Covalently bind (alkylation) to DNA leading to cross-linked and dysfunctional DNA strands Give e xample s of anticance r mitotic inhibitors: Paclitaxel; docetaxel; vincristine; vinblastine; vinorelbine Are the anticance r mitotic inhibitors CCS? Yes (M phase) What is the me chanism of action of vincristine and vinblastine ? T hey are vinca alkaloids that inhibits the ability of tubulin to polymerize, thereby preventing formation of the microtubule structures needed during mitosis. What adve rse e ffe cts do vincristine and vinblastine have in common? Nausea; vomiting; diarrhea; alopecia; phlebitis; cellulites Are vincristine and vinblastine ve sicants? Yes, they are strong vesicants. Which adve rse e ffe ct is unique to vincristine ? Peripheral neuropathy Which adve rse e ffe ct is unique to vinblastine ? BMS What plant are the vinca alkaloids de rive d from? Periwinkle plant Which plant is paclitaxe l a de rivative of? Needles of the Western or Pacific yew tree What is the me chanism of action of paclitaxe l? Binds to tubulin and increases polymerization and stabilization of the microtubule structure, thereby preventing depolymerization What are the adve rse e ffe cts of paclitaxe l? Neutropenia; alopecia; hypersensitivity reactions How are hype rse nsitivity re actions pre ve nte d in patie nts re ce iving paclitaxe l cance r che mothe rapy? Pretreatment with diphenhydramine and dexamethasone Give two e xample s of e pipodophyllotoxin cance r che mothe rape utic age nts: 1. Etoposide 2. T eniposide What is the me chanism of action of the e pipodophyllotoxin cance r che mothe rape utic age nts? Inhibition of DNA topoisomerase II Give two e xample s of cance r che mothe rape utic age nts that inhibit DNA topoisome rase I:

1. T opotecan 2. Irinotecan What is the me chanism of action of L-asparaginase ? Hydrolyzes asparagine to aspartic acid and ammonia, thereby depriving tumor cells of asparagine required for protein synthesis T his short chapter is meant as an overview of basic concepts of cancer chemotherapy. Clinical pharmacological therapies for specific cancer subtypes are discussed later in the text in relevant chapters.

CHAPTER 4 Autonomic Agents CHOLINERGIC AGENTS What are the major subdivisions of the autonomic ne rvous syste m? It is divided into the sympathetic and the parasympathetic nervous systems. What is the major ne urotransmitte r of the parasympathe tic autonomic ne rvous syste m? Acetylcholine (ACh). ACh is released into the synaptic clefts from the pre-and the postsynaptic neurons of the parasympathetic nervous system. In the sympathe tic ne rvous syste m, what ne urotransmitte r is re le ase d from the pre ganglionic ne uron into the synaptic cle ft? ACh. Remember that while the postganglionic neurotransmitters may differ between the sympathetic and parasympathetic branches of the autonomic nervous system, the preganglionic neurotransmitter released into the synaptic cleft is identical—ACh. Whe re are sympathe tic pre ganglionic fibe rs locate d? In the paravertebral chains on either side of the spinal column or the prevertebral ganglia on the ventral surface of the aorta. Sympathetic preganglionic fibers are short. Whe re are parasympathe tic pre ganglionic fibe rs locate d? In or near the wall of the organ they innervate. Parasympathetic preganglionic fibers are very long. Whe re are nicotinic re ce ptors locate d? Postsynaptic neurons in ganglia of both the parasympathetic nervous system (PNS) and sympathetic nervous system (SNS); adrenal medulla; neuromuscular junction (NMJ); central nervous system (CNS) Whe re are muscarinic re ce ptors locate d? Organs innervated by the PNS; thermoregulatory sweat glands innervated by the SNS; CNS (cortex, hippocampus) What doe s the PNS do to he art rate ? It decreases the heart rate. Remember that at rest the heart is constantly under parasympathetic tone to slow the heart rate from the intrinsic rate set by the sinoatrial (SA) node at about 80 beats per minute. What e nz yme catalyz e s the re action be twe e n choline and ace tyl-CoA to form ACh? Choline acetyltransferase (CAT ) The ne uronal re le ase of ACh into the synapse is inhibite d by what toxin? Botulinum toxin What organism produce s botulinum toxin? Clostridium botulinum (anaerobic, spore forming, gram-positive rod) The ve nom of which spide rs re sult in the re le ase of store d ACh into the synapse ? Any spider of the genus Latrodectus (widow spiders) of which the black widow is the most common species found in North America. T hey produce α-latrotoxin which causes the release of ACh from the preganglionic neuron into the synaptic cleft. What e nz yme de grade s ACh? Acetylcholinesterase (AChE) What are the bre akdown products of ACh? Choline and acetate Whe re is AChE locate d in the autonomic ne rvous syste m? In the synaptic cleft What is muscarine ? It is an alkaloid found in various poisonous mushrooms. Whe re are e ach of the following type s of muscarinic re ce ptors found in the body? M1 Nerves; gastric parietal cells M2 Nerves; cardiac cells; smooth muscle M3 Smooth muscle; exocrine glands; lungs; gastrointestinal (GI) tract; eye; bladder M4 CNS M5 CNS For e ach of the following muscarinic re ce ptor type s, name the type of G-prote in it is couple d to and the se cond me sse nge r syste m re sponsible for e xe cution of its activity upon stimulation: M1 Gq coupled; inositol triphosphate (IP 3), diacylglycerol (DAG) cascade M2 Gi coupled; inhibition of cyclic AMP (cAMP) production, activation of potassium channels M3 Gq coupled; IP 3, DAG cascade M4 Gi coupled; inhibition of cAMP production M5 Gq coupled; IP 3, DAG cascade Doe s the PNS dire ctly inne rvate the vasculature ? No. Vascular tone is primarily determined by the degree of stimulation of adrenergic receptors of the sympathetic nervous system which directly innervate the vascular smooth muscle cells. However, there are muscarinic receptors located on the vasculature. How can ACh lowe r blood pre ssure ? ACh binds to ACh receptors in the vasculature leading to increased synthesis of nitric oxide (NO) via second messenger pathways. An increase in NO leads to vasodilation. NO is also known as what? Endothelial-derived relaxation factor (EDRF) What amino acid is a pre cursor to NO synthe sis? Arginine Doe s AChE have a high affinity for ACh?

Yes Doe s ACh incre ase or de cre ase the following (in othe r words, what is the e ffe ct of parasympathe tic stimulation of the following)? Blood pre ssure Decreases (both arterial and venous dilation via NO) He art rate Decreases (via M2 receptors) Salivation Increases (via M3 receptors) Lacrimation Increases (via M3 receptors) Swe ating Increases (via sympathetic stimulation of muscarinic cholinergic receptors) GI se cre tions Increases (via M3 receptors) GI motility Increases (via M3 receptors) Miosis (constriction of pupil) Increases (via M3 receptors) Bladde r de trusor muscle tone Increases (via M3 receptors) Bladde r sphincte r tone Decreases (in combination with increased detrusor tone this leads to increased urination also via M3receptors) Bronchodilation Decreases (via M3 receptors) What doe s ACh do to the ciliary muscle of the e ye ? Increased contraction which leads to increased accommodation How doe s ACh cause miosis? Increased contraction of the circular muscle in the iris Doe s be thane chol have muscarinic activity? Yes (agonist) Doe s be thane chol have nicotinic activity? No Doe s AChE have a high affinity for be thane chol? No (zero affinity). T his gives bethanechol a long duration of action. What is a clinical use for be thane chol? Nonobstructive urinary retention as can result from denervation of the urinary sphincter in conditions such as diabetes or spinal cord injury. Bethanechol can also be used for gastroesophageal reflux disease (GERD). As a cholinergic drug, it will increase detrusor tone and GI motility. Doe s carbachol have muscarinic activity? Yes, it is a muscarinic agonist. Doe s carbachol have nicotinic activity? Yes, it is also a nicotinic agonist. Doe s AChE have a high affinity for carbachol? No, the enzyme has zero affinity for carbachol. What is carbachol use d for? It is a miotic agent to reduce intraocular pressure (IOP) in emergency settings of narrow-angle and open-angle glaucoma. Doe s pilocarpine have muscarinic activity? Yes, it is a muscarinic agonist. Doe s pilocarpine have nicotinic activity? No Doe s AChE have a high affinity for pilocarpine ? No, the enzyme has zero affinity for pilocarpine. What is pilocarpine use d for? It is the miotic drug of choice to lower IOP in emergency settings of narrow-angle and open-angle glaucoma. Can pilocarpine cross the blood-brain barrie r (BBB)? Yes. Because it is a tertiary, uncharged amine. Give e xample s of re ve rsible AChE inhibitors: Neostigmine; pyridostigmine; physostigmine; edrophonium; rivastigmine; donepezil; galantamine; tacrine What are done pe z il, galantamine , rivastigmine , and tacrine use d for? Alzheimer-type dementia. T hey are AChE inhibitors, thereby increasing the levels of ACh in the brain. What two AChE inhibitors are quate rnary ammonium compounds and the re fore cannot cross the BBB? 1. Neostigmine 2. Pyridostigmine As a result, these drugs will not reverse the central nervous system effects of cholinergic toxicity. What short-acting AChE inhibitor is use d to diagnose myasthe nia gravis and is also use d to diffe re ntiate myasthe nic from choline rgic crisis? Edrophonium. T he trade name of edrophonium is T ensilon. T his test is commonly referred to as the T ensilon test. Which re ve rsible AChE inhibitor is use d as an antidote in atropine ove rdose ? Physostigmine, a tertiary amine, is able to cross the BBB to act on the CNS. Give e xample s of irre ve rsible AChE inhibitors: Echothiophate; isoflurophate; sarin; malathion; parathion Name an irre ve rsible AChE inhibitor that is use d as ne rve gas: Sarin Which two AChE inhibitors are use d as inse cticide s? 1. Malathion 2. Parathion

What is anothe r name for the irre ve rsible AChE inhibitors? Organophosphates How do organophosphate s irre ve rsibly inhibit AChE? T he phosphate group covalently binds to serine hydroxyl group in the active site of AChE, thereby rendering the enzyme permanently inactive. What is use d to counte ract the muscarinic and CNS e ffe cts of organophosphate poisoning? Atropine via competitive inhibition. Atropine binds the muscarinic receptors, outcompeting the increased levels of ACh thereby preventing overstimulation. What age nt is use d to re activate inhibite d AChE during organophosphate poisoning? Pralidoxime (2-PAM). It is critical to initiate treatment with pralidoxime early along with atropine to prevent the process of aging where AChE is irreversibly inactivated by the organophosphates. What are the signs and symptoms of organophosphate poisoning? SLUDGE: salivation; lacrimation; urination; diaphoresis; GI motility (diarrhea); emesis. Basically, parasympathetic overstimulation. Doe s atropine block nicotinic re ce ptors, muscarinic re ce ptors, or both? It blocks muscarinic receptors. What are the pharmacologic actions of atropine ? Mydriasis; cycloplegia; tachycardia; sedation; urinary retention; constipation; dry mouth; dry eyes; decreased sweating; hallucinations; sedation; hyperthermia; delirium; blurred vision; coma (high doses). Basically, anticholinergic/sympathetic overstimulation. What class of drugs can be use d to counte ract atropine ove rdose ? AChE inhibitors Name thre e drug classe s that may cause antimuscarinic adve rse e ffe cts: 1. Sedating/first-generation antihistamines (diphenhydramine) 2. T ricyclic antidepressants (T CAs) 3. Phenothiazines Low-dose (0.5-1 mg) atropine doe s what to he art rate ? Increases heart rate (parasympatholytic effect) What is be lladonna? A perennial plant also known as “ deadly nightshade” due to the toxic effects of its foliage and berries from which atropine is derived. Other toxins include scopolamine and hyoscyamine. T he name belladonna derives from the cosmetic enhancing effects of dilated pupils, blushing of the cheeks, and reddening of the lips for which the plant was originally used. How doe s scopolamine diffe r from atropine ? Scopolamine has a longer duration of action, more potent CNS effects, and is able to block short-term memory. What is the main the rape utic indication of scopolamine ? Motion sickness Giving drugs with anticholine rgic activity can pre cipitate an e me rge nt situation in patie nts with what me dical condition? Patients with narrow-angle glaucoma What are the signs and symptoms of acute -angle -closure glaucoma? General distress; pain; headache; red eye; photophobia; increased IOP; visual changes; malaise; nausea; vomiting What two anticholine rgic age nts are quate rnary ammonium compounds and use d for the tre atme nt of asthma and chronic obstructive pulmonary dise ase (CO PD)? 1. Ipratropium 2. T iotropium T iotropium has a longer half-life compared to ipratropium. Doe s ipratropium e ffe ct airway se cre tions? No (unlike atropine, which decreases airway secretions) Name thre e ganglionic blocking age nts: 1. Hexamethonium 2. Mecamylamine 3. T rimethaphan What are ganglionic blocking age nts primarily use d for? Lowering blood pressure; blocking autonomic nervous system reflexes; smoking cessation due to blockade of central nicotine receptors Why can ganglionic blocke rs cause a marke d postural hypote nsion? Since sympathetic tone to the blood vessels is blocked, both arterial and venous dilation occur, lowering blood pressure. Moreover, the ganglionic blockers prevent the sympathetically mediated baroreceptor response to a sudden decrease in blood pressure, such as that occurs with a rapid change in position from sitting to standing. Ne uromuscular blocking age nts (NMBs) can be groupe d into what two ge ne ral cate gorie s? 1. Depolarizing 2. Nondepolarizing Do NMBs work at muscarinic or nicotinic re ce ptors? Nicotinic receptors (remember the NMJ has nicotinic receptors). How many subunits is the nicotinic re ce ptor made of? Five subunits. T wo α:- and three β-subunits make up this transmembrane ligand-gated ion channel. Which subunit of the nicotinic re ce ptor doe s ACh bind to? Between the two α-subunits Binding of ACh to the nicotinic re ce ptor at the NMJ is re quire d to ope n which type of ion channe l? Sodium channel What is the most commonly use d NMB? Succinylcholine, the only depolarizing NMB. T his is an ideal drug for endotracheal intubation due to its fast onset of action and short duration of action.

How doe s succinylcholine work at the NMJ? It behaves as a cholinergic agonist that remains bound to the ACh receptor for a prolonged period. What happe ns during e ach of the following phase s of succinylcholine activity at the NMJ? Phase I T he receptor becomes depolarized and transient fasciculations are observed as various motor units depolarize. Phase II T he receptor becomes resistant to depolarization and a flaccid paralysis ensues. What are the two main use s of succinylcholine ? 1. It is used for facilitation of endotracheal intubation via relaxation of pharyngeal and laryngeal muscles. 2. It is used as an adjunct during electroconvulsive shock therapy to prevent prolonged full body convulsions which would result in muscle breakdown. A tourniquet is placed on a lower extremity to prevent the drug from reaching this location so that the seizure is visible in a localized area and the rest of the body is spared. Is succinylcholine short or long acting? It is short acting with a duration of 4-8 minutes because of rapid hydrolysis by plasma cholinesterase. What are the adve rse e ffe cts of succinylcholine ? Malignant hyperthermia; apnea; hypertension; hyperkalemia What are the signs and symptoms of malignant hype rthe rmia? Muscular rigidity; increased oxygen consumption; increased carbon dioxide production (usually the first sign detected during surgery); tachycardia; hyperthermia is a late finding How is malignant hype rthe rmia tre ate d? With dantrolene What is the me chanism of action of dantrole ne ? It inhibits calcium release from the sarcoplasmic reticulum of muscle cells, thereby relaxing muscle tone and reducing heat production. Succinylcholine may have a prolonge d half-life in what type of patie nts? Patients with a genetic deficiency or altered form of plasma cholinesterase What is the me chanism of action of nonde polariz ing NMBs? Competitive antagonists of ACh at the NMJ Which drug is the prototype of the nonde polariz ing NMBs? T ubocurarine What antidote is use d in tubocurarine ove rdose ? AChE inhibitor (increases ACh concentration which competes with tubocurarine at ACh receptors at the NMJ) List in orde r, from first to last, the muscle s that are paralyz e d by nonde polariz ing NMBs: 1. 2. 3. 4. 5.

Small muscles of the face and eye Fingers Limbs, neck, trunk Intercostals Diaphragm

Which antimicrobial class of drugs may act in syne rgy with nonde polariz ing NMBs by inhibiting re le ase of ACh from ne rve e ndings by compe ting with calcium ions, the re by incre asing ne uromuscular blockade ? Aminoglycosides (most likely to occur with high doses; patients with hypocalcemia, hypomagnesemia, or neuromuscular disorders) Give e xample s of nonde polariz ing NMBs: T ubocurarine; atracurium; mivacurium; rocuronium; vecuronium; pancuronium; pipercuronium What is the only nonde polariz ing NMB that doe s not re quire dosage re duction in patie nts with re nal failure ? Atracurium, which excreted in bile, not in urine What nonde polariz ing NMB has the most rapid onse t of action? Rocuronium. T hink: ROcuronium—Rapid Onset In what situations are nonde polariz ing NMBs use d? Adjunct to general anesthesia to facilitate endotracheal intubation and to relax skeletal muscles during surgery; to facilitate mechanical ventilation in ICU patients

ADRENERGIC AGENTS What are the major ne urotransmitte rs of the SNS? Epinephrine; norepinephrine; dopamine What amino acid is the pre cursor to dopamine , e pine phrine , and nore pine phrine ? T yrosine What are the ste ps, in orde r, to the synthe sis of e pine phrine starting from tyrosine ? Tyrosine is converted into DOPA by tyrosine hydroxylase (rate-limiting step); DOPA is converted into dopamine by DOPA decarboxylase; dopamine is converted into norepinephrine by dopamine β-hydroxylase; norepinephrine is converted into epinephrine by methylation in the adrenal medulla. What two e nz yme s me taboliz e nore pine phrine ? 1. Monoamine oxidase (MAO) 2. Catechol-O-methyltransferase (COMT ) What is the me chanism of action of re se rpine ? It inhibits the transport of norepinephrine from the neuronal cytoplasm into the synaptic vesicles. What are the common side e ffe cts of re se rpine ? Depression; sedation What bre akdown products of nore pine phrine are e xcre te d in the urine and can be me asure d to he lp diagnose phe ochromocytoma? Vanillylmandelic acid (VMA); metanephrine; normetanephrine What are the two major classe s of adre ne rgic re ce ptors? 1. α-Receptors

2. β-Receptors What ne urotransmitte rs are me taboliz e d by MAO type A? Norepinephrine; epinephrine; serotonin; tyramine; dopamine What ne urotransmitte rs doe s MAO type B me taboliz e ? Dopamine. Dopamine is metabolized by both the A and B type of the enzyme. How doe s cocaine incre ase nore pine phrine le ve ls in the synaptic cle ft? It inhibits the reuptake of neurotransmitter back into the presynaptic neuron. How do amphe tamine , e phe drine , and tyramine incre ase nore pine phrine le ve ls? T hey act as indirect sympathomimetic agents by entering the presynaptic neuron releasing stored norepinephrine into the synaptic cleft. Whe re are α1-re ce ptors found? Vascular smooth muscle; papillary dilator muscle; pilomotor smooth muscle; prostate; heart Whe re are α2-re ce ptors found? Postsynaptic CNS adrenoceptors; pancreatic β-cells; platelets; adrenergic and cholinergic nerve terminals; vascular smooth muscle; fat cells Whe re are β1-re ce ptors found? Heart; juxtaglomerular cells Whe re are β2-re ce ptors found? Respiratory, uterine, and vascular smooth muscle; skeletal muscle; liver Whe re are D1-re ce ptors found? Smooth muscle Whe re are D2-re ce ptors found? Nerve endings Compare and contrast the local ve rsus the syste mic e ffe cts of α2-re ce ptor activation. Local infusion of an α2-agonist will activate the α2-receptors in the vasculature, causing vasoconstriction. Systemic administration will activate the central α2receptors in the locus ceruleus which inhibits norepinephrine release and sympathetic activation. T he central effects overwhelm the local effects leading to decreased blood pressure. Give e xample s of α2-re ce ptor agonists: Clonidine; α-methyldopa; guanabenz; guanfacine; dexmedetomidine What are the the rape utic indications of clonidine ? Hypertension; severe pain; heroin withdrawal; nicotine withdrawal; ethanol dependence; clozapine-induced sialorrhea; prevention of migraines What is de xme de tomidine use d for? Sedation of intubated and mechanically ventilated patients; prolongation of spinal anesthesia With drugs that activate both α- and β-re ce ptors, which re ce ptors are ge ne rally activate d first (which re ce ptors are more se nsitive )? β-Receptors Activation of what re ce ptor type in the e ye will le ad to contraction of the radial muscle and subse que ntly le ad to mydriasis? α1-Receptor For e ach of the following re ce ptor type s, name the type of G-prote in it is couple d to: α1 Gq α2 Gi β1,β2,D1 Gs Name the major e ffe cts me diate d by e ach of the following re ce ptor type s: α1 Mydriasis; vasoconstriction → increased blood pressure; decreased urination; increased glycogenolysis; decreased renin release; ejaculation α2 Inhibition of norepinephrine release (central effect); inhibition of insulin release; platelet aggregation β1 Increased heart rate; increased conduction velocity; increased force of heart contraction; increased renin release β2 Vasodilation; bronchodilation; relaxation of uterine, respiratory, and vascular smooth muscle; increased insulin secretion; increased potassium uptake; increased glycogenolysis Pe riphe ral Vasodilation of coronary, renal, and mesenteric vasculature; increased glomerular filtration rate (GFR); increased renal blood flow (RBF); increased sodium excretion Will α2-re ce ptor activation in the pancre as cause insulin se cre tion to incre ase or de cre ase ? It will cause insulin secretion to decrease. Will β2-re ce ptor activation in the pancre as cause insulin se cre tion to incre ase or de cre ase ? It will cause insulin secretion to increase. Which re ce ptor type doe s e pine phrine pre fe re ntially bind to at low dose s? β-Receptors (vasodilation in vasculature) Which re ce ptor type doe s e pine phrine pre fe re ntially bind to at high dose s? α-Receptors (vasoconstriction in vasculature) What is the drug of choice in patie nts with type 1 (imme diate ) hype rse nsitivity re actions? Epinephrine What is the dose of e pine phrine give n for anaphylaxis? 0.1-0.5 mg. Note: T he EpiPen (epinephrine auto-injector) that many patients with a history of anaphylaxis carry is 0.3 mg. What is the conce ntration of e pine phrine use d for anaphylaxis? 1:1000 What is the conce ntration of e pine phrine use d for advance d cardiac life support (ACLS) protocol? 1:10,000 Why is e pine phrine ofte n give n in combination with local ane sthe tics? Epinephrine causes a vasoconstriction, thereby inhibiting the local anesthetics redistribution away from its site of action, so it increases the duration of local anesthesia. What is the conce ntration of e pine phrine whe n give n in combination with local ane sthe tics? 1:100,000

State whe the r e ach of the following cardiovascular e ffe cts incre ase s or de cre ase s with low-dose e pine phrine : Pe riphe ral vascular re sistance Decreases Systolic blood pre ssure Increases Diastolic blood pre ssure Decreases Pulse pre ssure Increases What re ce ptors are activate d by isoprote re nol? β1 = β2 What re ce ptors are activate d by dopamine ? D>β>α What re ce ptors are activate d by dobutamine ? β1 > β2 What re ce ptors are activate d by phe nyle phrine ? α1 > α2 Doe s nore pine phrine activate β2-re ce ptors? No Activation of dopamine re ce ptors will cause what type of re sponse in the me se nte ric and re nal vasculature ? Vasodilation What is dopamine me taboliz e d to? Homovanillic acid (HVA) What is dobutamine use d for? Increases cardiac output in congestive heart failure (CHF) without affecting RBF (unlike dopamine) Tyramine is a bre akdown product of which amino acid? T yrosine Whe re is tyramine found? Examples of foods and beverages which contain tyramine include: beer, ale, robust red wines, chianti, vermouth, homemade breads, cheese, sour cream, bananas, red plums, figs, raisins, avocados, fava beans, Italian broad beans, green bean pods, eggplant, pickled herring, liver, dry sausages, canned meats, salami, yogurt, soup cubes, commercial gravies, chocolate, and soy sauce. What e nz yme is re sponsible for the bre akdown of tyramine ? MAO type A What can pote ntially happe n if a patie nt on a monoamine oxidase inhibitor (MAO I) consume s large amount of fe rme nte d che e se ? Hypertensive crisis due to tyramine in the cheese which leads to the release of norepinephrine from storage vesicles in presynaptic neurons What is phe nyle phrine and pse udoe phe drine use d to tre at? Nasal congestion What are mixe d action adre ne rgic agonists? Substances that release stored norepinephrine from nerve terminals and also directly stimulate α- and β-receptors What are some e xample s of mixe d action adre ne rgic agonists? Ephedrine; pseudoephedrine; metaraminol Which drug is a nonse le ctive , compe titive antagonist at both α1- and α2-re ce ptors? Phentolamine Which drug is a nonse le ctive , irre ve rsible antagonist at both α1- and α2-re ce ptors? Phenoxybenzamine What are phe ntolamine and phe noxybe nz amine mainly use d for? To achieve α-receptor blockade before surgical removal of pheochromocytoma, to achieve perioperative blood pressure control, and to prevent intraoperative hypertension from release of catecholamines during surgical manipulation What is the me chanism of action of praz osin? Selective α1-antagonist What are praz osin, te raz osin, and doxaz osin use d to tre at? Benign prostatic hyperplasia (BPH); hypertension Doe s the “H” in BPH stand for hype rtrophy or hype rplasia? Hyperplasia, though hypertrophy is still sometimes erroneously used. T here is an actual increase in the number of prostatic cells in BPH. T he cells do not simply increase in volume as do, for example, skeletal or cardiac muscle cells in response to increased used. This drug is a se le ctive α1A-re ce ptor antagonist, use d in the tre atme nt of BPH, and has le ss cardiovascular side e ffe cts ve rsus traditional α1antagonists. T amsulosin What advantage s do se le ctive α1-antagonists have ove r nonse le ctive α-antagonists? Less reflex tachycardia What CNS pre junctional α2-re ce ptor antagonist is use d to tre at postural hypote nsion and e re ctile dysfunction (impote nce )? Yohimbine What CNS pre junctional α2-re ce ptor antagonist is use d to tre at de pre ssion? Mirtazapine Give e xample s of β1-se le ctive antagonists: Acebutolol; atenolol; bisoprolol; betaxolol; esmolol; metoprolol Give e xample s of nonse le ctive β-antagonists: Propranolol; timolol; pindolol; nadolol Give e xample s of mixe d-a (Xj/P ntagonists: Carvedilol; labetalol What is the name of a β-antagonist that also blocks potassium channe ls and is use d as an antiarrhythmic? Sotalol What is intrinsic sympathomime tic activity (ISA)? Drugs act as partial agonists and only work when there is increased sympathetic drive such as with exercise; less bradycardia; less effects on lipid metabolism Which two β-antagonists have ISA? 1. Acebutolol 2. Pindolol

What happe ns to e xe rcise tole rance in patie nts be ing tre ate d with β-blocke rs? Decreased exercise tolerance What are the main the rape utic indications of β-blocke rs? Angina; arrhythmias; hypertension; CHF (not all β-blockers); thyrotoxicosis; glaucoma (ophthalmic formulations) What are some noncardiovascular use s of propranolol? Migraine prophylaxis; performance anxiety “ stage fright” β-Blocke rs can inhibit the majority of e ffe cts cause d by thyrotoxicosis e xce pt for what sign? Diaphoresis. Remember that sweat glands have muscarinic receptors and are cholinergic rather than adrenergic. β-Blocke rs can inhibit the majority of e ffe cts cause d by hypoglyce mia e xce pt for what sign? Diaphoresis. β-Blockers are contraindicated in diabetic patients treated with oral hypoglycemics for this reason. What doe s propranolol do to se rum triglyce ride s? Increases serum triglycerides What doe s propranolol do to se rum low-de nsity lipoprote in (LDL)? Increases serum LDL Why doe s propranolol cause vivid dre ams? Crosses the BBB Why should β-blocke rs be tape re d down inste ad of abruptly discontinue d? Chronic therapy leads to upregulation of β-receptors; therefore, abrupt discontinuation may lead to life-threatening cardiovascular rebound effects (tachycardia; hypertension; arrhythmias; death)

CLINICAL VIGNETTES A mothe r brings he r 4-month-old infant to the pe diatrician. The child de ve lope d what appe are d to be a viral fe brile illne ss which rapidly progre sse d. The infant now appe ars listle ss and has de cre ase d muscle tone . The mothe r re ports de cre ase d fe e ding and bowe l move me nts and says that the child’s cry sounds diffe re nt from normal. The only tre atme nt she has initiate d is warm milk with hone y to he lp the child’s immune syste m. What is the most like ly diagnosis? While the child may have initially had a viral upper respiratory tract infection (URI), ingestion of honey exposed the infant to C. botulinum spores which are able to colonize the immature pediatric small intestines. Botulinum toxin is then released into the systemic circulation which blocks the release of ACh from the synaptic cleft, leading to the flaccid paralysis known as “ floppy baby syndrome.” A patie nt has be e n found to inconsiste ntly have e le vate d blood pre ssure during various office visits ove r the past se ve ral ye ars. A de cision is made to try propranolol to control the patie nt’s blood pre ssure . O n re turn visit 2 we e ks late r, the patie nt’s blood pre ssure is marke dly e le vate d and the patie nt re ports he adache s and blurre d vision. What is the most like ly diagnosis? Pheochromocytoma. T he patient most likely had sporadic release of catecholamines from the tumor leading to periods of increased blood pressure. T he most common neurotransmitter released from pheochromocytomas is norepinephrine. Nonselective blockade of the β-adrenergic receptors by propranolol leads to unopposed α-adrenergic receptor activation by the norepinephrine released by the pheochromocytoma leading to a hypertensive emergency. A 62-ye ar-old man come s to his primary care physician for a re gular che ckup. His blood pre ssure has be e n mildly e le vate d on the last two office visits. Today’s re ading is 140/92. He is also complaining of some difficulty initiating urination as we ll as ge tting up two or more time s pe r night to urinate . What is the be st pharmacothe rapy to initiate at this time ? Prazosin or another selective α1-antagonist such as terazosin or doxazosin. Prazosin blocks α1-receptors found in the base of the bladder and prostate, preventing constriction and decreasing the resistance to urine flow. It also relaxes vascular smooth muscle leading to decreased blood pressure. Always try to minimize the amount of medications you prescribe for patients. Here, the dual action of prazosin makes it an ideal choice for this patient with two separate health concerns.

CHAPTER 5 CNS Agents GENERAL AND LOCAL ANES THETICS State which stage of ane sthe sia e ach of the following de scriptions re fe rs to? De lirium; viole nt be havior; incre ase d blood pre ssure ; incre ase d re spiratory rate ; irre gular bre athing rate and volume ; amne sia; re tching and vomiting with stimulation; disconjugate gaz e Stage II (excitement) De pre ssion of vasomotor ce nte r; de pre ssion of re spiratory ce nte r; de ath may occur Stage IV (medullary depression) Eye move me nts ce ase ; fixe d pupils; re gular re spiration; re laxation of ske le tal muscle s Stage III (surgical anesthesia) Loss of pain se nsation; patie nt is conscious; no amne sia in e arly part of this stage Stage I (analgesia) Give e xample s of inhale d ane sthe tics: Halothane; nitrous oxide; isoflurane; enflurane; sevoflurane; desflurane; methoxyflurane With re gard to inhale d ane sthe tics, what doe s MAC stand for? Minimum alveolar concentration. Note: this is not to be confused with monitored anesthesia care also commonly referred to as MAC, which is a combination of regional anesthesia, sedation, and analgesia. What is MAC in re gard to inhale d ane sthe tics? T he concentration of inhaled anesthetic required to stop movement in 50% of patients given a standardized skin incision; a measure of potency for inhaled anesthetics For pote nt inhale d ane sthe tics, is the MAC small or large ? Small (inverse of the MAC is used as an index of potency for inhaled anesthetics) Which inhale d ane sthe tic has the large st MAC? Nitrous oxide (>100%) Which inhale d ane sthe tic has the smalle st MAC? Halothane (0.75%) As lipid solubility of an inhale d ane sthe tic incre ase s, what happe ns to the conce ntration of inhale d ane sthe tic ne e de d to produce ane sthe sia, that is, doe s it incre ase or de cre ase ? It decreases. What is the blood/gas partition coe fficie nt? T he ratio of the total amount of gas in the blood relative to the gas equilibrium phase. It refers to an inhaled anesthetic’s solubility in the blood. If an inhale d ane sthe tic has a high blood/gas partition coe fficie nt, will time s of induction and re cove ry be incre ase d or de cre ase d? It will be increased because the time to increase arterial tension is longer. Give an e xample of an inhale d ane sthe tic with a low blood/gas partition coe fficie nt (low blood solubility): Nitrous oxide (0.5); desflurane (0.4) Give an e xample of an inhale d ane sthe tic with a high blood/gas partition coe fficie nt (high blood solubility): Halothane (2.3); enflurane (1.8) Which inhale d ane sthe tic, halothane or nitrous oxide , will take longe r to change the de pth of ane sthe sia whe n the conce ntration of the inhale d ane sthe tic has be e n change d? Halothane Are MAC value s additive ? Yes Are MAC value s highe r or lowe r in e lde rly patie nts? T hey are lower, thus elderly patients generally require lower concentrations of inhaled anesthetics. Are MAC value s highe r or lowe r whe n opioid analge sics and/or se dative hypnotics are use d concomitantly? T hey are lower. Do inhale d ane sthe tics incre ase or de cre ase the re sponse to Pco 2 le ve ls? Decrease Do inhale d ane sthe tics incre ase or de cre ase ce re bral vascular flow? Increase Do inhale d ane sthe tics incre ase or de cre ase intracranial pre ssure ? Increase Do inhale d ane sthe tics re lax or stre ngthe n ute rine smooth muscle contractions? Relax (except methoxyflurane when briefly inhaled, therefore, can be used during childbirth) Which of the inhale d ane sthe tics is not a haloge nate d hydrocarbon? Nitrous oxide Are the inhale d haloge nate d hydrocarbon ane sthe tics volatile or nonvolatile gase s? Volatile gases Which inhale d ane sthe tic is associate d with malignant hype rthe rmia? Halothane What characte riz e s malignant hype rthe rmia? Hyperthermia; muscle rigidity; acidosis; hypertension; hyperkalemia Should a patie nt with a family history positive for malignant hype rthe rmia be conce rne d? Yes, because a genetic defect in ryanodine receptors may be inherited. What drug is give n to tre at malignant hype rthe rmia? Dantrolene Which inhale d ane sthe tic is associate d with incre ase d bronchiolar se cre tions? Isoflurane Which inhale d ane sthe tic is associate d with he patitis? Halothane Halothane is not he patotoxic in what patie nt population? Pediatric patients Which inhale d ane sthe tic is the le ast he patotoxic? Nitrous oxide Which inhale d ane sthe tic is associate d with incre ase d bronchiolar spasms? Isoflurane

Which inhale d ane sthe tic re laxe s bronchial smooth muscle ? Halothane Which inhale d ane sthe tic is associate d with cardiac arrhythmias? Halothane Which inhale d ane sthe tics incre ase he art rate (via re fle x se condary to vasodilation)? Isoflurane; desflurane Which inhale d ane sthe tics de cre ase he art rate ? Halothane; enflurane; sevoflurane Which inhale d ane sthe tic de cre ase s re nal and he patic blood flow? Halothane Give e xample s of intrave nous (IV) ane sthe tics: Propofol; fentanyl; ketamine; midazolam; thiopental; etomidate Which of the pre viously me ntione d IV ane sthe tics is a barbiturate ? T hiopental Which of the pre viously me ntione d IV ane sthe tics is a be nz odiaz e pine ? Midazolam Which of the pre viously me ntione d IV ane sthe tics is an opioid? Fentanyl Is thiope ntal use d for induction, mainte nance , or both? Induction Pharmacodynamically, how doe s re cove ry occur with the rapid-acting barbiturate s? Rapid redistribution from the central nervous system (CNS) to peripheral tissues State whe the r thiope ntal incre ase s, de cre ase s, or doe s not change e ach of the following physiologic e ffe cts: Ce re bral blood flow No change Re spiratory function Decreases Blood pre ssure Decreases Why should caution be take n whe n administe ring thiope ntal to asthmatic patie nts? May cause laryngospasm Midaz olam offe rs which type of amne sia making it use ful for monitore d ane sthe sia care ? Anterograde amnesia What is the antidote for midaz olam-induce d re spiratory de pre ssion? Flumazenil, which is also the antidote for any benzodiazepine overdose What adve rse drug re action may be cause d by fe ntanyl whe n give n intrave nously? Chest wall rigidity Doe s propofol have good analge sic prope rtie s? No About which alle rgie s should a patie nt be que stione d be fore administration of propofol? Egg and soybeans. Propofol is prepared as a lipid emulsion using egg and soybean lecithin. T his gives propofol its white color and can cause allergic reactions in patients with sensitivities to these substances. Doe s propofol incre ase or de cre ase blood pre ssure ? It decreases blood pressure. Is propofol use d for induction, mainte nance , or both? It is used for both. Which IV ane sthe tic cause s dissociative ane sthe sia? Ketamine What is dissociative ane sthe sia? T he patient is unconscious and feels no pain, yet appears awake. Eyes may open and the swallowing reflex is present, but the patient is sedated, immobile, and usually amnestic. Hallucinations and delirium are common. Which ane sthe tic has antie me tic prope rtie s? Propofol Which IV ane sthe tic is a cardiovascular stimulant (incre ase s blood pre ssure and cardiac output)? Ketamine Which IV ane sthe tic cause s vivid dre ams and hallucinations? Ketamine Doe s ke tamine incre ase or de cre ase ce re bral blood flow? Increase What is the most cardiac-stable IV ane sthe tic age nt? Etomidate Are local ane sthe tics we ak acids or we ak base s? Weak bases Give e xample s of amide local ane sthe tics: Lidocaine; prilocaine; articaine; mepivacaine; bupivacaine (all have >1 “i” in their generic name) Give e xample s of e ste r local ane sthe tics: Cocaine; benzocaine; procaine (all have only one “ i” in their generic name) Which me dication, whe n use d in combination, re duce s syste mic toxicity and incre ase s the duration of action of local ane sthe tics? Epinephrine, by inducing a local vasoconstriction Epine phrine should not be combine d with local ane sthe tics whe n inje cting ne ar which anatomic site s? Digits; nose; ears; penis; and any end-artery circulation Which type of e nz yme s me taboliz e amide local ane sthe tics and whe re are the y locate d? Amidases located in the liver Which type of e nz yme s me taboliz e e ste r local ane sthe tics and whe re are the y locate d? Esterases located in tissues and blood What is the me chanism of action of local ane sthe tics? Inhibition of sodium channels in axonal membranes via binding to the channels in their inactivated state and preventing a structural change to the resting state Do local ane sthe tics ne e d to be in the ioniz e d or nonioniz e d form to bind to the sodium channe l? Ionized form Do local ane sthe tics ne e d to be in the ioniz e d or nonioniz e d form to gain acce ss to the sodium channe l, which is locate d on the inne r side of the axonal me mbrane ? Nonionized form (must be able to cross lipophilic axonal membrane)

All local ane sthe tics cause vasodilation with the e xce ption of which drug? Cocaine (causes vasoconstriction) Ne rve fibe rs most se nsitive to blockade are of smalle r or large r diame te r? Smaller diameter Ne rve fibe rs most se nsitive to blockade have low or high firing rate s? High firing rates Which ne rve fibe rs are most se nsitive to local ane sthe tics? T ype B fibers; type C fibers Which ne rve fibe rs are le ast se nsitive to local ane sthe tics? T ype A α-fibers What are the adve rse e ffe cts of local ane sthe tics? Hypotension (except cocaine); nystagmus; seizures; dizziness; allergic reactions (rare) Alle rgic re actions are more associate d with e ste r or amide local ane sthe tics? Esters (via para-aminobenzoic acid [PABA] formation)

OPIOID ANALGES ICS AND ANTAGONIS TS Which ne urotransmitte r binds to the δ-opioid re ce ptor? Enkephalin Which ne urotransmitte r binds to the δ-opioid re ce ptor? Dynorphin Which ne urotransmitte r binds to the β-opioid re ce ptor? β-Endorphin What is the me chanism of action of me dications that activate presynaptic opioid re ce ptors? Inhibits calcium influx through voltage-gated ion channels, thereby inhibiting neurotransmitter release What is the me chanism of action of me dications that activate postsynaptic opioid re ce ptors? Increases potassium efflux from cells leading to membrane hyperpolarization and thereby inhibition of neurotransmitter release O pioid re ce ptors are couple d to what type of prote ins? Inhibitory G-proteins (inhibits adenylyl cyclase) What is the prototype opioid analge sic? Morphine Why must caution be take n whe n using opioids in patie nts with he ad injurie s? Opioids may increase intracranial pressure Whe re in the midbrain are opioid re ce ptors locate d? Periaqueductal gray region (binding to these receptors leads to activation of descending pathways to the raphe nuclei, thereby decreasing transmission throughout pain pathways) Whe re in the dorsal horn of the spinal cord are opioid re ce ptors locate d? Primary afferent fibers (binding to these receptors leads to inhibition of substance P release) Are opioid analge sics be tte r at re lie ving inte rmitte nt or pe rsiste nt pain? Persistent pain What is the me chanism of morphine -induce d hypote nsion and pruritus? Increased histamine release from mast cells Do opioid analge sics incre ase or de cre ase gastrointe stinal (GI) pe ristalsis? Decrease (they cause constipation) Which two opioids are use d spe cifically to tre at diarrhe a? 1. Loperamide 2. Diphenoxylate Which opioid analge sic doe s not incre ase the tone of the biliary tract, bladde r, and ure te r? Meperidine (antagonizes muscarinic receptors) Do opioid analge sics incre ase or de cre ase ute rine contractions during pre gnancy? T hey decrease uterine contractions, thus a good contraction pattern should be achieved before placement of an epidural catheter during labor. Do opioid analge sics cause miosis or mydriasis of the pupils? Miosis (common sign of opioid overdose is pinpoint pupils) What is the me chanism of opioid-induce d miosis? Increased parasympathetic (cholinergic) activity in the pupilary constrictor muscles Which opioid analge sic doe s not cause miosis? Meperidine (antagonizes muscarinic receptors) Which two opioids are use d spe cifically to tre at cough? 1. Codeine 2. Dextromethorphan Opioids suppress the cough reflex. Synthetic Is de xtrome thorphan a natural or synthe tic opioid? Synthetic What is the me chanism of opioid-induce d urinary re te ntion? Increases antidiuretic hormone (ADH) Do opioid analge sics promote e me sis or act as antie me tics? Promote emesis What is the me chanism of opioid-induce d e me sis? Activation of the chemoreceptor trigger zone (CT Z) Whe re is the CTZ locate d? Area postrema What is the me chanism of opioid-induce d re spiratory de pre ssion? Reduced sensitivity of respiratory center to carbon dioxide levels

What is the most common cause of de ath in opioid ove rdose ? Respiratory depression What are the two most lipophilic opioids? 1. Heroin 2. Fentanyl T hese two medications rapidly cross the blood-brain barrier (BBB) to produce euphoric effects. Which opioid is the le ast lipophilic? Morphine Is morphine me taboliz e d via phase I or phase II re actions? Phase II metabolism (glucuronidation) Doe s morphine -3-glucuronide have analge sic activity? No Doe s morphine -6-glucuronide have analge sic activity? Yes Which two opioid-induce d e ffe cts do patie nts not de ve lop tole rance to? 1. Constipation 2. Miosis What are the signs and symptoms of opioid withdrawal? Lacrimation; rhinorrhea; diaphoresis; yawning; goose bumps; anxiety; muscle spasms; diarrhea; increased pain sensation Which me dication is use d to counte ract the re spiratory de pre ssion se e n in opioid ove rdose ? IV naloxone (may need to give multiple doses as naloxone has a shorter half-life than morphine) What is the me chanism of action of naloxone ? (μ,-Receptor antagonist Which opioid antagonist is give n orally to de cre ase cravings in alcoholism? Naltrexone Which opioid analge sic is use d to pre ve nt withdrawal symptoms in patie nts discontinuing he roin use ? Methadone Which ce ntral-acting α2-agonist is use d to pre ve nt withdrawal symptoms in patie nts discontinuing he roin use ? Clonidine Give e xample s of strong opioid agonists: Morphine; fentanyl; heroin; methadone; meperidine; hydrocodone; hydromorphone Give e xample s of we ak opioid agonists: Codeine; propoxyphene Give e xample s of partial opioid agonists: Buprenorphine; pentazocine Propoxyphe ne is a de rivative of which opioid analge sic? Methadone Name two synthe tic opioid analge sics: 1. Meperidine 2. Methadone Fe ntanyl is che mically re late d to which synthe tic opioid analge sic? Meperidine Doe s morphine have a high or low oral bioavailability? Low Which two opioids should not be give n in combination with monoamine oxidase inhibitors (MAO Is)? 1. Meperidine 2. Dextromethorphan T hese combinations may produce serotonin syndrome. What drug do you ge t by ace tylating morphine ? Heroin Is code ine itse lf an active opioid analge sic? No (must be metabolized via cytochrome β-450 2D6 to active morphine) Which me dication is commonly give n in combination with code ine for the tre atme nt of pain? Acetaminophen

ANXIOLYTIC AND S EDATIVE-HYPNOTIC AGENTS For e ach of the following se dative -hypnotic-induce d CNS e ffe cts, place in orde r from e ffe ct cause d by lowe st to highe st dose of drug: coma; ane sthe sia; hypnosis; se dation/anxiolysis; me dullary de pre ssion. Sedation/anxiolysis; hypnosis; anesthesia; medullary depression; coma What is the first ste p in e thanol me tabolism? Alcohol dehydrogenase converts ethanol to acetaldehyde. What is the se cond ste p in e thanol me tabolism? Acetaldehyde dehydrogenase converts acetaldehyde to acetate. What e nz yme doe s disulfiram inhibit? Acetaldehyde dehydrogenase, leading to a build up of acetaldehyde Which me tabolite of e thanol is re sponsible for causing he adache , hypote nsion, nause a, and vomiting (“hangove r”)? Acetaldehyde

What doe s GABA stand for? Gamma-aminobutyric acid How many subunits make up the GABA re ce ptor? Five subunits Which subunit doe s GABA bind to? α-Subunit Which subunit on the GABA re ce ptor do be nz odiaz e pine s bind to? γ-Subunit (binding potentiates the affinity of the GABA receptor for GABA; does not activate the receptor alone without GABA) Which subunit on the GABA re ce ptor do barbiturate s bind to? β-Subunit (binding potentiates the affinity of the GABA receptor for GABA; does not activate the receptor alone without GABA) What physiologic proce ss take s place whe n GABA binds to the GABAAre ce ptor? Increased chloride ion influx into cells leading to membrane hyperpolarization and subsequent decreased neuronal firing What physiologic proce ss take s place whe n GABA binds to the GABAB re ce ptor? Increased potassium ion efflux out of cells leading to membrane hyperpolarization and subsequent decreased neuronal firing What me dication binds spe cifically to the GABAB re ce ptor? Baclofen What is baclofe n use d for? Muscle relaxation Do be nz odiaz e pine s pote ntiate GABA by incre asing the duration or fre que ncy of chloride ion channe l ope ning? Frequency Do barbiturate s pote ntiate GABA by incre asing the duration or fre que ncy of chloride ion channe l ope ning? Duration Name thre e nonbe nz odiaz e pine sle e p aids that spe cifically bind to the BZ1-re ce ptor subtype : 1. Eszopiclone 2. Zolpidem 3. Zaleplon Doe s z olpide m display anticonvulsant, antianxie ty, or muscle re laxant prope rtie s? No, it is a selective hypnotic along with zaleplon and eszopiclone. Do be nz odiaz e pine s have good analge sic prope rtie s? No What type s of actions do be nz odiaz e pine s display? Muscle relaxant; anticonvulsant; antianxiety; sedative-hypnotic; anterograde amnesia (midazolam); alcohol withdrawal Give e xample s of be nz odiaz e pine s: Diazepam; lorazepam; alprazolam; chlordiazepoxide; clonazepam; clorazepate; midazolam; flurazepam; flunitrazepam; temazepam; triazolam; oxazepam What is the name of the prototype be nz odiaz e pine ? Chlordiazepoxide What be nz odiaz e pine is colloquially re fe rre d to as the “date rape ” drug and is ille gal in the Unite d State s? Flunitrazepam (trade name: Rohypnol; slang: “ roofies”) Which be nz odiaz e pine is the longe st acting? Diazepam Which be nz odiaz e pine is the shorte st acting? Midazolam What thre e be nz odiaz e pine s unde rgo phase II me tabolism? 1. Lorazepam 2. Oxazepam 3. T emazepam Which be nz odiaz e pine s are commonly use d as anticonvulsants? Diazepam; clonazepam Which be nz odiaz e pine s are commonly use d to tre at alcohol withdrawal? Oxazepam; lorazepam; diazepam; chlordiazepoxide Which be nz odiaz e pine s are commonly use d as sle e p aids? T emazepam; triazolam; flurazepam Which be nz odiaz e pine s are commonly use d as anxiolytics? Diazepam; lorazepam; alprazolam Why is alpraz olam not the drug of choice whe n tre ating patie nts with chronic anxie ty? It has a short half-life, and therefore may cause withdrawal symptoms, such as anxiety, which subsequently worsens the condition and leads to higher addiction rates. Give e xample s of long-acting be nz odiaz e pine s (duration of action of 1-3 d): Diazepam; chlordiazepoxide; flurazepam; clorazepate Give e xample s of inte rme diate -acting be nz odiaz e pine s (duration of action of 10-20 h): Lorazepam; temazepam; alprazolam Give e xample s of short-acting be nz odiaz e pine s (duration of action of 3-8 h): Midazolam; oxazepam; triazolam Give e xample s of be nz odiaz e pine withdrawal signs and symptoms: Insomnia; anxiety; agitation; seizures; restlessness; confusion What are the adve rse e ffe cts of be nz odiaz e pine s? Confusion; drowsiness; ataxia; cognitive impairment; amnesia; respiratory depression Is withdrawal more like ly to occur with long-acting or short-acting be nz odiaz e pine s? Short-acting benzodiazepines (abrupt withdrawal may ensue as drug levels are rapidly decreased versus long-acting benzodiazepines which offer a “ self-tapering” mechanism which decreases the chance of withdrawal) What is the antidote for be nz odiaz e pine -induce d CNS de pre ssion? Flumazenil (short half-life; therefore, multiple administrations may be necessary) What is the me chanism of action of flumaz e nil? Benzodiazepine receptor antagonist

Will flumaz e nil de cre ase the e ffe cts of barbiturate s? No (barbiturates act at a different GABA receptor subtype than benzodiazepines) Which two se dative -hypnotic drug classe s are pote ntially fatal with ove rdose and/or withdrawal? 1. Barbiturates 2. Alcohols Benzodiazepines may be potentially fatal but to a lesser extent than barbiturates and alcohols. Give e xample s of long-acting barbiturate s (duration of action of 1-2 d): Phenobarbital; pentobarbital Give e xample s of short-acting barbiturate s (duration of action of 3-8 h): Amobarbital; secobarbital Give an e xample of an ultra-short-acting barbiturate (duration of action of 30 min): T hiopental What is phe nobarbital commonly use d to tre at? Seizures (generalized tonic-clonic and partial seizures) What is thiope ntal commonly use d for? Induction of anesthesia What are short-acting barbiturate s commonly use d for? Sedation; hypnosis What kind of drug inte ractions can barbiturate s produce ? Induction of cytochrome β-450 enzymes What are the adve rse e ffe cts of barbiturate s? Drowsiness; impair cognitive function (especially in pediatric patients); “ hangover” effect; nausea; dizziness; increase heme synthesis (contraindicated in patients with acute intermittent porphyria); coma; respiratory depression; cardiovascular depression; addiction Give e xample s of barbiturate withdrawal signs and symptoms: Insomnia; tremors; anxiety; restlessness; nausea; vomiting; seizures; cardiac arrest; delirium; hyperreflexia; agitation What drug class is use d to pre ve nt barbiturate withdrawal? Long-acting benzodiazepines O ve r-the -counte r (O TC) sle e p aids have what type s of me dications in the m? Sedating antihistamines Give e xample s of se dating antihistamine s: Diphenhydramine; doxylamine; hydroxyzine (prescription only) Give e xample s of antide pre ssants that have be e n use d for se dation and hypnosis: T razodone; amitriptyline Which me dication is a partial agonist at 5-HT1A re ce ptors and is e ffe ctive in tre ating ge ne raliz e d anxie ty disorde r? Buspirone Doe s buspirone have anticonvulsant and muscle re laxant prope rtie s? No Is buspirone se dating? No How long doe s it take for buspirone to e xe rt its anxiolytic e ffe cts? 1 to 2 weeks (therefore, not useful in treating an acute anxiety attack) What type s of withdrawal signs and symptoms doe s buspirone cause ? None

ANTIDEPRES S ANT AGENTS According to the bioge nic amine the ory, de pre ssion is due to a de ficie ncy of which two ne urotransmitte rs in the brain? 1. Serotonin (5-HT ) 2. Norepinephrine (NE) MAO A inactivate s which ne urotransmitte rs? 5-HT ; NE MAO B inactivate s which ne urotransmitte r? Dopamine (DA) What is the me chanism of action of MAO Is? Inactivation of MAO, thereby increasing levels of 5-HT , NE, and DA in presynaptic neurons with subsequent leakage of neurotransmitter into the synaptic cleft Name thre e nonse le ctive MAO Is: 1. T ranylcypromine 2. Phenelzine 3. Isocarboxazid Se le giline is commonly use d in the tre atme nt of which dise ase ? Parkinson disease MAO A also inactivate s which monoamine commonly found in ce rtain che e se s (age d), alcoholic be ve rage s, fish, chocolate s, re d wine s, and proce sse d me ats? T yramine (inactivated by MAO in the GI tract) MAO Is are the drugs of choice for tre ating what type of de pre ssion? Atypical depression How long doe s it take to se e antide pre ssant e ffe cts in patie nts who are starte d on an MAO I? 2 to 4 weeks How long should a patie nt wait from the time of discontinuing an MAO I to the time of starting a ne w antide pre ssant me dication? At least 2 weeks

What are the adve rse e ffe cts of MAO Is? Orthostatic hypotension; xerostomia; blurred vision; drowsiness; constipation; urinary retention What life -thre ate ning condition may de ve lop whe n MAO Is and se le ctive se rotonin re uptake inhibitors (SSRIs) are use d concomitantly? Serotonin syndrome What characte riz e s se rotonin syndrome ? Rigidity; diaphoresis; hyperthermia; seizures; autonomic instability; myoclonus What life -thre ate ning condition may de ve lop whe n patie nts taking MAO Is inge st foods containing tyramine ? Hypertensive crisis What characte riz e s hype rte nsive crisis? Hypertension; headache; tachycardia; nausea; vomiting; stroke; cardiac arrhythmias A se rotonin-like syndrome may de ve lop whe n MAO Is are use d concomitantly with what common O TC me dication use d to suppre ss cough? Dextromethorphan A se rotonin-like syndrome may de ve lop whe n MAO Is are use d concomitantly with which opioid analge sic? Meperidine What is the me chanism of action of tricyclic antide pre ssants (TCAs)? Inhibits reuptake of 5-HT and NE into presynaptic neurons, thereby increasing neurotransmitter concentrations in the synaptic cleft What othe r type s of ne urotransmitte r and hormonal re ce ptors, othe r than 5-HT and NE, are inhibite d by TCAs? Muscarinic; α-adrenergic; histaminergic How long doe s it take to se e antide pre ssant e ffe cts in patie nts who are starte d on a TCA? 4 to 6 weeks Give e xample s of TCAs: Amitriptyline; imipramine; clomipramine; nortriptyline; protriptyline; desipramine; doxepin What is the name of the active me tabolite of amitriptyline that se le ctive ly inhibits NE re uptake ? Nortriptyline What is the name of the active me tabolite of imipramine that se le ctive ly inhibits NE re uptake ? Desipramine Which TCA is also use d for the tre atme nt of ce rtain type s of ne uropathic pain and for the pre ve ntion of migraine he adache s? Amitriptyline Which TCA is commonly use d to tre at obse ssive -compulsive disorde r (O CD)? Clomipramine Which TCA is also use d to tre at pe diatric nocturnal e nure sis? Imipramine Why is imipramine use d for childhood e nure sis? Increases contraction of internal sphincter of bladder; decreases stage 3 and 4 sleep What are the adve rse e ffe cts of the TCAs? Xerostomia; blurred vision; urinary retention; constipation; precipitation of acute glaucoma attacks; cardiac arrhythmias; seizures (lowers seizure threshold); orthostatic hypotension with reflex tachycardia; sedation; serotonin syndrome when combined with MAOIs and/or SSRIs; coma What are the “3 C,’s” of TCA toxicity? Coma, convulsions, and cardiotoxicity Compare TCA toxicity to SSRI toxicity. SSRI toxicity is much lower than that seen with T CAs, and mortality is most often reported only when SSRIs are used in combination with another agent, such as alcohol. T his is in contrast to T CAs, which can lead to death at much lower concentrations as compared to SSRIs. What is the tre atme nt for TCA ove rdose ? Activated charcoal to absorb the drug in the GI tract; sodium bicarbonate to correct acidosis and cardiac arrhythmias; phenytoin; magnesium What is the me chanism of action of traz odone and ne faz odone ? Inhibits reuptake of 5-HT into presynaptic neurons, thereby increasing neurotransmitter concentrations in the synaptic cleft What adve rse e ffe ct is unique to traz odone ? Priapism What is the me chanism of action of bupropion? Weak inhibitor of DA, 5-HT , and NE reuptake Bupropion is also use d in what se ttings (othe r than de pre ssion)? Smoking cessation; OCD Doe s bupropion cause se xual dysfunction? No, making it a useful alternative in patients that experience sexual dysfunction with other antidepressant medications such as SSRIs Is bupropion safe to use in patie nts with e pile psy? No, because it lowers the seizure threshold. What is the me chanism of action of mirtaz apine ? α2-Adrenoceptor antagonist (results in increased release of NE and 5-HT ) What is the major side e ffe ct of mirtaz apine ? Weight gain via appetite stimulation (may be beneficial in depressed patients who become anorexic) Name an α2-antagonist that is use d in the tre atme nt of e re ctile dysfunction: Yohimbine What is the me chanism of action of ve nlafaxine ? Potent inhibitor of 5-HT and NE reuptake; weak inhibitor of DA reuptake What is the major side e ffe ct of ve nlafaxine ? Hypertension What is the me chanism of action of duloxe tine ? Selective serotonin and norepinephrine reuptake inhibitor (SSNRI) Give e xample s of SSRIs: Fluoxetine; paroxetine; fluvoxamine; sertraline; citalopram; escitalopram What is the prototype of the SSRI drug class? Fluoxetine What is the longe st-acting SSRI? Fluoxetine Fluoxe tine is also use d in what se ttings (othe r than de pre ssion)? Premenstrual dysphoric disorder (PMDD); bulimia nervosa; OCD; panic disorder; selective mutism What kind of drug inte ractions may SSRIs produce ? Inhibition of cytochrome β-450 enzymes Do SSRIs cause se xual dysfunction? Yes, they can cause anorgasmia. Do SSRIs cause cardiac arrhythmias?

No, they do not, in contrast to T CAs. What are the adve rse e ffe cts of SSRIs? Sexual dysfunction; agitation; anxiety; seizures (with overdose); nausea; vomiting; diarrhea; sedation Which SSRI is also FDA approve d for social anxie ty disorde r (SAD), ge ne raliz e d anxie ty disorde r (GAD), and posttraumatic stre ss disorde r (PTSD)? Paroxetine Which SSRIs cause the le ast amount of drug-drug inte ractions? Citalopram; escitalopram; sertraline

AGENTS FOR MANIC-DEPRES S ION Manic-de pre ssion is also known as what? Bipolar disorder What is the traditional drug of choice for tre ating manic-de pre ssion? Lithium Whe n is lithium use d in the tre atme nt of manic-de pre ssion (what phase s of the dise ase )? Prevention; treatment of manic episodes Doe s lithium have a narrow or wide the rape utic window? Narrow. T herapeutic levels range from 0.6 to 1.2 mEq/L and toxicity is seen at 1.5 mEq/L (and at 0.8 mEq/L in elderly patients). Lithium is e liminate d re nally in a similar fashion to which othe r salt? Sodium Is lithium toxicity e xace rbate d by low or high sodium plasma le ve ls? Low sodium levels What is the me chanism of action of lithium? Unknown; may decrease cAMP levels thereby decreasing its function as a second messenger; may inhibit dephosphorylation of IP 3 to IP 2 and IP 2 to IP thereby interfering with the recycling of inositol What are the adve rse e ffe cts of lithium? Acne; seizures; visual disturbances; edema; ataxia; drug-induced nephrogenic diabetes insipidus; hypothyroidism; tremors How doe s lithium cause drug-induce d ne phroge nic diabe te s insipidus? Uncoupling of the vasopressin V2 receptor in the kidney Which diure tic is usually use d to tre at ne phroge nic diabe te s insipidus? T hiazide diuretics Which drug is use d to tre at lithium-induce d ne phroge nic diabe te s insipidus? Amiloride (thiazide diuretics cause increased reabsorption of lithium which may lead to lithium toxicity) How doe s lithium cause hypothyroidism? Inhibition of 5’-deiodinase What is 5’-de iodinase re sponsible for? Converting thyroxine (T 4) to triiodothyronine (T 3) What is the name of the cardiac anomaly that may be found in ne onate s born to mothe rs using lithium? Ebstein anomaly What characte riz e s Ebste in anomaly? Malformation of the tricuspid valve leaflets (partly attached to the fibrous tricuspid valve annulus; partly attached to the right ventricular endocardium; inferior displacement of the tricuspid leaflets) What othe r me dications (mood stabiliz e rs) may be use d in the tre atme nt of manic-de pre ssion? Carbamazepine; valproic acid; benzodiazepines; gabapentin; topiramate Why are antide pre ssant age nts not use d to tre at the de pre ssion phase of manic-de pre ssion? May cause patients to enter into the manic phase

ANTIPS YCHOTIC AGENTS According to the dopamine (DA) hypothe sis of schiz ophre nia, do symptoms arise due to an e xce ss or a lack of DA in the CNS? An excess of DA What characte riz e s the positive symptoms of schiz ophre nia? Hallucinations; delusions; thought disorders What characte riz e s the ne gative symptoms of schiz ophre nia? Speech disorders; flat affect; amotivation; social withdrawal What is the me chanism of action of “typical” antipsychotic age nts? Inhibition of D2 receptors in the mesolimbic system of the brain What is the me chanism of action of “atypical” antipsychotic age nts? Inhibition of 5-HT 2 receptors (D2 receptors are still involved to some extent) Give e xample s of typical antipsychotics: Haloperidol; chlorpromazine; thioridazine; fluphenazine; pimozide Give e xample s of atypical antipsychotics: Clozapine; risperidone; aripiprazole; olanzapine; ziprasidone; quetiapine Name two typical antipsychotics commonly use d in the tre atme nt of Toure tte syndrome : 1. Haloperidol 2. Pimozide What are e xtrapyramidal symptoms (EPS)? Parkinsonian-like symptoms (dystonia, rigidity, tremor, and bradykinesia); akathisia; tardive dyskinesia (T D) What is akathisia? Motor restlessness What is tardive dyskine sia (TD)? Inappropriate movements of the tongue, neck, trunk, and limbs (associated with long-term use of DA antagonists) What is the me chanism of anti-psychotic-induce d TD? Long-term DA receptor inhibition leads to an upregulation and supersensitivity of DA receptors, thereby leading to DA overstimulation, especially when the antipsychotic has been discontinued. Which antipsychotics are more like ly to cause EPS?

High potency typical antipsychotics such as haloperidol and fluphenazine secondary to their weak anticholinergic activity Why do thioridaz ine and chlorpromaz ine have a lowe r EPS pote ntial? T hey have high anticholinergic activity. What me dications are use d to tre at antipsychotic-induce d EPS? Benztropine; amantadine; diphenhydramine (due to its anticholinergic action) How do antipsychotics cause parkinsonian-like symptoms? Inhibition of DA receptors in the nigrostriatal pathway How do antipsychotics cause orthostatic hypote nsion? Inhibition of α-adrenergic receptors in the vasculature How do antipsychotics cause prolactine mia? Inhibition of DA receptors in the anterior pituitary Which two antipsychotics posse ss the highe st antimuscarinic activity? 1. Chlorpromazine 2. T hioridazine Characte riz e the antimuscarinic activity of chlorpromaz ine and thioridaz ine : Xerostomia; blurred vision; confusion; constipation; urinary retention What is xe rostomia? Dry mouth Which antipsychotic age nts are more e ffe ctive at tre ating ne gative symptoms? Atypical antipsychotics Which phe nothiaz ine antipsychotic is also use d to tre at intractable hiccups? Chlorpromazine Which phe nothiaz ine antipsychotic may cause priapism, agranulocytosis, blue -gray discoloration of the skin, and lowe r se iz ure thre shold? T hioridazine Which two typical antipsychotics can be formulate d as de pot intramuscular inje ctions that may last up to 3 we e ks? 1. Haloperidol 2. Fluphenazine Which atypical antipsychotic is usually re se rve d as a third-line age nt to tre at schiz ophre nic patie nts re fractory to traditional the rapy? Clozapine What is the major dose -limiting side e ffe ct of cloz apine ? Agranulocytosis How are patie nts re ce iving cloz apine the rapy monitore d? WBCs obtained at baseline and weekly for the first 6 months of therapy, every other week for the second 6 months, and monthly thereafter What adve rse e ffe ct is more common with cloz apine , agranulocytosis, or se iz ure s? Seizures (occur in 10% of patients, whereas agranulocytosis occurs in l%-2% of patients) Why is gyne comastia a common adve rse e ffe ct of rispe ridone ? Risperidone induces prolactinemia (only atypical antipsychotic that increases prolactin levels significantly) What is a pote ntial life -thre ate ning adve rse e ffe ct of antipsychotic me dications? Neuroleptic malignant syndrome (NMS) What characte riz e s NMS? Hyperthermia; rigidity; altered mental status; cardiovascular instability What is the tre atme nt for NMS? Dantrolene + bromocriptine (DA agonist) What are the additional adve rse e ffe cts of olanz apine ? Weight gain; hyperglycemia; sialorrhea What are the additional adve rse e ffe cts of que tiapine ? Hypercholesterolemia; hypertriglyceridemia; weight gain; hepatotoxicity What is the me chanism of action of aripipraz ole ? Antagonist at S-HT 2A receptors; partial agonist at D2 and 5-HT 1A receptors What are the additional adve rse e ffe cts of aripipraz ole ? Weight gain; rash; sialorrhea; hepatotoxicity Which antipsychotic has the highe st incide nce of sialorrhe a? Clozapine β1%-48%)

AGENTS FOR PARKINS ON DIS EAS E What are the signs of Parkinson dise ase (PD)? Bradykinesia; muscular rigidity; tremors; gait abnormalities; postural instability What ne urotransmitte r pathway is affe cte d in PD? Dopaminergic pathway (inhibitory neurons) in the substantia nigra and corpus striatum (neurotransmitter ratio shifts toward decreased DA and increased acetylcholine [ACh]) What doe s incre ase d le ve ls of DA in the tube roinf undibular tract le ad to? Decreased prolactin levels What doe s incre ase d le ve ls of DA in the CTZ le ad to? Increased emesis What doe s incre ase d le ve ls of DA in the me solimbic-me socortical tracts le ad to? Increased psychomotor activity; psychosis; schizophrenia; increased reinforcement Which DA re ce ptor subtype is implicate d in PD? D2 receptor subtype (inhibitory receptor subtype that decreases cyclic adenosine monophosphate [cAMP] levels in the corpus striatum) Give n the abnormal DA/ACh shift in the striatum, give two pharmacologic strate gie s in the tre atme nt of PD: 1. Medications that will increase DA levels 2. Medications that will decrease ACh levels

Do anti-Parkinson me dications e ffe ct pathology, symptoms, or both? Symptoms Name two antimuscarinic me dications that are use d in the tre atme nt of PD: 1. Benztropine 2. T rihexyphenidyl Is be nz tropine more or le ss lipid soluble than atropine ? More lipid soluble (therefore, greater CNS penetration) Which characte ristic of PD is not affe cte d by anticholine rgics? Bradykinesia Is be nz tropine use ful in the tre atme nt of tardive dyskine sia? No, it may actually exacerbate tardive dyskinesia. What are the adve rse e ffe cts of be nz tropine and trihe xyphe nidyl? Xerostomia; blurred vision; constipation; urinary retention; sedation; mydriasis How doe s amantadine work in the tre atme nt of PD? May inhibit reuptake of DA into presynaptic neurons; may increase DA release from presynaptic fibers Amantadine is use d to tre at what condition othe r than PD? Influenza A What is the name of the de rmatologic adve rse e ffe ct cause d by amantadine ? Livedo reticularis What is live do re ticularis? A network-patterned discoloration of the skin caused by dilation of capillaries and venules Which characte ristic of PD is not affe cte d by amantadine ? T remors Which se le ctive MAO B inhibitor is commonly use d as first-line tre atme nt for PD? Selegiline What is the me chanism of action of se le giline ? Inhibition of DA metabolism in presynaptic neurons located in the CNS via inhibition of MAOB What are the two active me tabolite s of se le giline ? 1. Amphetamine 2. Methamphetamine What are the main adve rse e ffe cts of se le giline ? Cardiovascular stimulation (hypertension, tachycardia, palpitations; arrhythmias; angina) Name two e rgot de rivative s that act as DA agonists in the nigrostriatal syste m: 1. Bromocriptine 2. Pergolide Bromocriptine is use d to tre at what conditions othe r than PD? Prolactin-secreting microadenomas; NMS; acromegaly; postpartum lactation What are the adve rse e ffe cts of bromocriptine ? Headache; dizziness; nausea; orthostatic hypotension; dyskinesia; hallucinations; confusion; psychosis Name two none rgot DA agonists: 1. Pramipexole 2. Ropinirole What are the adve rse e ffe cts of the none rgot DA agonists? Sedation; syncope; nausea; vomiting; hallucinations; dyskinesia What is the me chanism of action of tolcapone and e ntacapone ? Inhibition of peripheral catechol-O-methyltransferase (COMT ), thereby increasing CNS uptake of L-dopa What re action doe s CO MT catalyz e ? Conversion of L-dopa to 3-O-methyldopa (partial DA agonist) in peripheral tissues What are the adve rse e ffe cts of CO MT inhibitors? Orthostatic hypotension; headache; fatigue; nausea; diarrhea; anorexia; dyskinesia; muscle cramps; brown-orange urine discoloration; hallucinations; diaphoresis Which of the CO MT inhibitors is he patotoxic? T olcapone Are CO MT inhibitors use d as first-line the rapy, adjunctive the rapy, or both in the tre atme nt of PD? Adjunctive therapy (in combination with carbidopa/levodopa) Are the none rgot DA agonists use d as first-line the rapy, adjunctive the rapy, or both in the tre atme nt of PD? Both What is the pre cursor of DA? Levodopa (L-dopa) What e nz yme conve rts L-dopa to DA? Dopa decarboxylase (DDC) Doe s DA cross the BBB? No Doe s le vodopa cross the BBB? Yes, it does and is subsequently converted to DA by dopaminergic neurons in the substantia nigra. Is le vodopa e ffe ctive in tre ating PD whe n all of the dopamine rgic ne urons in the substantia nigra have be e n de stroye d? No, since dopaminergic neurons in the substantia nigra are required to convert levodopa to DA. What is the me chanism of action of carbidopa?

Inhibition of peripheral DDC, thereby increasing the amount of levodopa that is available to cross the BBB into the CNS. T his allows for lower doses of levodopa needed, thereby decreasing levodopa adverse effects. Doe s carbidopa cross the BBB? No, it does not and therefore only inhibits peripheral DDC. How doe s le vodopa work in the tre atme nt of PD? Decreases symptoms of PD, such as rigidity, bradykinesia, and tremors What is the “on-off” phe nome non? Levodopa has such a short half-life (1-2 h) that plasma concentrations may decline rapidly causing the patient to experience sudden rigidity, bradykinesia, and tremors. Which amino acids compe te with le vodopa for GI absorption? Isoleucine; leucine What are the adve rse e ffe cts of le vodopa? Anorexia; nausea; vomiting; tachycardia; hypotension; discoloration of saliva and urine; mydriasis; hallucinations; dyskinesia; increased intraocular pressure; cardiac arrhythmias Why should vitamin B 6 (pyridoxine ) not be use d in combination with le vodopa? Pyridoxine enhances peripheral metabolism of levodopa, thereby rendering the medication ineffective.

ANTICONVULS ANT AGENTS What are the two type s of partial se iz ure s? 1. Simple 2. Complex What are the thre e type s of ge ne ral se iz ure s? 1. T onic-clonic 2. Absence 3. Myoclonic What is anothe r name for tonic-clonic se iz ure s? Grand mal seizures What is anothe r name for abse nce se iz ure s? Petit mal seizures Doe s hypove ntilation or hype rve ntilation lowe r the se iz ure thre shold? Hyperventilation (via alkalinization of blood pH) What is the most common se iz ure type ? T onic-clonic seizure What type of se iz ure is associate d with “fe brile se iz ure s”? T onic-clonic seizure What type of se iz ure most commonly pre se nts during childhood? Absence seizure What is status e pile pticus? Epileptic seizure lasting longer than 30 minutes or absence of full recovery of consciousness between seizures (can be life threatening) In ge ne ral, how do antie pile ptic age nts work? Inhibit initiation of an abnormal electrical discharge from the focal area; prevent dissemination of abnormal electrical discharge to surrounding areas of the brain What is the me chanism of action of phe nytoin? Inhibition of axonal sodium channels to produce membrane stabilization Doe s phe nytoin inhibit the axonal sodium channe l in its activate d or inactivate d state ? Inactivated state What antiarrhythmic class doe s phe nytoin be long to? Class Ib What type of drug inte ractions can phe nytoin produce ? Induces cytochrome β-450 metabolic enzymes; highly protein bound, therefore, can potentially displace other medications from plasma proteins What are the adve rse e ffe cts of phe nytoin? Diplopia; sedation; ataxia; gingival hyperplasia; acne; hirsutism; megaloblastic anemia (interferes with folate absorption); granulocytopenia; hypotension (IV); osteomalacia; drug-induced lupus; hyperglycemia; nystagmus; Stevens-Johnson syndrome; hepatotoxicity Is phe nytoin safe to use during pre gnancy? No, it causes fetal hydantoin syndrome characterized by cleft lip and palate. What prodrug is hydrolyz e d to phe nytoin and is commonly give n intrave nously se condary to its supe rior wate r solubility ve rsus phe nytoin? Fosphenytoin Name two othe r me dications that may cause gingival hype rplasia: 1. CyclosporinA 2. Nifedipine Which type s of se iz ure s doe s phe nytoin tre at? Simple partial; complex partial; tonic-clonic; status epilepticus What is the me chanism of action of carbamaz e pine ? Inhibition of axonal sodium channels to produce membrane stabilization What type of drug inte ractions can carbamaz e pine produce ? Induces cytochrome β-450 metabolic enzymes; auto-induces its own metabolism What conditions, othe r than e pile psy, can carbamaz e pine be use d in? Manic-depression; trigeminal neuralgia What are the adve rse e ffe cts of carbamaz e pine ? Diplopia; sedation; ataxia; osteomalacia; aplastic anemia; hyponatremia; Stevens-Johnson syndrome; alopecia; pancreatitis; hepatotoxicity; nystagmus Is carbamaz e pine safe to use in pre gnancy?

No, it causes neural tube defects and craniofacial abnormalities. Which type s of se iz ure s doe s carbamaz e pine tre at? Simple partial; complex partial; tonic-clonic What antie pile ptic me dication can cause alope cia? Valproic acid (VPA) What is the me chanism of action of VPA? Inhibition of axonal sodium channels; inhibition of T -type calcium channels; inhibition of GABA transaminase What type of drug inte ractions can VPA produce ? Inhibition of cytochrome β-450 enzymes What conditions, othe r than e pile psy, can VPA be use d in? Manic-depression; migraine What are the adve rse e ffe cts of VPA? Hepatotoxicity; pancreatitis; alopecia; nausea; vomiting; photosensitivity; sedation; diarrhea; abdominal pain; thrombocytopenia; rash; amenorrhea; dysmenorrhea; weight gain; tinnitus Is VPA safe to use in pre gnancy? No, it causes neural tube defects. Which type s of se iz ure s doe s VPA tre at? Simple partial; complex partial; tonic-clonic; absence; myoclonic Which antie pile ptic me dication is use d to tre at partial and tonic-clonic se iz ure s during pre gnancy? Phenobarbital Which be nz odiaz e pine s are use d in the tre atme nt of status e pile pticus? Diazepam; lorazepam Which be nz odiaz e pine is use d to tre at myoclonic and abse nce se iz ure s? Clonazepam Which antie pile ptic me dication is the drug of choice for tre ating abse nce se iz ure s and its only FDA indication is for abse nce se iz ure s? Ethosuximide What is the me chanism of action of e thosuximide ? Inhibition of T -type calcium channels in the thalamus What are the adve rse e ffe cts of e thosuximide ? EPS; nausea; vomiting; diarrhea; abdominal pain; fatigue; hirsutism; Stevens-Johnson syndrome; drug-induced lupus; hiccups Which antie pile ptics de cre ase the e fficacy of oral contrace ptive s? Phenytoin; carbamazepine How do phe nytoin and carbamaz e pine de cre ase the e fficacy of oral contrace ptive s? Induction of cytochrome β-450 enzymes O ve rdose or abrupt withdrawal of antie pile ptics may cause what adve rse e ffe ct? Seizures Give e xample s of ne we r antie pile ptic me dications: Gabapentin; pregabalin; lamotrigine; topiramate; tiagabine; vigabatrin; felbamate; levetiracetam Name two antie pile ptics that are conside re d fre e of drug-drug inte ractions: 1. Levetiracetam 2. Gabapentin Which of the ne we r antie pile ptics may inhibit carbonic anhydrase , the re by causing a me tabolic acidosis? T opiramate What conditions, othe r than e pile psy, can topiramate be use d in? Migraine; manic-depression; neuropathic pain What is the me chanism of action of topiramate ? Inhibition of glutamate (AMPA) receptors; increases GABA effects; blocks neuronal voltage-gated sodium channels Which type s of se iz ure s doe s topiramate tre at? Simple partial; complex partial; seizures associated with Lennox-Gastaut syndrome What are the adve rse e ffe cts of topiramate ? Impaired cognition; word finding difficulty; sedation; weight loss; renal stones; metabolic acidosis Which of the ne we r antie pile ptics may cause Ste ve ns-Johnson syndrome ? Lamotrigine What is the me chanism of action of lamotrigine ? Inhibition of sodium channels; inhibition of glutamate receptors Which type s of se iz ure s doe s lamotrigine tre at? Absence; simple partial; complex partial; seizures associated with Lennox-Gastaut syndrome What are the adve rse e ffe cts of lamotrigine ? Stevens-Johnson syndrome; sedation; headache; dizziness; ataxia; nausea; diplopia; amenorrhea; dysmenorrhea What ne urotransmitte r is gabape ntin structurally re late d to? GABA Doe s gabape ntin bind to the GABA re ce ptor? No What is the me chanism of action of gabape ntin? Unknown (may bind to voltage-gated calcium channels specifically possessing the α-2-δ-1 subunit) Which type s of se iz ure s doe s gabape ntin tre at? Simple partial; complex partial What conditions, othe r than e pile psy, can gabape ntin be use d in? Migraine; neuropathic pain; manic-depression; insomnia; chronic pain; social phobia What are the adve rse e ffe cts of gabape ntin? Sedation; ataxia; viral infection (in children); weight gain; diarrhea; nausea; vomiting

AGENTS FOR MIGRAINE What antihype rte nsive me dication is use d in the prophylaxis of migraine he adache ? Propranolol Give e xample s of me dications in the “triptan” drug class: Sumatriptan; almotriptan; naratriptan; rizatriptan; zolmitriptan

What is the me chanism of action of sumatriptan? 5-HT 1D-receptor agonist which causes vasoconstriction in cranial arteries Why should sumatriptan be use d with caution in a patie nt with angina? Can precipitate an anginal attack secondary to vasospasm of coronary arteries How long afte r the first dose can a se cond dose of sumatriptan be give n if migraine he adache has not re solve d? Dose may be repeated once after 2 hours. What is the me chanism of action of the e rgot alkaloids? Causes vasoconstriction of cranial arteries less selectively than “ triptans” by acting as an agonist at various 5-HT receptors throughout the body Give e xample s of e rgot alkaloids: Ergotamine; ergonovine What are some common side e ffe cts of e rgotamine ? Nausea; vomiting; diarrhea; chest pain; toxic levels can cause gangrene Why is e rgotamine some time s give n with caffe ine ? Caffeine increases gut absorption of ergotamine. Are opioid analge sics usually e ffe ctive against migraine he adache ? No

CLINICAL VIGNETTES A 39-ye ar-old man is unsatisfie d with the re sults he is se e ing from his curre nt antide pre ssant the rapy. He de cide s to stop taking the me dication and to start se e ing a ne w psychiatrist. To avoid re pe ating the same the rapy he doe s not re ve al that he was pre viously tre ate d for his major de pre ssive disorde r. The ne w psychiatrist pre scribe s a first-line SSRI to tre at the patie nt. Shortly afte r taking the SSRI, the patie nt be come s tachycardic, diaphore tic, and e xpe rie nce s myoclonic twitche s. He de ve lops a ge ne raliz e d tonic-clonic se iz ure shortly afte r admission to the e me rge ncy room. What class of me dication was the patie nt most like ly taking pre viously? T his patient was most likely taking a monoamine oxidase inhibitor (MAOI) prior to his arrival at his new psychiatrist. Results may not be seen for up to a month after initiation of treatment with an MAOI, leading to patient dissatisfaction and the belief that the medication is not effective. Should the MAOI therapy be discontinued, adequate time should be allowed for complete clearance of the drug before alternate therapy is initiated. SSRIs are particularly dangerous when combined with MAOIs because of their risk for causing serotonin syndrome, which this patient has developed. An 80-ye ar-old man has come to a cocktail party ce le brating the e nd-of-the -ye ar holidays. At the party he e njoys some hors d’oe uvre s including small sausage s and che e se s. In a fe w minute s, howe ve r, he collapse s and is rushe d to the e me rge ncy room. The re he is found to have a blood pre ssure of 200/98. His wife informs the physician that he only take s one antihype rte nsive me dication. What me dication doe s he most like ly take , and how has it contribute d to his curre nt condition? He has suffered a hypertensive crisis due to the combination of tyramine and MAOIs. Cured meats and aged cheeses, such as those consumed by the patient contain tyramine, which may displace norepinephrine from storage vesicles and cause a buildup of catecholamines, leading to extreme hypertension. A 29-ye ar-old man is unde rgoing abdominal surge ry. Shortly afte r induction and succe ssful intubation, the patie nt’s pe ak airway pre ssure s be gin to rise . Bilate ral auscultation confirms e qual bre ath sounds, and stat che st x-ray re ve als no pne umothorax. What intrave nous ane sthe tic age nt is most like ly re sponsible for this de ve lopme nt? Fentanyl, a synthetic high-potency opioid, is frequently used as a part of balanced anesthesia and can cause chest wall and laryngeal rigidity, interfering with mechanical ventilation. Low doses of opiates should be used to avoid this complication. A 72-ye ar-old woman with Parkinson dise ase is unable to move afte r sitting through a film at the movie the ate r with he r family. What is the most like ly the rapy she is re ce iving for he r dise ase , and why might the me dication be re sponsible for he r curre nt situation? What fast-acting dopamine agonist would be appropriate for use in such a situation? T he patient is most likely on levodopa therapy. Levodopa has a very short half-life leading to the “ on-off” phenomenon frequently seen with its use. Patients may find they are suddenly unable to stand or walk and may require rescue therapy with a fast-acting dopamine agonist. Apomorphine is such an agent, and is useful in emergency situations such as that presented in the vignette. It does not require enzymatic conversion to an active product, and so works quickly—about 10 minutes after injection. Nausea and vomiting limit its use to rescue situations, as does dyskinesia and hypotension.

CHAPTER 6 Cardiovascular/Renal Agents ANTIARRHYTHMIC (ANTIDYS RRHYTHMIC) AGENTS De scribe what happe ns during e ach of the following phase s of the cardiac action pote ntial for fast-re sponse fibe rs: Phase 0 Sodium ion channels open (inward) which leads to membrane depolarization. Phase I Sodium ion channels are inactivated; potassium ion channels (outward) are activated; chloride ion channels (inward) are activated. Phase II Plateau phase; slow influx of calcium ion balanced by outward potassium ion current (delayed rectifier current I K) Phase III Repolarization phase; outward K+ current increases and inward calcium ion current decreases Phase IV Membrane returns to resting potential.

Figure 6.1 O n what phase (s) of the cardiac action pote ntial do amiodarone and sotalol work? Phase 0 and phase III O n what phase (s) of the cardiac action pote ntial do lidocaine , fle cainide , and quinidine work? Phase 0 O n what phase (s) of the cardiac action pote ntial do β-blocke rs work? Phase II and phase IV What is re sponsible for maintaining the e le ctroche mical gradie nt at re sting me mbrane pote ntial? Na+ /K+ -AT Pase What ion curre nt is re sponsible for the de polariz ation of sinoatrial (SA) and atriove ntricular (AV) nodal fibe rs? Calcium ion (inward) What ion curre nt is re sponsible for the re polariz ation of SA and AV nodal fibe rs? Potassium ion (outward) How doe s phase IV of the action pote ntial in slow-re sponse fibe rs (SA and AV node s) diffe r from that of fast-re sponse fibe rs? Slow-response fibers display automaticity (ability to depolarize spontaneously); rising phase IV slope of the action potential = pacemaker potential What ion curre nt is re sponsible for the “pace make r curre nt” (rising slope of phase IV) in slow-re sponse fibe rs? Sodium ion (inward); calcium ion (inward); potassium ion (outward) The pace make r of the he art has the faste st uprising phase IV slope ; whe re is this pace make r in nondise ase d patie nts? SA node Whe re is the SA node locate d? Right atrium How do the e ffe ctive re fractory pe riod (ERP) and re lative re fractory pe riod (RRP) diffe r from e ach othe r? No stimulus, no matter the strength, can elicit a response with fibers in the ERP, whereas a strong enough stimulus will elicit a response with fibers in the RRP. What are the thre e state s the voltage -gate d Na + channe l e xists in? 1. Resting state 2. Open state 3. Inactivated state What state (s) of the voltage -gate d Na + channe l is/are most susce ptible to drugs? Open state; inactivated state What two type s of gate s doe s the voltage -gate d Na + channe l have ?

1. M (activating) 2. H (inactivating) Why is the rate of re cove ry from an action pote ntial slowe r in ische mic tissue ? T he cells are already partly depolarized at rest. What class of antiarrhythmic age nts has me mbrane -stabiliz ing e ffe cts? β-Blockers Antiarrhythmic age nts are groupe d into four classe s according to what classification syste m? Vaughn-Williams classification Give the ge ne ral me chanism of action for e ach of the following antiarrhythmic drug classe s: Class I Na+ channel blockers Class II β-Blockers Class III K+ channel blockers Class IV Ca++ channel blockers Class I antiarrhythmics are furthe r subdivide d into what classe s? la; Ib; Ic Give e xample s of antiarrhythmic drugs in class la: Quinidine (antimalarial/antiprotozoal agent); procainamide; disopyramide Give e xample s of antiarrhythmic drugs in class Ib: Lidocaine; mexiletine; tocainide; phenytoin Give e xample s of antiarrhythmic drugs in class Ic: Encainide; flecainide; propafenone; moricizine Give e xample s of antiarrhythmic drugs in class II: Propranolol; esmolol; metoprolol Give e xample s of antiarrhythmic drugs in class III: Amiodarone; sotalol; ibutilide; dofetilide Give e xample s of antiarrhythmic drugs in class IV: Verapamil; diltiazem Name thre e antiarrhythmic drugs that do not fit in the Vaughn-Williams classification syste m: 1. Digoxin 2. Adenosine 3. Magnesium Magne sium is use d to tre at what spe cific type of arrhythmia? T orsades de pointes (polymorphic ventricular tachycardia) Ade nosine is use d to tre at what type s of arrhythmias? Paroxysmal supraventricular tachycardia (PSVT ), specifically narrow complex tachycardia or supraventricular tachycardia (SVT ) with aberrancy; AV nodal arrhythmias (adenosine causes transient AV block). Note: synchronized cardioversion and not adenosine should be used on symptomatic patients or unstable tachycardia with pulses. Whe re anatomically should the IV be place d to administe r ade nosine ? As close to the heart as possible, that is, the antecubital fossa since adenosine has an extremely short half-life. Adenosine rapid IV push should be followed immediately by a 5-10 cc (mL) flush of saline to facilitate its delivery to the heart. What is the me chanism of action of ade nosine ? Stimulates α1-receptors which causes a decrease in cyclic adenosine monophosphate (cAMP) (via Gr coupled second messenger system); increases K+ efflux leading to increased hyperpolarization; increases refractory period in AV node What are the adve rse e ffe cts of ade nosine ? Flushing; chest pain; dyspnea; hypotension What two drugs can antagoniz e the e ffe cts of ade nosine ? 1. T heophylline 2. Caffeine How is ade nosine dose d? 6 mg initially by rapid IV push; if not effective within 1-2 minutes, give 12 mg repeat dose (follow each bolus of adenosine with normal saline flush). T he 12 mg dose may be repeated once. What is the most de adly ion that can be administe re d? Potassium ion What ECG change s are se e n in hype rkale mia? Flattened P waves; widened QRS complex; peaked T waves; sine waves; ventricular fibrillation What ECG change s are se e n in hypokale mia? Flattened or inverted T waves; U waves; ST -segment depression What do class la antiarrhythmics do to e ach of the following? Action pote ntial duration Increase ERP Increase Conduction ve locity Decrease Phase IV slope Decrease What do class Ib antiarrhythmics do to e ach of the following?

Action pote ntial duration Decrease ERP Little or no change Conduction ve locity Decrease (primarily in ischemic tissue) Phase IV slope Decrease What do class Ic antiarrhythmics do to e ach of the following? Action pote ntial duration Little or no change ERP Little or no change Conduction ve locity Decrease Phase IV slope Decrease Drugs that affe ct the stre ngth of he art muscle contraction are re fe rre d to as what type s of age nts? Inotropes (either positive or negative) Drugs that affe ct the he art rate are re fe rre d to as what type s of age nts? Chronotropes (either positive or negative) Drugs that affe ct AV conduction ve locity are re fe rre d to as what type s of age nts? Dromotropes (either positive or negative) Q T inte rval prolongation, and the re fore torsade s de pointe s, is more like ly to occur with what two classe s of antiarrhythmics? 1. la 2. Ill Which class la antiarrhythmic also blocks α-adre ne rgic and muscarinic re ce ptors, the re by pote ntially le ading to incre ase d he art rate and AV conduction? Quinidine What are the adve rse e ffe cts of quinidine ? T achycardia; proarrhythmic; increased digoxin levels via protein-binding displacement; nausea; vomiting; diarrhea; cinchonism What is cinchonism? Syndrome that may include tinnitus; high-frequency hearing loss; deafness; vertigo; blurred vision; diplopia; photophobia; headache; confusion; delirium What are the adve rse e ffe cts of procainamide ? Drug-induced lupus (25%-30% of patients); proarrhythmic; depression; psychosis; hallucination; nausea; vomiting; diarrhea; agranulocytosis; thrombocytopenia; hypotension What drugs can cause drug-induce d lupus? Procainamide; isoniazid (INH); chlorpromazine; penicillamine; sulfasalazine; hydralazine; methyldopa; quinidine; phenytoin; minocycline; valproic acid; carbamazepine; chlorpromazine Which class la antiarrhythmic can cause pe riphe ral vasoconstriction? Disopyr amide What are the adve rse e ffe cts of disopyramide ? Anticholinergic adverse effects, such as urinary retention; dry mouth; dry eyes; blurred vision; constipation; sedation True or False ? Lidocaine is use ful in the tre atme nt of ve ntricular arrhythmias? T rue True or False ? Lidocaine is use ful in the tre atme nt of atrial arrhythmias? False True or False ? Lidocaine is use ful in the tre atme nt of AV junctional arrhythmias? False What are the adve rse e ffe cts of lidocaine ? Proarrhythmic; sedation; agitation; confusion; paresthesias; seizures What class Ib antiarrhythmic is structurally re late d to lidocaine ? Mexiletine What class Ib antiarrhythmic can cause pulmonary fibrosis? T ocainide Propaf e none , e ve n though a class Ic antiarrhythmic, e xhibits what othe r type of antiarrhythmic activity? β-Adrenergic receptor blockade What famous trial showe d that e ncainide and fle cainide incre ase d sudde n cardiac de ath in postmyocardial infarction (MI) patie nts with arrhythmias? Cardiac Arrhythmia Suppression T rial (CAST ) Sotalol, e ve n though a class III antiarrhythmic, e xhibits what othe r type of antiarrhythmic activity? β-Adrenergic receptor blockade Eve n though this age nt is labe le d as a Vaughn-Williams class III antiarrhythmic, it displays class I, II, III, and IV antiarrhythmic activity. Amiodarone What is the half-life of amiodarone ? 40 to 60 days What are the adve rse e ffe cts of amiodarone ? Pulmonary fibrosis; tremor; ataxia; dizziness; hyperthyroidism; hypothyroidism; hepatotoxicity; photosensitivity; blue skin discoloration; neuropathy; muscle weakness; proarrhythmic; corneal deposits; lipid abnormalities; hypotension; nausea; vomiting; congestive heart failure (CHF); optic neuritis; pneumonitis; abnormal taste; abnormal smell; syndrome of inappropriate secretion of antidiuretic hormone (SIADH) How should patie nts on amiodarone the rapy be monitore d? ECG; thyroid function tests (T FT s); pulmonary function tests (PFT s); liver function tests (LFT s); electrolytes; ophthalmology examinations Ve rapamil should not be give n in what type s of arrhythmias? Wolff-Parkinson-White (WPW) syndrome; ventricular tachycardia What are the adve rse e ffe cts of ve rapamil? Drug interactions; constipation; hypotension; AV block; CHF; dizziness; flushing Digoxin is use d to control ve ntricular rate in what type s of arrhythmias? Atrial fibrillation; atrial flutter Digoxin-induce d arrhythmias are tre ate d by what drugs?

Lidocaine; phenytoin Digoxin doe s what to e ach of the following? Stre ngth of he art muscle contraction Increases (positive inotrope) He art rate Decreases (negative chronotrope) AV conduction ve locity Decreases (negative dromotrope) What doe s Q Tc stand for? Corrected QT interval How is Q Tc calculate d? (QT )/(square root of R to R interval) Why must the Q T inte rval be corre cte d? T he QT interval is dependent on heart rate, so higher heart rates will display shorter QT intervals on ECG. It is corrected to remove the variable of the heart rate. What is the normal value for Q Tc? Less than 440 milliseconds What doe s a long Q T inte rval put a patie nt at risk for? T orsades de pointes, a ventricular arrhythmia that can degenerate into ventricular fibrillation

CONGES TIVE HEART FAILURE AGENTS What is the cardiac output e quation? Cardiac output (CO) = heart rate (HR) × stroke volume (SV) What is normal CO ? 5 L/min What is the most common cause of right-side d he art failure ? Left-sided heart failure Name thre e compe nsatory physiologic re sponse s se e n in conge stive he art failure (CHF): 1. Fluid retention 2. Increased sympathetic drive 3. Hypertrophy of cardiac muscle De fine pre load: T he pressure stretching the ventricular walls at the onset of ventricular contraction; related to left ventricular end-diastolic volume/pressure De fine afte rload: T he load or force developed by the ventricle during systole What drugs are use d to de cre ase pre load? Diuretics; vasodilators; angiotensin-converting enzyme inhibitors (ACEIs); angiotensin II receptor blockers (ARBs); nitrates What drugs are use d to de cre ase afte rload? Vasodilators; ACEIs; ARBs; hydralazine What drugs are use d to incre ase contractility? Digoxin; phosphodiesterase inhibitors (amrinone and milrinone); β-adrenoceptor agonists What is the me chanism of action of digoxin? Inhibition of the Na+ /K+ -AT Pase pump which leads to positive inotropic action (via increased intracellular sodium ions that exchanges with extracellular calcium ions; resulting increase in intracellular calcium ions leads to increased force of contraction) What are the two digitalis glycoside s? 1. Digoxin 2. Digitoxin What are the adve rse e ffe cts of digoxin? Arrhythmias; nausea; vomiting; anorexia; headache; confusion; blurred vision; visual disturbances, such as yellow halos around light sources What e le ctrolyte disturbance s pre dispose to digoxin toxicity? Hypokalemia; hypomagnesemia; hypercalcemia Digoxin can cause what type s of arrhythmias? Supraventricular tachycardias; AV nodal tachycardias; AV block; ventricular tachycardias; ventricular fibrillation; complete heart block Can digoxin be use d in WPW syndrome ? No. Since digoxin slows conduction through the AV node, the accessory pathway present in WPW is left unopposed, leading to supraventricular tachycardias and atrial arrhythmias. How is digoxin toxicity tre ate d? Correction of electrolyte disturbances; antiarrhythmics; anti-digoxin Fab antibody (Digibind) What drugs can incre ase digoxin conce ntrations? Quinidine; amiodarone; erythromycin; verapamil What drugs can de cre ase digoxin conce ntrations? Loop diuretics; thiazide diuretics; corticosteroids Doe s digoxin the rapy in CHF le ad to prolonge d survival? No. It is of symptomatic benefit only, improving quality, but not necessarily duration of life. What classe s of me dications have be e n shown to incre ase survival in CHF patie nts? ACEs/ARBs; β-blockers How doe s dobutamine work in CHF? β-Adrenergic agonist (sympathomimetic that binds to (β 1-adrenoceptors) that increases force of contraction and vasodilation via increased cAMP How do amrinone and milrinone work in CHF? Inhibits phosphodiesterase (PDE) thereby increasing cAMP levels; increased cAMP leads to increased intracellular calcium; increased intracellular calcium leads to increased force of contraction; increased cAMP also leads to increased vasodilation What are the side e ffe cts of the PDEIs? Milrinone may actually decrease survival in CHF; amrinone may cause thrombocytopenia.

How do diure tics work in CHF? Decrease in intravascular volume thereby decrease in preload; reduce pulmonary and peripheral edema often seen in CHF patients How can incre ase d sympathe tic activity in CHF be counte racte d? β-Blockers What two β-blocke rs have spe cific indications for the tre atme nt of CHF? 1. Metoprolol 2. Carvedilol (mixed α-/β-blocker) What is the me chanism of action of ne siritide ? Recombinant B-type natriuretic peptide that binds to guanylate cyclase receptors on vascular smooth muscle and endothelial cells, thereby increasing cyclic guanosine monophosphate (cGMP) levels; increased cGMP leads to increased relaxation of vascular smooth muscle How do ACEIs work in CHF? Inhibition of angiotensin-II (AT -II) production thereby decreasing total peripheral resistance (T PR) and thus afterload; prevents left ventricular remodeling

ANTIANGINAL AGENTS De fine angina pe ctoris: Chest pain resulting from a myocardial oxygen demand that is not met by adequate oxygen supply; seen in patient with myocardial ischemia What type of angina is cause d by spontane ous coronary vasospasm? Prinzmetal (variant) angina What type of angina is cause d by athe roscle rosis of coronary ve sse ls and is pre cipitate d by e xe rtion? Classic angina What type of angina can be acute in onse t and is cause d by plate le t aggre gation? Unstable angina What two me chanistic strate gie s are use d in the tre atme nt of angina? 1. Increase oxygen supply to the myocardium 2. Decrease myocardial oxygen demand What type s of drugs can incre ase oxyge n supply? Nitrates; calcium channel blockers (CCBs) What type s of drugs can de cre ase oxyge n de mand? Nitrates; CCBs; β-blockers What is the drug of choice for imme diate re lie f of anginal symptoms? Sublingual nitroglycerin (NT G) What is the me chanism of action of nitrate s? Nitrates form nitrites; nitrites form nitric oxide (NO); NO activates guanylyl cyclase to increase cGMP; increased cGMP leads to increased relaxation of vascular smooth muscle How doe s cGMP le ad to re laxation of vascular smooth muscle ? Causes dephosphorylation of myosin light chains How do nitrate s incre ase oxyge n supply? Dilation of coronary vessels which leads to increased blood supply How do nitrate s de cre ase oxyge n de mand? Dilation of large veins which leads to preload reduction; decreased preload reduces the amount of work done by the heart; decreased amount of work results in decreased myocardial oxygen requirement What are the adve rse e ffe cts of nitrate s? Headache; hypotension; reflex tachycardia; facial flushing; metnemoglobinemia Why must patie nts have at le ast a 10- to 12-hour “nitrate -fre e ” inte rval e ve ry day? T olerance (tachyphylaxis) develops to nitrates if given on a continuous (around-the-clock) basis Nitrate s are contraindicate d in patie nts taking any of what thre e me dications? 1. Sildenafil 2. Vardenafil 3. T adalafil Me the moglobin formation, spe cifically by amyl nitrite , can be use d to tre at what type of poisoning? Cyanide What are the common formulations of nitrate s? NT G; isosorbide mononitrate; isosorbide dinitrate What is the time to pe ak e ffe ct of sublingual NTG? 2 minutes What is the dosing fre que ncy of sublingual NTG during an anginal e pisode ? Every 5 minutes for a maximum of three doses How do β-blocke rs work in the tre atme nt of angina? Inhibition of α1-adrenoceptors which leads to decreased CO, HR, and force of contraction, thereby reducing the workload of the heart and oxygen demand Do α-blocke rs incre ase oxyge n supply? No For e ach of the following CCBs, state whe the r the ir primary e ffe cts are on the myocardium or pe riphe ral vasculature : Ve rapamil Myocardium (greater negative inotropic effects) Dihydropyridine s (DHP; nife dipine , amlodipine , fe lodipine , isradipine , nicardipine ) Peripheral vasculature (more potent vasodilators) Diltiaz e m Myocardium How do CCBs work in the tre atme nt of angina? Block vascular L-type calcium channels which leads to decreased heart contractility and increased vasodilation

ANTIHYPERTENS IVE AGENTS According to JNC 7 guide line s, ple ase de fine the following: Normal blood pre ssure 110 mm Hg); evidence of intracranial hemorrhage; suspected aortic dissection; seizure at the onset of stroke; current use of anticoagulants or an INR >1.7; lumbar puncture within 1 week What are the the rape utic indications of thrombolytic the rapy? Acute MI; acute PE; acute ischemic stroke What is the “the rape utic time window” for administe ring thrombolytic age nts to patie nts with acute ische mic stroke ? Within the first 3 hours of the onset of symptoms Give e xample s of thrombolytic age nts: Alteplase; anistreplase; streptokinase; urokinase What two drugs can counte ract thrombolytic age nt the rapy? 1. Aminocaproic acid 2. T ranexamic acid (both agents inhibit plasminogen activation) With re gard to thrombolytic age nts, what doe s “clot-spe cific” me an? T he drug specifically activates plasminogen that is bound to fibrin in a thrombus with a low affinity for free, circulating plasminogen Which thrombolytic age nt is “clot-spe cific”? Alteplase Alte plase is also known as what? tPA (tissue plasminogen activator) What is the half-life of tPA? 5 minutes Whe re doe s tPA come from? Recombinant DNA technology What type of e nz ymatic activity doe s tPA posse ss? Serine protease activity Whe re doe s stre ptokinase come from? Group C β-hemolytic streptococci How doe s stre ptokinase work as a thrombolytic age nt? Forms a 1:1 complex with plasminogen; complexed plasminogen then converts free plasminogen into plasmin (active form) Doe s stre ptokinase have any e nz ymatic activity? No Is stre ptokinase “clot-spe cific”? No What othe r prote ins doe s the stre ptokinase -plasminoge n comple x de grade ? Fibrinogen; factor V; factor VII What laboratory value is monitore d with stre ptokinase the rapy? T hromboplastin time Why is stre ptokinase antige nic? It is recognized as a foreign protein (antigen). What adve rse re actions are spe cific to stre ptokinase ? Anaphylaxis; rash; fever

What is the half-life of anistre plase ? 90 minutes How doe s anistre plase work as a thrombolytic? Anisoyl group blocks the active site of plasminogen; as complex binds to fibrin, anisoyl group is removed and the complex becomes activated. Doe s urokinase have e nz ymatic activity? Yes O riginally, whe re did urokinase come from? Human urine Whe re doe s urokinase come from now? Fetal renal cells (human) Is urokinase antige nic? No, since it is not a foreign protein. Will stre ptokinase , at normal dose s, work in patie nts with a re ce nt history of stre ptococcal infe ction? No, because antibodies made against recent streptococcal antigens will bind to and inactivate streptokinase.

CLINICAL VIGNETTES A 72-ye ar-old man with a past me dical history of CHF pre se nts to the e me rge ncy room with colicky right-side d flank pain. Re nal ultrasonography shows a 5 mm calculus in the right ure te r. What me dication is the patie nt like ly on that has e xace rbate d this situation? What me dication could the patie nt be switche d to? Furosemide and other loop diuretics are frequently employed in the treatment of CHF to help decrease preload to the heart and vascular congestion that can cause pulmonary edema. However, loop diuretics increase the urinary excretion of calcium which can lead to renal stone formation. As this patient is presenting with a stone, if his CHF symptoms can be controlled on an alternative agent, the loop diuretics should be discontinued. A thiazide diuretic, such as hydrochlorothiazide, may provide adequate diuresis to control his CHF symptoms, while at the same time decreasing urinary calcium excretion, preventing further stone formation. A 32-ye ar-old woman is found to have a de e p ve nous thrombosis (DVT) in he r right calf and the rapy is starte d with he parin bridging the rapy with close monitoring of he r activate d partial thromboplastin time . Warfarin is adde d to the the rapy, with the goal of discontinuing he parin once the patie nt re ache s a the rape utic INR. A re pe at Dopple r ultrasonography of the right calf afte r 1 we e k of the rapy une xpe cte dly shows e xpansion of the clot. What is like ly to be found on this patie nt’s comple te blood count (CBC), and what is the appropriate the rapy? Paradoxical clot expansion or new clot formation is most likely the result of heparin-induced thrombocytopenia (HIT ) in this patient. HIT is a rare complication seen after at least 7 days of treatment with unfractionated heparin. Heparin molecules induce the formation of antiheparin antibodies (IgG). T hese antibodies bind to heparin and platelet factor 4, causing platelet activation and aggregation, with a subsequent drop in platelet numbers. T herefore, the CBC will reveal a low platelet count. Heparin therapy should be stopped immediately. Low-molecular-weight heparins such as enoxaparin are contraindicated in patients with a history of HIT, and therefore would not be appropriate alternative therapy in this patient. Here, we could measure the patient’s INR, and if therapeutic, warfarin monotherapy could be continued for 3 to 6 months. Alternative anticoagulation therapies available to patients with HIT include lepirudin, a factor Ha inhibitor, and argatroban, a synthetic direct thrombin inhibitor. A 63-ye ar-old man who is be ing tre ate d for hype rchole ste role mia with an HMG-CoA re ductase inhibitor (a statin) come s in for a follow-up visit. He be gan taking the me dication 1 we e k ago. He is complaining of some mild muscle ache s and pains, which he attribute s to the ne w e xe rcise re gime n he just starte d on your re comme ndation. Laboratory studie s show a minor incre ase in cre atine kinase (CK) activity. He asks if the ne w me dication could be causing this pain, and if he should stop taking the me dication. How should the physician advise the patie nt? Statin myopathy is a concerning but relatively rare complication of statin therapy, with an incident of 0.1% to 0.5%. Rhabdomyolysis due to statin therapy is notably less common. Rhabdomyolysis is an indication for immediate cessation of statin therapy, but myopathy is not necessarily. Myopathy usually manifests within 1 week of starting a statin, and is dose dependent. If statin therapy is successful in lowering serum cholesterol, and CK elevation is not significantly elevated above baseline, lowering the dose of the medication or switching to a different statin may be appropriate. A controversial topic is the use of coenzyme Q10 (CoQIO), or ubiquinone, to treat and prevent statin-induced myopathy. Statins also inhibit the reaction that forms CoQIO. CoQIO is found in the mitochondria of many cell types and is an antioxidant that might maintain muscle health. It has been shown in limited studies to decrease muscle pain in patients taking statins. Its long-term safety and efficacy are yet unknown.

CHAPTER 7 Pulmonary Agents DRUGS FOR AS THMA What are the classifications of asthma se ve rity? Mild intermittent; mild persistent; moderate persistent; severe persistent What are the main classifications of drugs for asthma? Bronchodilators; anti-inflammatory agents Name the drug class for e ach of the following me dications: Albute rol Short-acting β 2-adrenergic agonist Epine phrine Short-acting β 2-adrenergic agonist Te rbutaline Short-acting β 2-adrenergic agonist Salme te rol Long-acting β 2-adrenergic agonist Formote rol Long-acting β 2-adrenergic agonist Isoprote re nol Short-acting β-adrenergic agonist Me taprote re nol Short-acting β-adrenergic agonist Fluticasone Inhaled corticosteroid Flunisolide Inhaled corticosteroid Be clome thasone Inhaled corticosteroid Triamcinolone Inhaled corticosteroid Bude sonide Inhaled corticosteroid Me thylpre dnisolone Systemic corticosteroid Pre dnisone Systemic corticosteroid Cromolyn Mast cell stabilizer Ne docromil Mast cell stabilizer Ipratropium Inhaled anticholinergic Tiotropium Inhaled anticholinergic The ophylline Phosphodiesterase inhibitor; adenosine antagonist; methylxanthine Zile uton 5-Lipoxygenase inhibitor Zafirlukast Leukotriene receptor antagonist Monte lukast Leukotriene receptor antagonist How do β2-agonists he lp tre at asthma? Bronchodilation via β 2-adrenoceptor-mediated smooth muscle relaxation How do corticoste roids he lp tre at asthma? Decrease production and release of proinflammatory cytokines; decrease inflammatory cell activation, recruitment, and infiltration; decrease vascular permeability; decrease mucous production; increase number and sensitivity of β 2-adrenergic receptors How do mast ce ll stabiliz e rs he lp tre at asthma? Prevent mast cell degranulation, thereby decreasing release of histamine, platelet activating factor, leukotrienes, and other mediators that cause bronchoconstriction. T herefore only useful before exposure to allergen. How do inhale d anticholine rgics he lp tre at asthma? Competitively inhibit muscarinic receptors, thereby inhibiting vagal-mediated bronchoconstriction; reduce mucous production How do phosphodie ste rase inhibitors he lp tre at asthma? Increase cAMP which causes bronchodilation How do 5-lipoxyge nase inhibitors he lp tre at asthma? Inhibits production of leukotrienes (LT C4, LT D4, LT E4) from arachidonic acid, thereby preventing bronchoconstriction What β2-adre ne rgic agonist is commonly use d as a tocolytic age nt (stops pre mature labor by re laxing ute rine smooth muscle )? T erbutaline What cation can be use d as a tocolytic age nt? Mg2+ (Magnesium ion) Is cromolyn use d for tre atme nt or pre ve ntion of an asthma attack? Prevention Is ne docromil e ffe ctive during an acute asthma attack? No What are the side e ffe cts of mast ce ll stabiliz e rs?

Bitter taste; throat irritation What are the side e ffe cts of β2-adre ne rgic agonists? Tachycardia; muscle tremors; anxiety; arrhythmias; hyperglycemia; hypokalemia; hypomagnesemia (systemic side effects are minimized when drug is delivered via inhalation) What are the side e ffe cts of 5-lipoxyge nase inhibitors and le ukotrie ne antagonists? Increased liver function tests (LFT s); headache; Churg-Strauss syndrome What is the main nonsyste mic side e ffe ct of inhale d corticoste roids? T hrush What is thrush? Oropharyngeal candidiasis How can you pre ve nt thrush whe n using inhale d corticoste roids? Use of a spacer device; rinse mouth with water after medication use What is a possible syste mic side e ffe ct of inhale d corticoste roids in childre n? Decreased growth of long bones If using an inhale d corticoste roid and β2-adre ne rgic agonist toge the r, which do you use first? β 2-adrenergic agonist (bronchodilates the airways, thereby increasing amount of corticosteroid that is delivered to its site of action) What are the side e ffe cts of the ophylline ? T achycardia; arrhythmias; nausea; diarrhea; central nervous system (CNS) excitation (narrow therapeutic index) Give an e xample of a me thylxanthine othe r than the ophylline : Caffeine; theobromine; aminophylline Why do inhale d anticholine rgics have a minimal side e ffe ct profile ? Quaternary ammonium derivatives of atropine, therefore, do not leave the pulmonary system and cannot cross the blood-brain barrier What are e xample s of syste mic anticholine rgic side e ffe cts? Dry mouth; dry eyes; constipation; urinary retention; blurred vision; mydriasis; drowsiness; tachycardia How do you tre at β-blocke r-induce d bronchospasm? With anticholinergics such as ipratropium and tiotropium Name two drugs use d to tre at an acute asthma attack: 1. Epinephrine 2. Albuterol What is the IV form of the ophylline calle d? Aminophylline (2:1 complex of theophylline and ethylenediamine) What is the te rm use d to de scribe a se ve re asthma attack that doe s not re spond to usual asthma the rapy? Status asthmaticus How is status asthmaticus tre ate d? Oxygen; inhaled albuterol; intravenous or oral corticosteroids; inhaled anticholinergics What is the drug of choice for mild asthma? Short-acting α2-adrenergic agonist What is the mainte nance drug of choice for chronic asthma? Inhaled glucocorticoid How is the ophylline primarily me taboliz e d? Hepatic cytochrome P-450 enzymes (CYP 1A2 and CYP 3A4) Give e xample s of me dications that can le ad to incre ase d the ophylline le ve ls whe n use d concomitantly: Cimetidine; erythromycin; fluoroquinolones What drug can cause asthma, nasal polyps, and rhinitis? Aspirin (“ aspirin triad”); seen in the rare case of aspirin sensitivity where inhibition of cyclooxygenase (COX) leads to a buildup of leukotrienes

CHRONIC OBS TRUCTIVE PULMONARY DIS EAS E AGENTS What dise ase proce sse s fall unde r the cate gory of chronic obstructive pulmonary dise ase (CO PD)? Asthma; chronic bronchitis; emphysema State whe the r the following pulmonary function te sts (PFTs) will be incre ase d, de cre ase d, or re main unchange d in patie nts with CO PD: Force d e xpiratory volume in 1 se cond (FEV1) Decreased Force d vital capacity (FVC) Unchanged or increased FEV1/FVC Decreased ( 50) taking high doses of PPIs for more than a year were 2.6 times as likely to break a hip as were people not taking PPIs. Histamine H2-receptor inhibitors also increased fracture risk, but not to the extent as did PPIs. What is the most se rious side e ffe ct of cisapride ? Prolongation of the QT interval Cisapride should be avoide d in which type of patie nts? Patients with prolonged QT intervals; patients taking medications that inhibit cytochrome P-ISO 3A4 (fluconazole, ketoconazole, itraconazole, erythromycin, clarithromycin, ritonavir) What arrhythmia can be cause d by prolongation of the Q T inte rval? T orsades de pointes (a polymorphic ventricular tachycardia)

Which drugs incre ase cisapride blood le ve ls by inhibiting the cytochrome P-450 3A4 e nz yme s that me taboliz e cisapride ? (Ple ase me ntion at le ast four drugs). 1. 2. 3. 4. 5. 6. 7. 8. 9. 10. 11.

Erythromycin Clarithromycin Itraconazole Fluconazole Ketoconazole Indinavir Ritonavir Class 1A antiarrhythmics Class III antiarrhythmics Certain T CAs Certain antipsychotics

AGENTS FOR PEPTIC ULCER DIS EAS E What thre e me diators can stimulate acid se cre tion from parie tal ce lls? 1. Acetylcholine 2. Histamine (via H2 receptor) 3. Gastrin Name thre e cause s of pe ptic ulce r dise ase (PUD): 1. Helicobacter pylori infection (primary cause) 2. Nonsteroidal anti-inflammatory drugs (NSAIDs) 3. Extreme physiologic stress (ie, patients in the ICU setting being ventilated, burn patients) What type of patie nts do acute pe ptic ulce rs occur in? Hospitalized patients who are critically ill (stress ulcers) What is the name of the syndrome that is characte riz e d by hype rse cre tion of gastric acid se condary to a gastrin-se cre ting tumor? Zollinger-Ellison syndrome Which are the drug the rapy goals in tre ating PUD? Control H. pylori infection; alleviate symptoms; promote healing; prevent recurrences; prevent complications (eg, hemorrhage) What type s of me dications are use ful for the tre atme nt of PUD? Antimicrobial agents; H2-receptor antagonists; PPIs; prostaglandins; antimuscarinic agents; antacids; mucosal protective agents; bismuth salts How might H. pylori play a role in pe ptic ulce r de ve lopme nt? Direct mucosal damage; alterations in inflammatory response; induced hypergastrinemia Me als worse n the pain associate d with what type of ulce r? Gastric ulcer Me als re lie ve the pain associate d with what type of ulce r? Duodenal ulcer What tre atme nt options are available to e radicate H. pylori? T riple therapy with a PPI added to two antimicrobial agents such as metronidazole, amoxicillin, tetracycline, or clarithromycin; four-drug regimens consisting of triple therapy plus bismuth subsalicylate; (must use triple or quadruple antibiotic therapy to eradicate H. pylori) Why should you not give bismuth subsalicylate to childre n? May be associated with Reye syndrome (contains salicylates) What is Re ye syndrome ? Acute onset encephalopathy and fatty liver formation. Symptoms begin with vomiting, lethargy, and confusion progressing to stupor, respiratory distress, coma, and seizures. Its cause is unknown, but has been found to be associated with aspirin use in young children. T herefore, aspirin administration is to be avoided in pediatric patients. How do prostaglandins he lp tre at PUD? Prostaglandins such as PGE2 and PGI 2inhibit gastric acid secretion and stimulate secretion of bicarbonate and mucus (cytoprotective activity); used to treat NSAID-induced peptic ulcers Which prostaglandin analog is commonly use d as a cytoprote ctive age nt for the tre atme nt of PUD? Misoprostol (synthetic PGE1 analog) Why should misoprostol not be give n to a pre te rm pre gnant woman? Induction of premature uterine contractions (abortifacient properties)

AGENTS FOR INFLAMMATORY BOWEL DIS EAS E What are the two forms of inflammatory bowe l dise ase (IBD)? 1. Crohn disease 2. Ulcerative colitis (UC) Doe s tre atme nt of IBD cure or control the dise ase proce ss? Control What type s of me dications are use d to tre at IBD? Corticosteroids; aminosalicylates; immunosuppressives; monoclonal antibodies Sulf asalaz ine is cle ave d by gut bacte ria in the colon to produce what two compounds?

1. Sulfapyridine (sulfonamide antibiotic) 2. Mesalamine (5-aminosalicylic acid, 5-ASA) What is the active compone nt of sulfasalaz ine for IBD? Mesalamine or 5-ASA; 5-ASA is the metabolite active against IBD, while sulfapyridine is the metabolite active against rheumatoid arthritis. Formulation into sulfasalazine is necessary to prevent rapid proximal gut absorption so that sufficient 5-ASA is delivered to the distal gut to effectively treat IBD. How doe s me salamine work in the tre atme nt of IBD? Anti-inflammatory effects; immunomodulating effects What type of vitamin supple me ntation should patie nts re ce ive while on sulfasalaz ine ? Folic acid since sulfasalazine may interfere with absorption of folic acid in the gut, leading to megaloblastic anemia What type s of immunosuppre ssive s are use d to tre at IBD? Cyclosporine A; methotrexate; azathioprine; 6-mercaptopurine What is the name of the monoclonal antibody indicate d for the tre atme nt of Crohn dise ase ? Infliximab What is infliximab’s me chanism of action? Monoclonal antibody that binds to soluble and bound forms of tumor necrosis factor-alpha (T NF-α) True or False ? O nce re mission has be e n achie ve d with ulce rative colitis, corticoste roids are use d as mainte nance the rapy. False. Corticosteroids should not be used to maintain disease remission due to their high systemic toxicity. Aminosalicylates or immunosuppressive agents are used for maintenance therapy for UC.

AGENTS FOR NAUS EA AND VOMITING Name a major che mose nsory are a for e me sis: Chemoreceptor trigger zone (CT Z) Whe re is the CTZ found? Area postrema of the fourth ventricle of the brain Give e xample s of drug classe s that are e ffe ctive in the tre atme nt of nause a and vomiting: Antihistamine-anticholinergics; benzodiazepines; butyrophenones; cannabinoids; corticosteroids; phenothiazines; substituted benzamides; 5-HT 3-receptor antagonists; neurokinin receptor antagonists Give e xample s of spe cific drugs in e ach of the following drug classe s use d in the tre atme nt of nause a and vomiting: Antihistamine -anticholine rgics Diphenhydramine; hydroxyzine; meclizine; cyclizine; promethazine; pyrilamine; scopolamine; trimethobenzamide Be nz odiaz e pine s Alprazolam; diazepam; lorazepam Butyrophe none s Haloperidol; droperidol; domperidone Cannabinoids Dronabinol; nabilone Corticoste roids Dexamethasone; methylprednisolone Phe nothiaz ine s Prochlorperazine; chlorpromazine; perphenazine Substitute d be nz amide s Metoclopramide 5-HT3- -re ce ptor antagonists Ondansetron; dolasetron; granisetron Ne urokinin re ce ptor antagonists Aprepitant (oral); fosaprepitant (IV formulation converted to aprepitant) Do synthe tic cannabinoids have psychotropic activity? No How doe s me toclopramide work as an antie me tic? Blocks dopamine receptors centrally in the CT Z What is intractable e me sis le ading to de hydration and hypote nsion during pre gnancy calle d? Hyperemesis gravidarum What are the drugs of choice for tre ating e me sis during pre gnancy? Meclizine; cyclizine; promethazine What antihistamine is ofte n use d to tre at motion sickne ss? Meclizine What anticholine rgic is ofte n use d to tre at motion sickne ss? Scopolamine How is scopolamine normally administe re d? As a transdermal patch to prevent systemic anticholinergic effects What me dication is ofte n use d in combination re gime ns to e nhance antie me tic activity? Dexamethasone What are the side e ffe cts of cannabinoids? Anxiety; memory loss; confusion; motor incoordination; hallucinations; euphoria; relaxation; hunger; gynecomastia What are the side e ffe cts of the phe nothiaz ine antie me tics? Extrapyramidal symptoms; sedation; hypotension Why doe sn’t ondanse tron cause e xtrapyramidal side e ffe cts? Blocks 5-HT 3 instead of dopamine receptors in the CT Z What che mothe rapy age nt has one of the highe st e me toge nic pote ntials? Cisplatin What ove r-the -counte r (O TC) me dication can be give n in combination with me toclopramide to re duce its e xtrapyramidal side e ffe cts? Diphenhydramine can be used for its anticholinergic properties. EPS symptoms with metoclopramide use are due to central dopamine receptor blockade, and tardive dyskinesia, if it develops, may be irreversible. T herefore, metoclopramide should only be used for short-term therapy if possible. What macrolide antibiotic also has prokine tic prope rtie s for the GI tract? Erythromycin, though tolerance to this effect develops rapidly, limiting its usefulness

AGENTS FOR DIARRHEA AND CONS TIPATION

Name thre e classe s of drugs that are e ffe ctive in the tre atme nt of diarrhe a: 1. Adsorbents 2. Antimotility agents 3. Antisecretory agents De fine adsorbe nt: A substance offering a suitable active surface, upon which other substances may adhere to Give e xample s of spe cific drugs in e ach of the following drug classe s use d in the tre atme nt of diarrhe a: Adsorbe nts Kaolin; pectin; polycarbophil; attapulgite Antimotility age nts Diphenoxylate; loperamide; morphine Antise cre tory age nts Bismuth subsalicylate Give the antidiarrhe al me chanism of action for e ach of the following drug classe s: Adsorbe nts Adsorbs (adheres to) drugs, nutrients, toxins, and digestive juices Antimotility age nts Decrease peristalsis by activating presynaptic opioid receptors in the enteric nervous system Antise cre tory age nts Decrease fluid secretion in the bowel What adsorbe nt can absorb 60 time s its we ight in wate r and tre at both diarrhe a and constipation? Polycarbophil What are the pote ntial side e ffe cts of bismuth subsalicylate ? Salicylism (tinnitus, nausea, vomiting); darkening of tongue; darkening of stools; induce gout attacks in susceptible patients What antidiarrhe al can de cre ase te tracycline absorption if give n concomitantly? Bismuth subsalicylate What antidiarrhe al is ofte n formulate d in combination with atropine ? Diphenoxylate Which class of antidiarrhe als can cause paralytic ile us? Antimotility agents What me dication is ofte n use d to tre at flushing and diarrhe a se e n in carcinoid syndrome and vasoactive inte stinal pe ptide se cre ting tumors (VIPomas)? Octreotide What is octre otide ’s me chanism of action? Synthetic analog of somatostatin which blocks release of serotonin and other vasoactive peptides; direct inhibitory effects on intestinal secretion; direct stimulatory effects on intestinal absorption What are the non-antidiarrhe al use s of octre otide ? Esophageal varices; acromegaly What me dication can be use d in conjunction with antibiotics to bulk stools and absorb Clostridium difficile toxins A and B in C. difficile colitis? Cholestyramine, a nonabsorbable binding agent What type s of me dications cause constipation? Opioid analgesics; anticholinergics; calcium-containing antacids; aluminum-containing antacids; calcium channel blockers; clonidine; iron; sodium polystyrene sulfonate Give e xample s of drug classe s that are e ffe ctive in the tre atme nt of constipation: Bulk forming agents; irritants and stimulants; stool softeners Give e xample s of spe cific drugs in e ach of the following drug classe s use d in the tre atme nt of constipation: Bulk forming age nts Methylcellulose; psyllium; bran; magnesium-containing salts; polyethylene glycol O smotic laxative s Lactulose; magnesium hydroxide (milk of magnesia); sorbitol; magnesium citrate; sodium phosphate; polyethylene glycol Irritants and stimulants Cascara; senna; aloe; bisacodyl Stool softe ne rs Mineral oil; docusate (oral or enema, trade name: Colace); glycerin suppository Cl − channe l activators Lubiprostone (Amitiza) Give the me chanism of action for e ach of the following drug classe s: Bulk-forming age nts Form gels in large intestine which causes water retention and intestinal distention, thereby increasing peristaltic activity O smotic laxative s Nonabsorbable compounds which draw fluid into the colon to maintain osmotic neutrality Irritants and stimulants Irritate gut lining which subsequently increases peristalsis Stool softe ne rs Surfactants that become emulsified with stool, thereby softening feces Cl channe l activators Activate CIC-2 Cl− channels in the apical membrane of intestinal cells increasing fluid and intestinal motility without altering serum Na+ or K+ levels. T he effects are localized to the GI tract, increase fluid secretion into the intestinal lumen, and accelerate fecal transit. What are the pote ntial side e ffe cts of bisacodyl? Abdominal cramping; atonic colon

CLINICAL VIGNETTES A 50-ye ar-old woman is in the surgical inte nsive care unit (ICU) status post right middle ce re bral arte ry he morrhagic stroke . She is be ing me chanically ve ntilate d. He r morning laboratorie s come back with a plate le t count of 50,000 (normal 150,000-450,000). Give n he r history, she is not on de e p ve nous thrombosis (DVT) prophylaxis with he parin. What othe r prophylactic me dication, commonly e mploye d in a critical care se tting, may be re sponsible for he r drop in plate le t count?

Patients in ICUs are routinely put on DVT and GI prophylaxis given the prolonged immobility and high physiologic stress of intensive care. Pharmacologic DVT prophylaxis is not appropriate in this patient, and mechanical methods such as sequential compression stockings should be used instead. However, this patient was most likely started on omeprazole for prevention of stress-induced gastric ulcers. Omeprazole can cause thrombocytopenia. T his patient should be switched to a different GI prophylactic medication. Esomeprazole, the S-enantiomer of omeprazole, while controversial as to whether or not it is more effective to inhibit stomach acid secretion, does not carry the same risk of lowering platelets. A 60-ye ar-old woman with a past me dical history of rhe umatoid arthritis (RA) and chronic iron de ficie ncy ane mia is found to have blood in he r stool. Colonoscopy is ne gative , but a gastric ulce ration is discove re d upon uppe r e ndoscopy. Afte r succe ssful tre atme nt of the ulce r, what me dication could be adde d to the patie nt’s re gime n to pre ve nt a re pe at ulce r or gastric pe rforation? Patients with a chronic inflammatory disease such as RA are often successful in relieving their pain symptoms with long-term treatment with NSAIDs. However, this therapy can lead to gastric ulceration and chronic gastrointestinal bleeding that can lead to iron deficiency anemia. T his anemia is often overlooked in patients with chronic autoimmune disorders as anemia of chronic disease. All GI bleeding must be considered colon cancer until proven otherwise, especially in patients older than 50 years, but once ruled out the next most likely location in this patient is a gastric ulcer due to disruption of the gastric mucosa by long-term NSAID administration. Because of the patient’s RA, NSAID cessation is difficult. In patients such as this, addition of misoprostol, a prostaglandin analog, may be appropriate. An 86-ye ar-old woman with a history of chronic constipation is tre ate d with lactulose with good re lie f of he r constipation. Howe ve r, she complains of painful abdominal cramps and e mbarrassing flatus and wishe s to try anothe r me dication. What laxative has a similar me chanism of action to lactulose and will not cause large e le ctrolyte imbalance s, making it safe to use in this e lde rly patie nt? Polyethylene glycol (PEG) is an osmotic laxative used in high concentration formulas as a bowel cleanser prior to endoscopic procedures. It is effective in lower concentrations for treatment of chronic constipation. Unlike lactulose and sorbitol, it is not metabolized by colonic bacteria, decreasing colonic gas formation. It does not cause large fluid or electrolyte shifts. T herefore, unlike magnesium citrate, sodium phosphate, or magnesium hydroxide, PEG does not carry a risk for electrolyte disturbances, making it an ideal choice for elderly patients and/or patients with renal insufficiency.

CHAPTER 9 Endocrine Agents AGENTS FOR DIABETES MELLITUS What are the two ge ne ral cate gorie s of drugs that are use d to tre at diabe te s me llitus? 1. Insulin 2. Oral hypoglycemic agents Which type of diabe te s me llitus is e ach of the following state me nts re fe rring to? Loss of pancre atic β-ce lls T ype 1 Usually e arly onse t T ype 1 De cre ase d re sponse to insulin T ype 2 Ke toacidosis prone T ype 1 Usually adult onse t T ype 2 Not ke toacidosis prone T ype 2 Absolute de pe nde nce on insulin T ype 1 May be controlle d by die t and oral hypoglyce mics alone T ype 2 Usually thin T ype 1 Usually obe se T ype 2 Isle t ce ll antibodie s T ype 1 Ne ar 100% concordance in monoz ygotic twins T ype 2 What type s of drugs can e le vate blood glucose conce ntrations? Alcohol; β-adrenergic blockers; calcium channel blockers; combination oral contraceptives; diazoxide; diuretics; corticosteroids; lithium; niacin; phenytoin; sympathomimetics What are the signs and symptoms of diabe tic ke toacidosis? Kussmaul respirations; fruity breath; abdominal pain; nausea; vomiting; polyuria; polydipsia; dehydration; fatigue What che mical is re sponsible for causing “fruity bre ath” during ke toacidosis? Acetone What are the thre e ke tone s made during ke toacidosis? 1. β-Hydroxybutyric acid 2. Acetoacetic acid 3. Acetone What is the te rm use d to de scribe a rise in blood glucose usually be twe e n 4 and 11 AM due to the re le ase of growth hormone , cortisol, glucagons, and e pine phrine ? Dawn phenomenon. To determine the cause of elevated morning blood sugars, the patient must measure their glucose levels throughout the night. T hen alterations in diet, medication doses, or medication choice may be made. What is the te rm use d to de scribe a re bound rise in morning blood glucose se condary to a low ove rnight blood glucose ? Somogyi effect. T his usually results from hyperinsulinemia which decreases blood glucose. Glucagon is released when the patient becomes hypoglycemic, which causes a rebound spike in blood glucose levels. Decreasing the evening insulin dose is first-line therapy. For e ach of the following type s of insulin give the time of onse t, pe ak e ffe ct, and duration: Aspart 0.17 to 0.33 hours; 1 to 3 hours; 3 to 5 hours Lispro 0.25 hours; 0.5 to 1.5 hours; 6 to 8 hours Re gular 0.5 to 1 hours; 2 to 3 hours; 8 to 12 hours NPH (isophane insulin suspe nsion) 1 to 1.5 hours; 4 to 12 hours; 24 hours Le nte (insulin z inc suspe nsion) 1 to 2.5 hours; 8 to 12 hours; 18 to 24 hours Ultrale nte (e xte nde d insulin z inc suspe nsion) 4 to 8 hours; 16 to 18 hours; > 36 hours Glargine No peak; duration is 24 hours Can insulin glargine be mixe d with othe r insulins? No What is the most common side e ffe ct of insulin? Hypoglycemia What are the signs and symptoms of hypoglyce mia? Confusion; diaphoresis; tremors; tachycardia; seizures; coma; lethargy

Which sign/symptom of hypoglyce mia is not maske d by β-adre ne rgic antagonists? Diaphoresis What is the name of the incre tin mime tic that incre ase s insulin se cre tion, slows gastric e mptying, and de cre ase s food intake ? Exenatide What is the name of the human amylin analog that is cose cre te d with insulin and re duce s postprandial glucose by prolonging gastric e mptying time , re duce s postprandial glucagon se cre tion, and suppre sse s appe tite ? Pramlintide What hypoglyce mic age nt is not contraindicate d in a pre gnant woman with diabe te s me llitus? Insulin O the r than blood glucose re duction, re gular insulin can also be use d for what condition? Hyperkalemia. Insulin causes an intracellular shift of potassium. Insulin is given in combination with glucose to prevent hypoglycemia in this situation. For e ach of the following oral hypoglyce mic age nts, state which drug class it be longs to? Chlorpropamide First-generation sulfonylurea Tolaz amide First-generation sulfonylurea Tolbutamide First-generation sulfonylurea Glyburide Second-generation sulfonylurea Glipiz ide Second-generation sulfonylurea Glime piride Second-generation sulfonylurea Nate glinide D-phenylalanine derivative Rosiglitaz one T hiazolidinedione Pioglitaz one T hiazolidinedione Acarbose α-Glucosidase inhibitor Miglitol α-Glucosidase inhibitor Me tformin Biguanide Re paglinide Meglitinide Nate glinide Meglitinide Pramlintide Amylin analog Exe natide Incretin For e ach of the following drug classe s, give the me chanism of action: Sulfonylure as Block adenosine triphosphate (AT P)-dependent potassium channels, thereby depolarizing pancreatic β-cells which lead to insulin release (release mediated via calcium influx); insulin secretagogue Thiaz olidine dione s Bind to nuclear peroxisome proliferator activating receptor-gamma (PPAR-γ) which leads to increased sensitization of cells to insulin; decrease hepatic gluconeogenesis; upregulate insulin receptors D-phe nylalanine de rivative s Newest insulin secretagogue which closes the potassium channels on the β-cells leading to a rapid but short-acting release of insulin α-Glucosidase inhibitors Inhibit intestinal amylase and α-glucosidase causing a delay in the breakdown of complex carbohydrates into glucose which subsequently delays glucose absorption, thereby lowering postprandial glucose levels Biguanide s Decrease hepatic gluconeogenesis; increase tissue sensitivity to insulin Me glitinide s Nonsulfonylurea insulin secretagogue Amylin analogs Suppresses glucagon release, delays gastric emptying, decreases hunger Incre tins Synthetic glucagon-like-polypeptide (GLP-1) analogs which potentiates glucose-mediated insulin release, decreases postprandial glucagon release, decreases gastric emptying, decreases hunger What are the side e ffe cts of the sulfonylure as? Hypoglycemia; cross-reaction with sulfonamide allergy; weight gain What is the longe st acting sulfonylure a? Chlorpropamide Which sulfonylure a can cause disulfiram-like re actions? Chlorpropamide Which sulfonylure a can cause syndrome of inappropriate se cre tion of antidiure tic hormone (SIADH)? Chlorpropamide The dose of what se cond-ge ne ration sulf onylure a should be de cre ase d in patie nts with re nal dysfunction? Glyburide The dose of what se cond-ge ne ration sulf onylure a should be de cre ase d in patie nts with he patic dysfunction? Glipizide What are the side e ffe cts of the thiaz olidine dione s (TZDs)? Edema; congestive heart failure (CHF) exacerbation; weight gain; hepatotoxicity; macular edema (rare); increased bone fractures in women due to diminished osteoblast formation Name two concomitant he alth conditions in which TZDs may not be use d in a diabe tic patie nt.

1. CHF 2. Liver failure What are the side e ffe cts of the biguanide s? Diarrhea; lactic acidosis; decreased vitamin B12; abnormal taste What are the side e ffe cts of the me glitinide s? Hypoglycemia; upper respiratory tract infection What are the side e ffe cts of the α-glucosidase inhibitors? Abdominal cramping; diarrhea; flatulence What drug with positive inotropic and chronotropic activity can be use d to stimulate the he art during a β-blocke r ove rdose ? Glucagon Base d on live r function te st (LFT) re sults, whe n should the rapy with a thiaz olidine dione be withhe ld? When LFT s rise above 2.5 times the upper limit of normal What oral hypoglyce mic should be withhe ld whe n the se rum cre atinine is above 1.5 for male s and 1.4 for fe male s? Metformin Whe n use d alone , can TZDs cause hypoglyce mia? No, T ZDs do not cause hypoglycemia. Whe n use d alone , can biguanide s cause hypoglyce mia? No, biguanides do not cause hypoglycemia. What time of day should me glitinide s be give n? 15 to 30 minutes before each meal Would oral sucrose be e ffe ctive in a hypoglyce mic patie nt who is curre ntly taking an α-glucosidase inhibitor? No, it would not because sucrose is a disaccharide whose absorption is competitively blocked by the α-glucosidase inhibitors. Would oral glucose be e ffe ctive in a hypoglyce mic patie nt who is curre ntly taking an α-glucosidase inhibitor? Yes, it would since glucose is a monosaccharides. α-Glucosidase inhibitors do not block intestinal transporters for monosaccharides.

AGENTS FOR DIABETES INS IPIDUS What are the two type s of diabe te s insipidus? 1. Neurogenic 2. Nephrogenic Which type of diabe te s insipidus is characte riz e d by inse nsitivity to vasopre ssin in the colle cting ducts? Nephrogenic diabetes insipidus Which type of diabe te s insipidus is characte riz e d by inade quate se cre tion of vasopre ssin from the poste rior pituitary gland? Neurogenic diabetes insipidus What type s of drugs can cause a ne phroge nic diabe te s insipidus? Lithium; demeclocycline; vincristine; amphotericin B; alcohol What two e ffe cts doe s vasopre ssin have on the body? 1. Antidiuresis 2. Vasopressor Whe re is the V1re ce ptor found? Vascular smooth muscle (causes vasoconstriction) Whe re is the V2re ce ptor found? Renal collecting ducts (increases water permeability and reabsorption) What is the drug of choice for ne uroge nic diabe te s insipidus? Desmopressin What is de smopre ssin? Synthetic analog of vasopressin with longer half-life and no vasopressor activity (antidiuresis properties only) What is anothe r name for de smopre ssin? l-Deamino-8-D-arginine vasopressin (DDAVP) How is de smopre ssin administe re d? Intranasally; orally What are the side e ffe cts of vasopre ssin? Water intoxication; hyponatremia; tremor; headache; bronchoconstriction What is vasopre ssin also use d for? Esophageal varices What is de smopre ssin also use d for? Hemophilia A; von Willebrand disease; primary nocturnal enuresis How is ne phroge nic diabe te s insipidus tre ate d? T hiazide diuretics in combination with amiloride; chlorpropamide; clofibrate What drug that can cause ne phroge nic diabe te s insipidus is use d to tre at SIADH? Demeclocycline

AGENTS FOR THYROID DIS ORDERS What are the signs and symptoms of hype rthyroidism? Heat intolerance; nervousness; fatigue; weight loss with increased appetite; increased bowel movements; palpitations; irregular menses; proximal muscle weakness; moist skin; fine hair; hyperactive deep tendon reflexes; tachycardia; widened pulse pressure; tremor What are the signs and symptoms of hypothyroidism? Growth retardation in children; slowing of physical and mental activity; weight gain; cold intolerance; constipation; weakness; depression; dry skin; cold skin; coarse skin; coarse hair; bradycardia; muscle cramps; delayed relaxation of deep tendon reflexes What are the signs and symptoms of thyroid storm? High fever; dehydration; delirium; tachycardia; tachypnea; nausea; vomiting; diarrhea; coma

What are the two main active thyroid hormone s circulating in the body? 1. T hyroxine (T 4) 2. T riiodothyronine (T 3) Which thyroid hormone is more active in the body? T 3 (up to five times more active) What is the half-life of T4? 7 days What is the half-life of T3? 1.5 days What is the name of the e nz yme that conve rts T4 to T 3 in the pe riphe ry? 5’-Deiodinase (5 “ prime” deiodinase) What is the name of the e nz yme that conve rts active T4 to inactive re ve rse T3? 5-Deiodinase What is the drug of choice for hypothyroidism? Levothyroxine (T 4) Whe n a hypothyroid patie nt is starte d on le vothyroxine the rapy, how long will the drug take to re ach a ste ady state ? 6 to 8 weeks What are the adve rse e ffe cts of le vothyroxine ? Same effects as physiologic hyperthyroidism: heat intolerance; nervousness; fatigue; weight loss with increased appetite; increased bowel movements; palpitations; irregular menses; proximal muscle weakness; moist skin; fine hair; hyperactive deep tendon reflexes; tachycardia; widened pulse pressure; tremor What bioche mical marke r is use d to asse ss for ade quate thyroid re place me nt? T hyroid-stimulating hormone (T SH) What antiarrhythmic age nt can pote ntially cause e ithe r hypothyroidism or hype rthyroidism (more commonly hypothyroidism)? Amiodarone (contains two iodine molecules) To which thre e prote ins in the blood are T4 and T3 bound e xte nsive ly? 1. T hyroid-binding globulin 2. T hyroid-binding prealbumin 3. Albumin The α-subunit of TSH is similar to the α-subunits of which hormone s (gonadotropins) in the body? Follicle-stimulating hormone (FSH); luteinizing hormone (LH); human chorionic gonadotropin (hCG) What drug is use d to ablate thyroid tissue ? Radioactive iodine ( 131I) What drug is use d to tre at cardiovascular e ffe cts se e n in thyrotoxicosis? Propranolol What is a side e ffe ct of surgical re moval of the thyroid gland? Hypothyroidism almost inevitably results. Other possible complications include hypocalcemia due to removal of the parathyroid glands along with the thyroid. What are the drugs of choice to tre at hype rthyroidism? T hionamides (propylthiouracil and methimazole) What drugs inhibit the re le ase of pre forme d thyroid hormone ? Iodide; lithium What drugs inhibit the iodination of tyrosyl re sidue s on thyroglobulin? Propylthiouracil; methimazole What drugs inhibit the coupling re actions that form T3 and T4? Propylthiouracil; methimazole; iodide What drugs block the conve rsion of T4 to T3 in the pe riphe ry by inhibiting 5’-de iodinase ? Propylthiouracil; propranolol What are the side e ffe cts of propylthiouracil and me thimaz ole ? Pruritic maculopapular rash; vasculitis; arthralgias; fever; leukopenia; agranulocytosis Do propylthiouracil and me thimaz ole cross the place nta? Yes What is the drug of choice for hype rthyroidism in pre gnancy? Propylthiouracil (more protein bound) What drugs can de cre ase le ve ls of thyroid-binding globulin? Androgens; glucocorticoids; L-asparaginase What drugs can incre ase le ve ls of thyroid-binding globulin? Estrogens; perphenazine; clofibrate; fluorouracil

ADRENAL S TEROIDS What are the thre e z one s of the adre nal corte x? 1. Zona glomerulosa (outer) 2. Zona fasciculata (middle) 3. Zona reticularis (inner) Name the ste roid hormone s produce d by e ach of the following laye rs of the adre nal corte x: Zona glome rulosa Mineralocorticoids Zona fasciculata Glucocorticoids Zona re ticularis

Adrenal androgens What is the major pre cursor of all ste roid hormone s? Cholesterol What is the principal mine ralocorticoid? Aldosterone What is the principal glucocorticoid? Cortisol, but note that cortisol does have some mineralocorticoid activity as well. How do corticoste roids work bioche mically in the body? Bind to intracellular cytoplasmic receptors in target tissues then subsequently translocate to the nucleus where they act as transcription factors What is the pre cursor of adre nocorticotropic hormone (ACTH)? Proopiomelanocortin (POMC) What che micals are re le ase d whe n PO MC is cle ave d? ACT H; lipotropin; β-endorphin; metenkephalin; melanocyte-stimulating hormone (MSH) What drug is use d to diagnose adre nal insufficie ncy? ACT H (cosyntropin) A cosyntropin stimulation te st that re sults in no cortisol se cre tion characte riz e s which type of adre nal insufficie ncy? Primary adrenal insufficiency (defect at the level of the adrenal gland) A cosyntropin stimulation te st that re sults in re asonable cortisol se cre tion characte riz e s which type (s) of adre nal insufficie ncy? Secondary adrenal insufficiency (defect at the level of the pituitary gland) and possibly tertiary adrenal insufficiency (defect at the level of the hypothalamus) What is the primary me chanism by which corticoste roids incre ase the ne utrophil count? Demargination, where the polymorphonuclear cells are detached from the endovascular wall and are therefore available to be counted in the peripheral blood smear How is the production of arachidonic acid de cre ase d by glucocorticoids? Inhibition of phospholipase A2 How is Gushing syndrome diagnose d? Dexamethasone suppression test Give e xample s of short-acting glucocorticoids: Cortisone; hydrocortisone Give e xample s of inte rme diate -acting glucocorticoids: Methylprednisolone; prednisone; triamcinolone Give e xample s of long-acting glucocorticoids: Betamethasone; dexamethasone Give an e xample of a synthe tic mine ralocorticoid: Fludrocortisone How long afte r initiation of glucocorticoid the rapy doe s it take to suppre ss the hypothalamic-pituitary-adre nal (HPA) axis? 2 weeks How should long-te rm glucocorticoid the rapy be discontinue d? T aper regimen (taper regimen commonly used when discontinuing >2 weeks of glucocorticoid therapy) How do you pre ve nt suppre ssion of the HPA axis while using glucocorticoids? Alternate day dosing (every other day dosing) List the adve rse e ffe cts of glucocorticoids: Acne; insomnia; edema; hypertension; osteoporosis; cataracts; glaucoma; psychosis; increased appetite; hirsutism; hyperglycemia; muscle wasting; pancreatitis; striae; redistribution of body fat to abdomen, back, and face What drug inhibits glucocorticoid synthe sis by inhibiting 11-hydroxylase activity? Metyrapone What drug inhibits the conve rsion of chole ste rol to pre gne nolone ? Aminoglutethimide What antifungal can be use d to lowe r cortisol le ve ls in Gushing dise ase and may cause gyne comastia as an adve rse e ffe ct? Ketoconazole What diure tic blocks mine ralocorticoid re ce ptors and also inhibits the synthe sis of aldoste rone and androge ns (e g, te stoste rone )? Spironolactone What is the main side e ffe ct of spironolactone ? Gynecomastia; hyperkalemia How doe s spironolactone work as a diure tic? Antagonizes mineralocorticoid receptors in the kidney, thereby preventing sodium reabsorption in the distal tubules but not increasing potassium loss (therefore potassium-sparing) Using an angiote nsin-conve rting e nz yme (ACE) inhibitor in combination with spironolactone can cause what major e le ctrolyte abnormality? Hyperkalemia Name a potassium-sparing diure tic that doe s not have anti-androge n side e ffe cts: Eplerenone, which has increased specificity for the mineralocorticoid receptor when compared to spironolactone

ANDROGENS AND ANTI-ANDROGENS Is te stoste rone e ffe ctive whe n administe re d orally? No, as it is inactivated by first-pass metabolism. What e nz yme conve rts te stoste rone to dihydrote stoste rone (DHT)? 5-α-reductase Whe re is 5-α-re ductase found? Skin; epididymis; prostate; seminal vesicles What drug inhibits 5-α-re ductase ? Finasteride; dutasteride What are the two main the rape utic indications of finaste ride ? 1. Benign prostatic hyperplasia (BPH) 2. Male pattern baldness What are the use s of te stoste rone and its de rivative s (danaz ol; stanoz olol; nandrolone ; oxandrolone )? Male hypogonadism; increase muscle mass; increase RBCs; decrease nitrogen excretion; endometriosis (not first-line therapy for endometriosis given side effects) What are the adve rse e ffe cts of te stoste rone ?

Edema; premature closing of the epiphysis; increased aggression (“ road rage”); psychosis; increased low-density lipoprotein (LDL); decreased high-density lipoprotein (HDL); cholestatic jaundice; decreased spermatogenesis; gynecomastia; increased masculinization What antifungal drug inhibits the synthe sis of androge ns and is also use d as an antifungal? Ketoconazole What drugs act as androge n re ce ptor blocke rs and are use d in the tre atme nt of prostate cance r? Flutamide; bicalutamide Which drug act as androge n re ce ptor blocke r and is use d in the tre atme nt of stomach ulce rs? Cimetidine (a histamine H2-receptor blocker) What is the me chanism of action of le uprolide ? Gonadotropin-releasing hormone (GnRH) agonist (daily administration suppresses LH and FSH secretion, thereby inhibiting ovarian and testicular steroidogenesis) What are the the rape utic use s of le uprolide ? Prostate cancer; endometriosis

ES TROGENS AND ANTI-ES TROGENS —S ELECTIVE ES TROGEN RECEPTOR MODULATORS What is the major natural e stroge n? 17-α estradiol Whe re do conjugate d e quine e stroge ns (Pre marin) come from? Urine of pregnant mares What is Pre marin use d for? Vasomotor symptoms associated with menopause; vulvar and vaginal atrophy; abnormal uterine bleeding Name two synthe tic ste roidal e stroge ns: 1. Ethinyl estradiol 2. Mestranol Name one synthe tic nonste roidal e stroge n: Diethylstilbestrol (DBS) Me stranol is me taboliz e d to what che mical compound? Ethinyl estradiol What are the the rape utic use s of e stroge ns? Contraception; hormone replacement therapy (HRT ); female hypogonadism; dysmenorrhea; uterine bleeding; acne; osteoporosis How do e stroge ns affe ct se rum lipids? Increased triglycerides; increased HDL; decreased LDL How is e stroge n use ful in pre ve nting oste oporosis? Decreases bone resorption What are the adve rse e ffe cts of e stroge ns? Nausea; vomiting; headache; breast tenderness; endometrial hyperplasia; cholestasis; increased blood coagulation; increased endometrial cancer risk; increased breast cancer risk Estroge n use is contraindicate d in which se ttings? History of or current deep vein thrombosis (DVT ); history of or current pulmonary embolism (PE); active or recent stroke; active or recent myocardial infarction (MI); carcinoma of the breast; estrogen-dependent tumors; hepatic dysfunction; pregnancy How do e stroge ns incre ase blood coagulation? Decrease antithrombin III; increase clotting factors II, VII, IX, and X What can happe n to the fe male offspring of wome n who took DES during pre gnancy? Clear cell cervical or vaginal adenocarcinoma How do e stroge nic compounds work as contrace ptive s? Suppresses ovulation What doe s the e nz yme aromatase do? Converts testosterone to estradiol Name thre e aromatase inhibitors: 1. Anastrozole 2. Letrozole 3. Exemestane What are aromatase inhibitors use d for? Breast cancer What is clomiphe ne use d as? Fertility drug How doe s clomiphe ne work? Antiestrogen that induces ovulation by inhibiting negative feedback of estrogen on the hypothalamus and pituitary (this suppression leads to increasing release of LH and FSH) What are pote ntial “adve rse e ffe ct” of clomiphe ne ? Multiple births; hot flashes (10% patients); ovarian hyperstimulation (7% patients); reversible visual disturbances (2% patients) What doe s SERM stand for? Selective estrogen receptor modulator Name two SERMs: 1. Raloxifene 2. T amoxifen What is raloxife ne use d for? Prevention and treatment of osteoporosis in postmenopausal women How doe s raloxife ne work? Estrogen receptor agonist in bone; estrogen receptor antagonist in breast and uterus

What is tamoxife n use d for? Breast cancer How doe s tamoxife n work? Estrogen receptor agonist in bone; estrogen receptor antagonist in breast; estrogen receptor partial agonist in uterus Can tamoxife n incre ase e ndome trial cance r risk? Yes, it has stimulatory effects on endometrial tissue. Can raloxife ne incre ase e ndome trial cance r risk? No, it does not increase the risk of endometrial cancer because it has selective estrogen effects on the breast and bone but not the endometrium. What are the adve rse e ffe cts of tamoxife n? Endometrial hyperplasia; hot flashes; nausea; vomiting; vaginal bleeding; menstrual irregularities How doe s raloxife ne affe ct se rum lipids? Decreases total and LDL cholesterol; no effect on HDL or triglycerides What must you te ll a fe male patie nt who is using hormonal contrace ption whe n she is give n a pre scription for antibiotics? Antibiotics may decrease the effectiveness of OCPs; recommend the use of a “ backup” contraceptive method in addition to the OCPs throughout the duration of the antibiotic course. T his applies to both oral and intravenous antibiotics. Which type of O CPs are use d to shorte n or suppre ss me nstruations? Combination OCPs Which O CPs are use d to palliate the e ffe cts of polycystic ovary syndrome (PCO S)? Combination OCPs (often in combination with (oral) hypoglycemics)

PROGES TINS AND ANTIPROGES TINS What is the major natural proge stin? Progesterone What are the the rape utic use s of proge stins? Contraception; HRT (with estrogens); control of uterine bleeding; dysmenorrhea; suppression of postpartum lactation; endometriosis Which proge stins also posse ss androge nic activity? Norethindrone; norgestrel Why is proge ste rone adde d to a HRT re gime n in a fe male with an intact ute rus? Decrease risk of endometrial cancer by preventing unopposed action of estrogen. If the patient is status post hysterectomy then combination (estrogen + progesterone) therapy is unnecessary. How ofte n is de pot me droxyproge ste rone give n? Every 3 months (13 wk) How long doe s the le vonorge stre l contrace ptive subde rmal implant last? 5 years What are the adve rse e ffe cts of proge stins? Edema; depression; glucose intolerance; breakthrough bleeding; increased LDL; decreased HDL; hirsutism (androgenic progestins); acne (androgenic progestins) What drug acts as a proge ste rone antagonist and is use d in combination with prostaglandin Ej as an abortifacie nt? Mifepristone (RU 486) What are the adve rse e ffe cts of mife pristone ? Abdominal cramping; uterine bleeding; pelvic infection; ectopic pregnancy How do proge stins work as contrace ptive s? Prevents implantation of the early embryo into the endometrium; increases thickness of cervical mucus, thereby decreasing sperm access through the cervix

CLINICAL VIGNETTES A 64-ye ar-old man with a past me dical history of hype rte nsion, GERD, and type 2 diabe te s me llitus pre se nts to the e me rge ncy room with se ve re right uppe r quadrant pain. Acute chole cystitis is suspe cte d and an abdominal CT with contrast orde re d. Upon inspe ction of the patie nt’s me dication list, the atte nding physician orde rs one of the patie nt’s me dications to be he ld imme diate ly and not re sume d until at le ast 48 hours afte r the CT and ade quate re nal function has be e n asce rtaine d. What me dication was the patie nt taking that cause d this conce rn? T he patient is taking metformin, a biguanide, for his diabetes. Metformin in combination with iodinated contrast materials can lead to lactic acidosis and decreased renal function. T his is particularly concerning in patients who may already have some degree of renal impairment, such as diabetics. Additionally, preexisting renal impairment in combination with oral contrast can lead to increased serum levels of metformin, leading to toxicity. T herefore, it is imperative that the medication be stopped before use of the contrast medium and not restarted for 48 hours following the procedure. Proper kidney function needs to be documented as well before the medication is resumed. A 24-ye ar-old woman with no significant past me dical history is be ing e valuate d at a pre natal visit. He r husband accompanie s he r. The husband, who happe ns to be balding, quips with the doctor, “I know I look too old to be the daddy, but by the time the baby ge ts he re I’m going to look te n ye ars younge r. I’m taking this gre at ne w me dication that is making all my hair grow back.” The obste trician shows mild conce rn and asks e xactly what me dication the man is taking. What me dication use d to tre at male patte rn baldne ss must be avoide d around pre gnant wome n? Finasteride, a 5-α-reductase inhibitor has been shown to be effective to treat some cases of male pattern baldness. Different formulations of the drug exist in various doses. Some patients will buy the less expensive brand of the drug and split the tablets to get the appropriate dose. T his is a hazardous practice around pregnant females since even contact with crushed or broken tablets can lead to birth defects, notably abnormalities of the male external genitalia. In light of the potential adverse effects, the obstetrician could replace finasteride with topical minoxidil, which opens potassium channels and stimulates hair growth by an unknown mechanism. A 19-ye ar-old man pre se nts to his primary care physician with complaints of incre asing fatigue . He has occasional diz z ine ss and fe e ls we ak. He be lie ve s re ce ntly he has contracte d the flu be cause he has had nause a and vomiting, as we ll as muscle pain. He state s that he has to be care ful not to stand up too quickly, as this make s his diz z ine ss worse . In-office e valuation re ve als a thin adult male . O rthostatics are positive . The physician also note s a ge ne ral darke ning of the patie nt’s skin, e spe cially in the skin cre ase s. Ele ctrolyte te sts are orde re d. What othe r diagnostic te st would be appropriate at this time , and what would be the like ly re sults? T he hyperpigmentation here is a red flag for Addison disease, or primary adrenal insufficiency. Decreased output of cortisol from the adrenals leads to a compensatory increase in corticotrophin-releasing hormone (CRH) from the hypothalamus, leading to release of POMC, the precursor of ACT H, from the anterior pituitary. POMC is cleaved into ACT H and melanocyte-stimulating hormone, thus leading to increased skin pigmentation in patients suffering from Addison disease. Decreased levels of the stress hormone cortisol account for the patient’s other symptoms. To ascertain the primary nature of the disease, an ACT H (or cosyntropin) stimulation test should be ordered. A failure to produce an increase in serum levels of cortisol at either low or high doses of ACT H indicates failure at the level of the adrenal glands, or primary adrenal insufficiency. On physical examination, the hyperpigmentation of Addison disease may be distinguished from a suntan by examining areas unlikely to be exposed to sun, such as the axilla. Skin creases also tend to be darkly pigmented in Addison disease, as seen in this patient.

CHAPTER 10 Anti-inflammatory Agents and p-Aminophenol Derivatives (Acetaminophen) Eicosanoids (inflammatory me diators) are synthe siz e d from what che mical compound? Arachidonic acid Give two e xample s of e icosanoids: 1. Leukotrienes (LT s) 2. Prostaglandins (PGs) How is arachidonic acid forme d? Phospholipase A2 acting on cell membrane phospholipids Corticoste roids block what part of the inflammatory pathway? Inhibition of phospholipase A2 Angiote nsin and bradykinin have what e ffe ct on the inflammatory pathway? Stimulation of phospholipase A2 What e nz yme acts on arachidonic acid to form LTs? 5-Lipoxygenase Which LT is involve d in ne utrophil che motaxis? LT B4 Which LTs are involve d in anaphylaxis and bronchoconstriction? LT A4; LT C4; LT D4 What drug inhibits 5-lipoxyge nase , the re by inhibiting LT synthe sis? Zileuton What is z ile uton use d for? Asthma; allergies What two drugs act as LT re ce ptor antagonists and are use d in the tre atme nt of asthma and alle rgy? 1. Montelukast 2. Zafirlukast What are the adve rse e ffe cts of z ile uton and the LT re ce ptor antagonists? Increased liver function tests (LFT s); headache; Churg-Strauss syndrome 5-Lipoxyge nase is found in which ce ll type s? Neutrophils; eosinophils; basophils; mast cells Which LTs are conside re d the slow-re le asing substance s of anaphylaxis (SRS-A)? LT A4; LT C4; LT D4 What e nz yme acts on arachidonic acid to form PGs and thromboxane s (TXAs)? Cyclooxygenase (COX) Whe re is CO X 1 found? Platelets; gastrointestinal (GI) mucosa; vasculature Whe re is CO X 2 found? Sites of inflammation; brain; kidney; GI tract (low amounts vs COX 1) Is CO X 1 a constitutive or inducible e nz yme ? It is a constitutive enzyme, meaning that its concentration is not influenced by the concentration of substrate in the cell. Is CO X 2 a constitutive or inducible e nz yme ? It is an inducible enzyme, meaning that in resting conditions the enzyme is present in only trace quantities in the cell. Upon entry of the enzyme’s substrate, the concentration of the enzyme increases exponentially. Prostaglandin E1 (PGE1) doe s what to the following? Pate nt ductus arte riosus Maintains patency Ute rine smooth muscle Increases contraction; used as an abortifacient during pregnancy Blood ve sse ls Vasodilation Gastric mucosa Cytoprotective effect (inhibition of HC1 secretion and stimulation of mucus and bicarbonate secretion) What two PGF2α analogs promote bronchiolar and ute rine smooth muscle contraction? 1. Carboprost 2. Dinoprost Why are nonste roidal anti-inflammatory drugs (NSAIDs) e ffe ctive in the tre atme nt of dysme norrhe a? Inhibition of PGE2 and PGF2α synthesis What PGE1 analog is use d for impote nce due to its vasodilatory e ffe cts? Alprostadil What is anothe r name for PGI2? Prostacyclin What are the actions of prostacyclin? Inhibits platelet aggregation; vasodilation What is the name of a prostacyclin analog and what is it use d for? Epoprostenol; pulmonary hypertension (HT N) What are the actions of TXA2? Promotes platelet aggregation; bronchoconstriction; vasoconstriction

Incre asing cyclic ade nosine monophosphate (cAMP) will do what to plate le t aggre gation? Decrease platelet aggregation (mechanism of action of PGI 2) What is the me chanism of action of the nonse le ctive NSAIDs? Inhibit both COX 1 and COX 2, thereby inhibiting synthesis of PGs and T XAs Name the thre e CO X 2-spe cific inhibitors: 1. Celecoxib 2. Rofecoxib 3. Valdecoxib Do CO X 2 inhibitors inhibit plate le t aggre gation? No Which adve rse e ffe cts of ce le coxib have sparke d de bate s about whe the r it should be pulle d from the marke t or not? Increased risk of cardiovascular events (myocardial infarction, stroke, and worsening of preexisting HT N) What are the main the rape utic e ffe cts of NSAIDs? Anti-inflammatory; analgesic; antipyretic; antiplatelet What is the prototype NSAID? Acetylsalicylic acid What is ace tylsalicylic acid also known as? Aspirin; ASA Doe s ASA act as a re ve rsible or irre ve rsible inhibitor of CO X 1? Irreversible How doe s ASA irre ve rsibly inhibit CO X? Acetylates serine hydroxyl group near active site of COX, thereby forming an irreversible covalent bond What is the half-life of a plate le t? 5 to 7 days Why can’t plate le ts produce more CO X afte r ASA the rapy? Nonnucleated cells, therefore, lacking the capability of protein synthesis What laboratory te st is prolonge d afte r ASA the rapy? Bleeding time How doe s ASA work as an antipyre tic? Inhibits IL-1 stimulated synthesis of PGE2 in the hypothalamus, thereby inhibiting alteration of the temperature “ set-point” Low-dose ASA doe s what to uric acid e limination? Decreases tubular secretion (increases serum uric acid levels) High-dose ASA doe s what to uric acid e limination? Decreases tubular reabsorption (decreases serum uric acid levels) What type of acid-base disturbance is se e n in ASA ove rdose ? Mixed respiratory alkalosis with metabolic acidosis Doe s ASA ove rdose cause an anion gap or nonanion gap me tabolic acidosis? Anion gap metabolic acidosis What are the signs/symptoms of salicylism? Decreased hearing; tinnitus; vertigo; nausea; vomiting; headache; hyperventilation; confusion; dizziness Why is ASA not give n to childre n e spe cially during time s of viral (varice lla and influe nz a) infe ctions? Reye syndrome What characte riz e s Re ye syndrome ? Encephalopathy; hepatotoxicity How can e xcre tion of ASA from the urine be e xpe dite d? Alkalinization of urine with NaHCO3 What should be give n inste ad of ASA to childre n with fe ve r? Acetaminophen (or acetyl-para-aminophenol, APAP) What is the me chanism of action of APAP? T he direct fashion in which acetaminophen produces analgesia is unknown. T he drug inhibits synthesis of PGs (via COX 3 inhibition) in the central nervous system (CNS), the only place in the body COX 3 is found. It also blocks pain impulse generation peripherally. Antipyresis is achieved via inhibition of the hypothalamic heat regulating center. ASA can do what to asthmatics? Exacerbate symptoms via bronchoconstriction due to unopposed production of leukotrienes What is the “triad” of ASA hype rse nsitivity? 1. Asthma 2. Nasal polyps 3. Rhinitis What is the me chanism of ASA-induce d hype rthe rmia at toxic dose s? Uncoupling of oxidative phosphorylation What are the GI adve rse e ffe cts of NSAIDs? Ulcers; gastritis; GI bleeding (via decreased PGs which act as GI mucosal protectants); nausea; abdominal cramping If a patie nt is taking ASA and warfarin concomitantly, what should the dose of ASA be ? 81 mg daily ASA can do what to blood glucose ? Decrease blood glucose What type of kine tics doe s ASA follow? Zero order Antiplate le t and analge sic e ffe cts of ASA occur at lowe r or highe r dose s than those re quire d for anti-inflammatory e ffe cts? Lower What is the drug of choice for closing a pate nt ductus arte riosus? Indomethacin What is the me chanism of NSAID-induce d re nal failure ? Inhibition of PGE2 and PGI 2 synthesis which are responsible for maintaining renal blood flow Give e xample s of nonse le ctive NSAIDs othe r than ASA:

Ibuprofen; naproxen; diclofenac; indomethacin; ketorolac; piroxicam; oxaprozin; nabumetone; sulindac What are the major diffe re nce s be twe e n nonse le ctive NSAIDs and se le ctive CO X-2 inhibitors? COX-2 inhibitors have less antiplatelet action and less GI adverse effects. Which CO X-2 inhibitors are pote ntially cross-re active in patie nts with sulfonamide alle rgy? Celecoxib; valdecoxib Name two drugs that have antipyre tic and analge sic e ffe cts ye t lack anti-inflammatory and antiplate le t e ffe cts: 1. APAP 2. Phenacetin APAP inhibits CO X ce ntrally, pe riphe rally, or both? Centrally (inhibits PG synthesis in the CNS) O ve rdose of APAP can pote ntially cause what life -thre ate ning condition? Hepatic necrosis How is APAP pre dominantly me taboliz e d? Glucuronidation; sulfation Cytochrome P-450 2E1 me taboliz e s APAP to which compound (this is a minor me tabolic pathway)? N-acetyl-benzoquinoneimine (NAPQI) Which me tabolite of APAP is he patotoxic? NAPQI Which compound binds to NAPQ I and ultimate ly le ads to its e xcre tion? Glutathione What happe ns to patie nts taking APAP whe n glutathione store s run out? Accumulation of NAPQI with subsequent hepatotoxicity What drug is use d to re ple nish re duce d glutathione during time s of APAP ove rdose ? N-acetylcysteine What is the maximum daily dose of APAP in patie nts with normal he patic function? 4 g per 24 hours What is the maximum daily dose of APAP in patie nts with abnormal he patic function? 2 g per 24 hours

CLINICAL VIGNETTES A 16-ye ar-old adole sce nt is brought into the e me rge ncy room by his pare nts who say that he trie d to kill himse lf by taking all of the e xtra-stre ngth Tyle nol (ace taminophe n) in the ir me dicine cabine t 1 hour ago. Blood te sts are orde re d which re turn normal, including normal live r function te sts. Afte r 12 hours of obse rvation in the e me rge ncy de partme nt (ED), the pare nts insist on taking the ir son home saying that he is obviously fine since he ’s be e n asymptomatic for so long. The fathe r adds that the re we re not that many pills le ft in the bottle anyway, so the boy was just making a dramatic ge sture . Be side s e xplaining that the ir child ne e ds a full psychiatric e valuation to e nsure he is not a dange r to himse lf and othe rs, what othe r re asons should you e xplain to the pare nts re garding e xte nde d monitoring of the patie nt? Acetaminophen toxicity has four phases. In the earliest phase, lasting up to 24 hours, patients may be largely asymptomatic and serum transaminases only begin to rise gradually approximately 12 hours after the toxic dose is taken. T his is important to remember in patients who are exhibiting no signs of toxicity, even several hours after ingestion. As serum transaminases rise, right upper quadrant pain, nausea, vomiting, anorexia, jaundice, hepatic and renal failure ensue, ultimately with fatal results if the dose taken is high enough. T herefore, this patient needs to be closely monitored medically and N-acetylcysteine given to decrease mortality. T he maximum daily dose of acetaminophen is only 4 g. When you consider that each extra-strength Tylenol Gelcap is 500 mg, even ingestion of a nearly empty bottle can have devastating consequences. A 26-ye ar-old woman is brought to the e me rge ncy room in re spiratory distre ss. She is using acce ssory muscle s to bre ath and appe ars tire d. Auscultation of he r lungs re ve als whe e z e s bilate rally. He r husband re late s that she has had a minor viral illne ss for the past 2 days associate d with abdominal discomfort, vomiting, and diarrhe a. She took Pe pto-Bismol (bismuth subsalicylate ) this morning to he lp he r gastrointe stinal issue s. From what chronic me dical condition doe s this patie nt like ly suffe r, and by what me chanism did the Pe pto-Bismol cause he r curre nt symptoms? Pepto-Bismol contains a salicylate, the class of medication to which aspirin belongs. Salicylates can cause bronchoconstriction in a small number (3%-5%) of asthmatics due to preferential production of leukotrienes via lipoxygenase from arachidonic acid when the cyclooxygenase enzyme is inhibited by the salicylates. Leukotrienes then contribute to inflammation of the respiratory mucosa leading to edema and respiratory distress. T herefore, asthmatic patients should be made aware of this rare, but potentially lethal side effect of all salicylate containing medications. A te rm infant is born to a 29-ye ar-old mothe r who has diabe te s. De spite oxyge n the rapy, the infant de ve lops incre asing cyanosis in the hours following birth. The cyanosis is incre ase d during crying spe lls. Transposition of the gre at arte rie s (TGA) is diagnose d. What me dication should the infant be give n be fore de finitive surgical corre ction can take place ? T GA is more common in infants born to diabetic mothers. T he great vessels are reversed in these patients so that the aorta arises from the right ventricle and the pulmonary artery arises from the left ventricle. Oxygenated blood cannot reach the systemic circulation in this fashion. At this point, the infant is being kept alive by the still patent ductus arteriosus, a connection between the aorta and pulmonary artery that allows the pulmonary circulation to be bypassed in utero. T he ductus begins to close shortly after birth, normally within 12 to 24 hours. Closure will be fatal in this infant’s case. Patency is maintained by prostaglandins. T herefore, prostaglandin analogs such as alprostadil or misoprostol should be used while awaiting surgery.

CHAPTER 11 Miscellaneous Topics in Pharmacology IMMUNOS UPPRES S IVE AGENTS What are the main targe ts of immunosuppre ssive age nts? Inhibition of gene expression; inhibition of lymphocyte signaling and activation; depletion of proliferating lymphocytes; inhibition of cytokine action; depletion of specific immune cells; blockade of costimulation; blockade of cell adhesion and migration; inhibition of complement activation Which age nts are signaling inhibitors of B-/T-ce ll prolife ration? Cyclosporine A, tacrolimus, and sirolimus What is the me chanism of action of cyclosporine A? Complexes with cyclophilin to inhibit calcineurin, thereby decreasing interleukin (IL)-2, which is the main inducer for activation of T lymphocytes What are the main the rape utic indications of cyclosporine A? Prevention of organ rejection in heart, kidney, and liver transplants What are the adve rse e ffe cts of cyclosporine A? Nephrotoxicity; hepatotoxicity; cytomegalovirus (CMV) infections; hypertension; hyperkalemia; hirsutism; glucose intolerance; gingival hyperplasia; tremor; upper respiratory infections; anemia; leukopenia; thrombocytopenia Name thre e me dications that can cause gingival hype rplasia: 1. Cyclosporine 2. Phenytoin 3. Nifedipine What major he patic e nz yme me taboliz e s cyclosporine A? CYP 3A4 Cyclosporine A be come s more e ffe ctive as an immunosuppre ssant whe n combine d with what class of drugs? Glucocorticoids FK506 is also known as? T acrolimus What are the main the rape utic indications of tacrolimus? Prevention of organ rejection in liver and kidney transplants What is the me chanism of action of tacrolimus? It inhibits calcineurin by binding to FK binding protein (FKBP) rather than cyclophilin, cyclosporin’s target protein. It inhibits the signaling pathway coupling cell receptor activation of IL-2 gene transcription (IL-2 synthesis). What is the me chanism of action of sirolimus? It blocks IL-2 receptor signaling required for cell activation and proliferation. It forms a complex with the immunophilin FKBP, it does not affect calcineurin activity, but inhibits the kinase mT OR, which is necessary for cell growth and proliferation. What are the adve rse e ffe cts of tacrolimus? Nephrotoxicity; neurotoxicity; hypertension; headache; tremor; rash; hyperglycemia; hyperkalemia; anemia; leukocytosis; thrombocytopenia Which murine monoclonal antibody inte rfe re s with T-lymphocyte function by binding to CD3 glycoprote in? Muromonab-CD3 (OKT 3) Give e xample s of cytotoxic age nts use d in immunosuppre ssive the rapy: Azathioprine, mycophenolate mofetil, methotrexate, leflunomide, hydroxychloroquine, cyclophosphamide Which cytotoxic age nts use d in immunosuppre ssive the rapy are pro-drugs? Azathioprine, mycophenolate mofetil, leflunomide Which cytotoxic immunosuppre ssant is an alkylating age nt? Cyclophosphamide Which immunosuppre ssant is a de rivative of 6-me rcaptopurine , antagoniz e s purine me tabolism, and is cytotoxic to lymphocyte s? Azathioprine Which immunosuppre ssant inhibits inosine monophosphate de hydroge nase , the re by inhibiting de novo guanosine nucle otide synthe sis? Mycophenolate mofetil Which immunosuppre ssive age nts are cytokine inhibitors? Etanercept, infliximab, adalimumab, certolizumab, thalidomide, anakinra Which cytokine inhibitor is not a tumor ne crosis alpha (TNF-α) inhibitor? Anakinra (It is an interleukin-1 [IL-1] receptor antagonist.) Give two e xample s of immunosuppre ssive age nts that inhibit costimulation: 1. Abatacept 2. Belatacept Give two e xample s of immunosuppre ssive age nts that block ce ll adhe sion: 1. Natalizumab 2. Efalizumab Give an e xample of an immunosuppre ssive age nt that blocks comple me nt action: Eculizumab

AGENTS FOR OBES ITY What anore xiant works by incre asing dopamine and nore pine phrine le ve ls in the brain? Phentermine What anore xiant works by blocking re uptake of se rotonin, nore pine phrine , and dopamine into pre synaptic ne rve te rminals in the brain? Sibutramine

What are the adve rse e ffe cts of phe nte rmine and sibutramine ? T achycardia; hypertension; headache; insomnia; dry mouth; constipation What drugs should be avoide d in patie nts taking sibutramine ? Monoamine oxidase inhibitors (MAOIs); selective serotonin reuptake inhibitors (SSRIs); dextromethorphan What popular name was give n to the combination re gime n fe nfluramine and phe nte rmine and why was it e ve ntually pulle d off of the marke t? Fen-phen (combination of fenfluramine and phentermine); pulmonary hypertension and heart valve abnormalities. Fenfluramine was removed from the markets in 1997 in the United States because of these side effects. What antiobe sity age nt inhibits gastric and pancre atic lipase , the re by inhibiting bre akdown of die tary fat? Orlistat What are the adve rse e ffe cts of orlistat? Oily spotting; abdominal discomfort; flatus with discharge; fatty stools; fecal urgency; increased defecation What vitamin supple me ntation is re comme nde d in patie nts taking orlistat? Fat-soluble vitamins What are the fat-soluble vitamins? A; D; E; K

AGENTS FOR OS TEOPOROS IS Give e xample s of me dications that be long to the bisphosphonate drug class: Alendronate; risedronate; etidronate; pamidronate; ibandronate What is the me chanism of action of the bisphosphonate s? Inhibition of osteoclast-mediated bone resorption Which bisphosphonate can only be administe re d intrave nously? Pamidronate What must patie nts be counse le d on re garding oral bisphosphonate the rapy? T ake medication with a full glass of water and stay in a sitting or standing position for 30 minutes following administration to prevent esophageal irritation. Intrave nous pamidronate can also be use d to tre at what conditions? Hypercalcemia of malignancy; osteolytic bone lesions associated with multiple myeloma or metastatic breast cancer What se le ctive e stroge n re ce ptor modulator can incre ase bone de nsity in wome n without incre asing the risk for e ndome trial cance r? Raloxifene How is calcitonin administe re d? Intranasally What is the source of me dically use d intranasal calcitonin? Salmon How much calcium should a postme nopausal woman take daily? Calcium should be take n with what othe r supple me nt to e nhance its absorption? Vitamin D What re combinant N-te rminal 34-amino-acid se que nce of parathyroid hormone stimulate s oste oblast function, incre ase s gastrointe stinal (GI) calcium absorption, and incre ase s re nal tubular re absorption of calcium? T eriparatide

AGENTS FOR ERECTILE DYS FUNCTION What prostaglandin ET(PGE1) analog is inje cte d into the corpus cave rnosum and cause s incre ase d arte rial inflow and de cre ase d ve nous outflow to and from the pe nis, re spe ctive ly? Alprostadil What is the me chanism of action of silde nafil? Enhances the vasodilatory effect of nitric oxide (NO) by inhibiting phosphodiesterase type 5 (PDE-5) which allows for increased cyclic guanosine monophosphate (cGMP) levels (cGMP causes smooth muscle relaxation in the corpus cavernosum) Name two othe r PDE-5 inhibitors use d in the tre atme nt of e re ctile dysfunction: 1. 2. 3. 4.

T adalafil Vardenafil Udenafil Avanafil

What are the adve rse e ffe cts of silde nafil? Headache; dizziness; color vision disturbances; hypotension Silde nafil is contraindicate d in patie nts taking what type of me dications? Nitrates (potentiates hypotension)

AGENTS FOR RHEUMATOID ARTHRITIS For the following dise ase -modifying antirhe umatic drugs (DMARDs), give the me chanism of action: Glucocorticoids Decrease production of inflammatory mediators; suppression of neutrophil migration Etane rce pt Recombinant tumor necrosis factor (T NF) receptor linked to the Fc portion of human IgG1; binds T NF and inhibits its interaction with cell surface receptors Infliximab Chimeric monoclonal antibody that binds to human T NF-α, thereby interfering with its activity Le flunomide Inhibits dihydro-orotic acid dehydrogenase, thereby inhibiting pyrimidine synthesis Anakinra IL-1 receptor antagonist Gold salts Inhibit phagocytosis and lysosomal enzyme activity of macrophages Me thotre xate Inhibits dihydrofolate reductase; increases adenosine levels (this is methotrexate’s anti-inflammatory mechanism of action)

Hydroxychloroquine Interferes with lysosomal function; inhibits chemotaxis of neutrophils and eosinophils Pe nicillamine Depresses circulating rheumatoid factor (RF); depresses T -lymphocyte activity What is RF? IgM directed against IgG What othe r dise ase is pe nicillamine use d to tre at? Wilson disease What are the adve rse re actions for the following DMARDs? Glucocorticoids Acne; insomnia; edema; hypertension; osteoporosis; cataracts; glaucoma; psychosis; increased appetite; hirsutism; hyperglycemia; muscle wasting; pancreatitis; striae; redistribution of body fat to abdomen, back, and face Etane rce pt Hypersensitivity; headache; local injection site reactions; respiratory tract infection; positive antinuclear antibody (ANA); activation of latent tuberculosis Infliximab Headache; rash; nausea; diarrhea; urinary tract infection; infusion reactions; arthralgia; upper respiratory infection; activation of latent tuberculosis Le flunomide Pregnancy category X; hepatotoxicity; rash; alopecia Anakinra Local injection site reactions; headache; infection Gold salts Nephrotoxicity; dermatitis; alopecia; eosinophilia; leukopenia; thrombocytopenia; hematuria; nausea; vomiting Me thotre xate Mucositis; nausea; vomiting; diarrhea; nephrotoxicity; leukopenia; thrombocytopenia; hepatotoxicity; pneumonitis; crystalluria Hydroxychloroquine Cardiomyopathy; alopecia; visual disturbances; anorexia; nausea; vomiting; diarrhea; aplastic anemia; agranulocytosis; hemolysis in G6PD deficiency; cinchonism; exacerbation of porphyria Pe nicillamine Vasculitis; alopecia; hypoglycemia; thyroiditis; eosinophilia; hemolytic anemia; thrombotic thrombocytopenic purpura (T T P); hepatotoxicity; proteinuria

AGENTS FOR GOUT What two cate gorie s do gout patie nts fall into pathophysiologically? 1. Overproducers 2. Underexcretors Which drug is a xanthine oxidase inhibitor? Allopurinol What bioche mical proce ss is the e nz yme xanthine oxidase involve d in? Purine metabolism Which two drugs re quire dosage re ductions whe n give n concomitantly with allopurinol (be cause the y are me taboliz e d by xanthine oxidase )? 1. 6-Mercaptopurine 2. Azathioprine (reduce to 25% of normal dose) What is the most common side e ffe ct of allopurinol? Skin rash Name othe r pote ntial side e ffe cts of allopurinol: Nausea; vomiting; renal impairment; acute tubular necrosis; agranulocytosis; Stevens-Johnson syndrome Name thre e uricosuric age nts: 1. Probenecid 2. Sulfinpyrazone 3. High-dose aspirin De fine uricosuric: Enhancing renal excretion of uric acid What is the me chanism of action of probe ne cid? Inhibits proximal tubular resorption of uric acid Probe ne cid inhibits the tubular se cre tion of what antibiotic and is some time s give n in combination to prolong its half-life ? Penicillin How is acute gout tre ate d? Colchicine; nonsteroidal anti-inflammatory drugs (NSAIDs) Why is allopurinol not use d in the tre atme nt of an acute gout attack? May actually precipitate acute gouty arthritis and therefore perpetuate the acute gout attack (used for prevention and not treatment of gout attacks) What is the me chanism of action of colchicine ? Increases depolymerization of microtubules; decreases leukocyte motility; decreases phagocytosis in joints and lactic acid production, thereby reducing deposition of urate crystals What are the adve rse e ffe cts of colchicine ? Nausea; vomiting; diarrhea; abdominal pain; agranulocytosis; aplastic anemia; bone marrow suppression; alopecia; myopathy; arrhythmia; hepatotoxicity

RETINOIDS Re tinoic acid share s a similar structure and function with what fat-soluble vitamin? Vitamin A 13-cis-Re tinoic acid is also known as what drug?

Isotretinoin (Accutane) What is the major the rape utic indication of isotre tinoin? Acne Isotre tinoin be longs in what pre gnancy risk factor cate gory? Pregnancy category X. T his drug should not be used in pregnant patients under any circumstances. Fetal isotretinoin syndrome involves fetal craniofacial, cardiac, and CNS defects. What is the me chanism of action of isotre tinoin? Decreases sebaceous gland size and reduces sebum production; regulates cell proliferation and differentiation; decreases hyperkeratinization; decreases androgen levels; decreases Propionibacterium acnes levels (bacterium associated with acne) Hype rvitaminosis A can damage what major organ? Liver All-trans-re tinoic acid (ATRA) is also known as what drug? T retinoin What is the major the rape utic indication of ATRA? Induction of remission in patients with acute promyelocytic leukemia (APL) What is the me chanism of action of tre tinoin (whe n use d for acne )? Binds to nuclear receptors and inhibits clonal proliferation and granulocyte differentiation

HERBAL MEDICATIONS Doe s the Unite d State s Food and Drug Administration (FDA) re gulate he rbal me dications? No Are he rbal me dications conside re d “drugs” by FDA standards? No, considered “ nutritional supplements” What he rbal me dication is use d for migraine and fe ve r? Feverfew What he rbal me dication is use d for “je t lag”? Melatonin What he rbal me dication is use d for de pre ssion? St. John’s wort What he rbal me dication is use d for be nign prostate hype rplasia (BPH)? Saw palmetto What he rbal me dication is use d for anxie ty? Kava kava What he rbal me dication is use d for Alz he ime r dise ase ? Gingko biloba What he rbal me dication is use d for hype rchole ste role mia? Garlic What he rbal me dication is use d for the common cold? Echinacea What he rbal me dication is use d for he patitis? Milk thistle What he rbal me dication is use d to tre at hot flushe s in me nopause ? Black cohosh Is St. John’s wort a me tabolic e nz yme induce r or inhibitor? Inducer What he rbal supple me nts should one use with caution whe n using warfarin concomitantly? T he G4 supplements: garlic, ginger, gingko, and ginseng. T hese substances may interact with warfarin, and some, especially gingko which has antiplatelet effects, may increase bleeding risk. Ginseng may decrease warfarin’s effects.

S UBS TANCE ABUS E AND TOLERANCE What are the physiologic e ffe cts of he roin? Euphoria; decreased motor function; respiratory depression; miosis What are some of the signs/symptoms of he roin withdrawal? Nausea; vomiting; muscle aches; yawning; lacrimation; rhinorrhea; diarrhea; sweating; fever; mydriasis; piloerection; insomnia What is anothe r name for piloe re ction? Goose bumps How can he roin ove rdose le ad to de ath? Acute respiratory depression What drug is use d for the tre atme nt of he roin abuse ? Methadone (jo,-receptor agonist) What drug is use d to tre at re spiratory de pre ssion during an opioid ove rdose ? Naloxone What drug is use d to counte ract the sympathe tic e ffe cts of he roin withdrawal? Clonidine What is the me chanism of action of cocaine ? Blocks reuptake of dopamine, serotonin, and norepinephrine What is the me chanism of action of amphe tamine s? Ultimately, they increase the release of catecholamines from presynaptic nerve endings. What are the physiologic e ffe cts of cocaine and amphe tamine s? Euphoria; reduced inhibitions; reduced sleep; reduced appetite; tachycardia; sweating; increased alertness; pupillary dilation What are the withdrawal signs/symptoms of cocaine and amphe tamine s? Depression; increased sleep; increased drug cravings; bradycardia; dysphoria What are the physiologic e ffe cts of marijuana? Analgesia; increased appetite; impairment of short-term memory; antiemetic; altered perception of time and space; change in motor and postural control What is the active psychotropic compone nt of marijuana? Delta-9-tetrahydrocannabinol (Δ9-T HC) What me dication is a synthe tic form of THC formulate d in se same oil and is use d as an appe tite stimulant in AIDS patie nts and as an antie me tic during che mothe rapy in cance r patie nts?

Dronabinol What are the signs/symptoms of marijuana withdrawal? Restlessness; irritability; agitation; insomnia; nausea Is ove rdose of marijuana fatal? No Give e xample s of hallucinoge nic drugs: Methylenedioxymethamphetamine (MDMA; has hallucinogenic properties, yet is usually classified as a stimulant); mescaline; psilocybin; lysergic acid diethylamide (LSD); phencyclidine (PCP) MDMA is also known as? Ecstasy Ecstasy can be ne urotoxic to which type of ne urons? Serotonergic neurons What is a major side e ffe ct of LSD? Flashbacks Is ove rdose of LSD fatal? No Doe s LSD have re inforcing e ffe cts? No What drug class doe s PCP and ke tamine be long to? Dissociative anesthetics Is ove rdose of PCP fatal? Yes, it is potentially fatal. Doe s PCP have re inforcing e ffe cts? Yes What type s of ocular disturbance s are se e n in PCP intoxication? Vertical and horizontal nystagmus

TOXICOLOGY Name the antidote for e ach type of poisoning: Atropine Acetylcholinesterase inhibitors Arse nic, gold Dimercaprol Le ad, me rcury Dimercaprol; succimer; penicillamine Ace tylcholine ste rase inhibitors Atropine with pralidoxime Ace taminophe n N-acetylcysteine Be nz odiaz e pine s Flumazenil β-Blocke rs Glucagon Coppe r Penicillamine Carbon monoxide Oxygen (hyperbaric) Digoxin Digoxin immune Fab He parin Protamine Iron Deferoxamine; deferasirox The ophylline β-Blockers Warfarin Vitamin K; fresh frozen plasma (FFP) Thrombolytics Aminocaproic acid; tranexamic acid O pioids Naloxone What are the signs and symptoms of arse nic poisoning? “ Rice water” stools; GI discomfort; seizures; pallor; skin pigmentation; alopecia; bone marrow suppression; stocking glove neuropathy What are the signs and symptoms of iron poisoning? Bloody diarrhea; shock; coma; dyspnea; necrotizing gastroenteritis; hematemesis What are the signs and symptoms of le ad poisoning? Nausea; vomiting; diarrhea; tinnitus; encephalopathy; anemia; neuropathy; nephropathy; infertility; hepatitis What are the signs and symptoms of me rcury poisoning? Ataxia; auditory loss; visual loss; chest pain; pneumonitis; nausea; vomiting; renal failure; shock What are the signs and symptoms of tricyclic antide pre ssant (TCA) poisoning? Hyperthermia; coma; convulsions; cardiotoxicity; mydriasis; constipation; prolonged QT interval What are the signs and symptoms of SSRI poisoning (must be whe n use d concomitantly with othe r se rotone rgic age nts such as MAO Is or TCAs)? T achycardia; hypertension; seizures; hyperthermia; agitation; muscle rigidity; hallucinations

CLINICAL VIGNETTES A 66-ye ar-old man with a past me dical history of stable angina we ll controlle d with oral nitroglyce rin come s in for a re gular che ck up to his primary care physician’s office . Afte r you polite ly ask about his wife he re plie s, “We ll, she ’s not too happy with me late ly.” Afte r some ge ntle prodding you e licit a history of e re ctile dysfunction. He asks you about a me dication he saw on TV, Viagra (silde nafil). You inform him that be cause he take s nitrate s he cannot also take this me dication due to pote ntially fatal inte ractions. What alte rnative me dication might you sugge st for this patie nt?

T he phosphodiesterase inhibiting properties of sildenafil cause vasodilation, allowing increased blood flow to maintain an erection. However, when used concomitantly with another vasodilating drug such as nitroglycerin, blood pressures may fall to levels insufficient to perfuse vital organs, especially in someone with preexisting heart disease. T herefore, an alternative agent must be used to treat this man’s erectile dysfunction. Alprostadil is a less popular medication than sildenafil since it must be injected directly into the corpus cavernosa, but it does not have the systemic effects seen with sildenafil, making it a viable alternative to treat this patient’s condition. A 74-ye ar-old woman unde rgoe s a bone scan to e valuate he r bone de nsity. He r T-score come s back at –2.7. What is the me chanism of the class of me dications that are first-line the rapy for this woman’s me dical condition? T his patient has osteoporosis, defined as a T-score of less than -2.5. Studies have found the most benefit in early preventative treatment in women with the highest risk for fracture, that is, those with lower (more negative) T-scores. Bisphosphonates are generally first-line therapy for osteoporosis. Bisphosphonates work by binding to hydroxyapatite crystal in bone and inhibiting osteoclast-mediated bone resorption. A 53-ye ar-old man pre se nts to your office with a warm, swolle n right me tacarpal phalange al joint. He note s that this pain be gan sudde nly, and is e xquisite ly painful. What are your options for imme diate tre atme nt for this man’s pain, as we ll as long-te rm manage me nt for his condition? T his is a classic presentation of gout, seen most commonly in older men. It is caused by an accumulation of uric acid crystals in a joint or tendon. Acute management involves NSAIDs, colchicine, or steroids (local or systemic). Long-term management includes life-style modifications to decrease purine breakdown such as decreased intake of red meat and alcohol. Allopurinol, a xanthine oxidase inhibitor, may be helpful as well.

APPENDIX Common Drug Suffixes/Prefixes

Suggested Readings DiPiro JT , T albert RL, Yee GC, et al. Pharmacotherapy: A Pathophysiologic Approach. 5th ed. New York, NY: McGraw-Hill; 2002. Gilbert DN, Moellering RC Jr, Eliopoulos GM, et al. The Sanford Guide to Antimicrobial Therapy. 36th ed. Sperryville, VA: Antimicrobial T herapy, Inc.; 2006. Katzung BG, ed. Basic and Clinical Pharmacology, 11th ed. New York, NY: Lange Medical Books; 2004. Koda-Kimble MA, Young LY, Kradjan WA, et al. Applied Therapeutics: The Clinical Use of Drugs. 7th ed. Philadelphia, PA: Lippincott Williams & Wilkins; 2001. Physicians’ Desk Reference. 59th ed. Montvale, NJ: T homson PDR; 2004.

Index A Abacavir Abatacept Abciximab Absence seizures Acarbose ACE. See Angiotensin-converting enzyme (ACE) Acebutolol ACEIs. See Angiotensin-converting enzyme inhibitors (ACEIs) Acetaldehyde Acetaldehyde dehydrogenase Acetaminophen poisoning by Acetazolamide Acetoacetic acid Acetone N-Acetyl-benzoquinoneimine (NAPQI) Acetylcholine (ACh) Acetylcholinesterase (AChE) Acetylcholinesterase inhibitors (AChEIs) poisoning by N-Acetylcysteine Acetyl-para-aminophenol (APAP) Acetylsalicylic acid ACh. See Acetylcholine (ACh) AChE. See Acetylcholinesterase (AChE) AChEIs. See Acetylcholinesterase inhibitors (AChEIs) Acid(s), weak Acidification, of urine ACLS. See Advanced cardiac life support (ACLS) Acne Acrolein ACT (artemisinin combination therapies) ACT H. See Adrenocorticotropic hormone (ACT H) Active transport Acute-angle-closure glaucoma Acyclovir Adalimumab Adenosine Adrenal androgens Adrenal cortex, zones of Adrenal insufficiency Adrenal steroids Adrenergic agents α-Adrenergic agonists β2-Adrenergic agonists in asthma α-Adrenergic antagonists β-Adrenergic blockers β-Adrenergic receptor blockade Adrenergic receptors β-Adrenoceptor agonists Adrenocorticotropic hormone (ACT H) Adsorbents Advanced cardiac life support (ACLS) Affinity of agonist African trypanosomiasis Afterload Agonist(s) β2 affinity of defined full maximal response of partial Agranulocytosis Akathisia Albendazole Albumin Albuterol Alcohol Alcohol use, warfarin and

Alcohol withdrawal Aldosterone Alendronate Alkalinization, of urine Alkaloids ergot vinca Alkalosis, metabolic Alkylating agents Allergic rhinitis, agents for Allopurinol All-trans-retinoic acid (AT RA) Almotriptan Aloe Alopecia Alprazolam Alprostadil Alteplase Alzheimer disease, herbal medication for Alzheimer-type dementia Amantadine Amebiasis American trypanosomiasis Amidases Amide local anesthetics Amifostine Amiloride Aminocaproic acid Aminoglutethimide Aminoglycoside(s) penicillins and Aminoglycoside antibiotics P-Aminophenol derivatives Aminophylline Aminosalicylates Amiodarone Amitriptyline Amlodipine Ammonium chloride Amobarbital Amodiaquine Amphetamine(s) Amphotericin B Amrinone Amylin analogs Anakinra adverse reactions to for rheumatoid arthritis Analgesics, opioid Anaphylaxis Anastrozole Androgens adrenal Anesthesia dissociative stages Anesthetics general inhaled IV local Angina pectoris, agents for Angiotensin Angiotensin II receptor blockers (ARBs) Angiotensin-converting enzyme (ACE) Angiotensin-converting enzyme inhibitors (ACEIs) Anistreplase Anorexiants Antacids calcium-containing Antagonism chemical defined pharmacologic physiologic Antagonist(s) competitive defined noncompetitive

α1-Antagonists α2-Antagonists β-Antagonists (α1/β-Antagonists, mixed Anthracycline antitumor antibiotics Anti-androgens Antiarrhythmic agents Antibacterial agents Antibiotics. See also specific types and drugs aminoglycoside antibacterial agents antifungal agents antihelminthic agents antiprotozoal agents antituberculosis antitumor antiviral agents bacitracin bacteriostatic fluoroquinolone glycylcycline α-lactam lipopeptide, cyclic macrolide oxazolidinone penicillins quinolone streptogramin sulfonamide allergy to tetracycline vancomycin Anticholinergics inhaled Anticoagulants Anticonvulsants Antidepressants tricyclic poisoning by Anti-estrogens Antifungal agents Antihelminthic agents Antihistamine-anticholinergics Antihistamines in allergic rhinitis for nausea and vomiting nonsedating sedating Antihyperlipidemic agents Antihypertensive agents Anti-inflammatory drugs nonsteroidal for gout Antimetabolite chemotherapeutic agents Antimicrobial agents. See also specific drugs and Antibiotics Antimotility agents Antiprogestins Antiprotozoal agents Antipsychotic agents “ atypical” “ typical” Antipyretics Antisecretory agents Antituberculosis medications Antitumor antibiotics Antitussive agents Antiviral agents Anxiety, herbal medication for Anxiolytic agents APAP. See Acetyl-para-aminophenol (APAP) Aprepitant Arachidonic acid ARBs. See Angiotensin II receptor blockers (ARBs) Arginine Aripiprazole Aromatase Aromatase inhibitors Arrhythmia(s), cardiac Arsenic poisoning

Artemisinin Artemisinin combination therapies (ACT ) Arthritis, rheumatoid, agents for Articaine ASA. See Aspirin (ASA) L-Asparaginase Aspart insulin Aspirin (ASA) high-dose hypersensitivity to “ Aspirin triad” Asthma drugs for Atazanavir Atenolol Atherosclerosis Atorvastatin AT RA. See All-trans-retinoic acid (AT RA) Atracurium Atropine poisoning by with pralidoxime Attapulgite Autonomic agents Autonomic nervous system, subdivisions of Avanafil Azathioprine Azithromycin Azole antifungals AZT (zidovudine) Aztreonam

B Bacitracin Baclofen Bacteriostatic antibiotics Barbiturate(s) Base(s), weak Beclomethasone Belatacept Belladonna Benign prostatic hyperplasia (BPH) herbal medication for Benzamides, substituted, for nausea and vomiting Benzodiazepine(s) for manic-depression poisoning by for status epilepticus CNS depression due to long-acting for nausea and vomiting Benztropine Betamethasone Bethanechol Bicalutamide Biguanides Bile acid sequestrants Bioavailability Bioequivalence Bipolar disorder Bisacodyl Bismuth subsalicylate Bisoprolol Bisphosphonates Black cohosh Bladder detrusor muscle tone, ACh effects on Bladder sphincter tone, ACh effects on Bleomycin β-Blocker(s) adverse effects of in anginal pectoris cardiac action potential phases and in CHF exercise tolerance effects of in hypertension membrane stabilizing effects of as poisoning antidote

poisoning by prototype Blood pressure (BP) ACh effects on increase in normal Blood pressure (BP) equation Blood volume Blood/gas partition coefficient Botulinum toxin BP. See Blood pressure (BP) BPH. See Benign prostatic hyperplasia (BPH) Bradykinesia Bradykinin Bran Bromocriptine Bronchial smooth muscle, relaxation of Bronchiolar secretions, increased Bronchiolar spasms Bronchodilation, ACh effects on Bronchodilators B-/T -cell proliferation Budesonide Bulk forming agents Bumetanide Buprenorphine Bupropion Buspirone Busulfan Butyrophenones

C Cl− channel activators Ca++ channel blockers Caffeine, ergotamine with CAIs. See Carbonic anhydrase inhibitors (CAIs) Calcitonin, intranasal Calcium Calcium channel blockers (CCBs) Calcium ion Calcium-containing antacids cAMP. See Cyclic adenosine monophosphate (cAMP) Candesartan Cannabinoids, for nausea and vomiting Carbachol Carbamazepine Carbapenem β-lactams Carbidopa Carbon monoxide poisoning Carbonic anhydrase inhibitors (CAIs) Carboprost Carcinoid syndrome Cardiac action potential, for fast-response fibers, phases Cardiac Arrhythmia Suppression T rial (CAST ) Cardiac arrhythmias Cardiac output (CO) increase in Cardiac output equation Carvedilol Cascara Caspofungin CAST . See Cardiac Arrhythmia Suppression T rial (CAST ) CAT . See Choline acetyltransferase (CAT ) Catechol-O-methyltransferase (COMT ) Catechol-O-methyltransferase (COMT ) inhibitors CCBs. See Calcium channel blockers (CCBs) CCS. See Cell-cycle specific (CCS) Cefepime Celecoxib Cell cycle, phases of Cell-cycle specific (CCS) cancer chemotherapeutic agent Central nervous system (CNS) agents Cephalosporins Cestodes Chagas disease Chemical antagonism Chemoreceptor trigger zone (CT Z)

Chemotherapeutic agents antimetabolite CCS pharmacology of CHF. See Congestive heart failure (CHF) Chloramphenicol Chlordiazepoxide Chloroquine Chlorothiazide Chlorpheniramine Chlorpromazine Chlorpropamide Chlorthalidone Cholesterol Cholestyramine Choline acetyltransferase (CAT ) Cholinergic agents “ Christmas factor” Chronic kidney disease, blood pressure with Chronic obstructive pulmonary disease (COPD) Chronotropes Cidofovir Cilastatin Cimetidine Cinchonism Cisapride Cisplatin Citalopram Clarithromycin Classic angina Clearance (Cl) Clindamycin Clofibrate Clomiphene Clomipramine Clonazepam Clonidine Clopidogrel Clostridium botulinum Clotting cascade Clotting factors Clozapine CNS agents CNS depression, benzodiazepine-induced CO. See Cardiac output (CO) Cocaine Codeine Colchicine Cold(s), herbal medication for Colitis, ulcerative Common cold, herbal medication for Competitive antagonist COMT . See Catechol-O-methyltransferase (COMT ) Congestion, nasal Congestive heart failure (CHF) agents for Constipation opioid-induced Contractility COPD. See Chronic obstructive pulmonary disease (COPD) Copper poisoning Corrected QT interval (QT c) Corticosteroid(s) in asthma biochemical actions of in IBD inhaled in asthma in COPD intranasal, in allergic rhinitis for nausea and vomiting Cortisol Cortisone Cosyntropin Cosyntropin stimulation test Cough expectorant COX. See Cyclooxygenase (COX) Crohn disease Cromolyn

intranasal Css CT Z. See Chemoreceptor trigger zone (CT Z) Cushing disease Cushing syndrome Cyanide poisoning Cyclic adenosine monophosphate (cAMP) Cyclizine Cyclooxygenase (COX) Cyclooxygenase (COX) inhibitors Cyclophosphamide Cyclosporine A CYP 1A2 CYP 2C9 CYP 2C19 CYP 2D6 CYP 2E1 CYP 3A4 Cystic fibrosis, agents for Cytarabine Cytochrome P-450 enzymes drugs metabolized by hepatic induction of inhibition of location metabolism of Cytokine inhibitors

D DA. See Dopamine (DA) Dactinomycin Dantrolene Dantrolene + bromocriptine Daptomycin “ Date rape” drug Daunorubicin Dawn phenomenon DDC. See Dopa decarboxylase (DDC) “ Deadly nightshade” Decongestants intranasal nasal Deferasirox Deferoxamine 5’-Deiodinase Delavirdine Demargination Demeclocycline Dementia, Alzheimer-type Depot medroxyprogesterone Depression herbal medication for respiratory during opioid overdose opioid-induced Dermatophytes Dermatophytosis, of scalp DES. See Diethylstilbestrol (DES) Desflurane Desipramine Desloratadine Desmopressin Dexamethasone Dexamethasone suppression test Dexmedetomidine Dexrazoxane Dextromethorphan DHFR (dihydrofolate reductase) DHP. See Dihydropyridines (DHP) DHT . See Dihydrotestosterone (DHT ) Diabetes insipidus agents for nephrogenic neurogenic types of Diabetes mellitus (DM)

agents for blood pressure with Diabetic ketoacidosis Diaphoresis Diarrhea, agents for Diazepam Diazoxide Didanosine Diethylstilbestrol (DES) Diffusion facilitated passive Digitalis glycosides Digitoxin Digoxin poisoning by Digoxin immune Fab Dihydrofolate reductase (DHFR) Dihydropyridines (DHP) Dihydrotestosterone (DHT ) Diloxanide Diltiazem Dimercaprol Dinoprost Diphenhydramine for allergic rhinitis for antipsychotic-induced EPS metoclopramide with for nausea and vomiting for sedation Diphenoxylate Dipyridamole Disease modifying antirheumatic drugs (DMARDs) Disopyramide Dissociative anesthesia Disulfiram Disulfiram-like reactions Diuretics in CHF hypertensive agents and loop potassium-sparing thiazide DM. See Diabetes mellitus (DM) DMARDs. See Disease modifying antirheumatic drugs (DMARDs) DNase Dobutamine Docusate Dofetilide Dolasetron Domperidone Dopa decarboxylase (DDC) Dopamine (DA) Dopamine (DA) agonists, nonergot Dorzolamide Dose-response curve graded quantal (cumulative) Doxazosin Doxorubicin Doxylamine Dromotropes Dronabinol Droperidol Drug(s). See also specific agents permeation across cellular membranes Drug development, clinical phases in Duloxetine Duodenal ulcer Dutasteride Dynorphin Dyskinesia(s), tardive

E Ebstein anomaly Echinacea Echothiophate Ecstasy

Eculizumab ECV (extracellular volume) ED50 EDRF. See Endothelial-derived relaxation factor (EDRF) Edrophonium Efalizumab Efavirenz Effective dose (ED50) Efficacy Eicosanoids Elimination first-order zero-order Emesis opioid-induced Emtricitabine Encainide Endocrine agents β-Endorphin Endothelial-derived relaxation factor (EDRF) Enflurane Enfuvirtide Enkephalin Enoxaparin Entacapone Enterobius vermicularis Enzyme(s), cytochrome P-450. See Cytochrome P-450 enzymes Ephedrine Epinephrine Epipodophyllotoxin cancer chemotherapeutic agents Eplerenone Eprosartan EPS. See Extrapyramidal symptoms (EPS) Eptifibatide Erectile dysfunction agents for Ergonovine Ergot alkaloids Ergotamine Ertapenem Erythromycin Escitalopram Esmolol Esomeprazole Essential hypertension Ester(s) Esterases 17-β-Estradiol Estrogens Eszopiclone Etanercept adverse reactions to for rheumatoid arthritis Ethacrynic acid Ethambutol Ethanol Ethinyl estradiol Ethosuximide Etidronate Etomidate Etoposide Exemestane Exenatide Extracellular volume (ECV) Extrapyramidal symptoms (EPS) Extreme physiologic stress Ezetimibe

F Facilitated diffusion Factor VII Famciclovir Famotidine Fanconi syndrome Fat-soluble vitamins Fatty acids, omega-3 FDA. See Food and Drug Administration (FDA)

Febrile seizures Felodipine Fenofibrate Fen-phen Fentanyl Fetal lung maturity Fever, herbal medication for Feverfew Fexofenadine FFP. See Fresh frozen plasma (FFP) Fibrates Fibrinogen Fibrosis(es) cystic, agents for pulmonary Finasteride First-order elimination First-order kinetics FK506 Flecainide Fluconazole Flucytosine Fludarabine Fludrocortisone Flukes Flumazenil Flunitrazepam Fluoroquinolone(s) 5-Fluorouracil Fluoxetine Fluphenazine Flurazepam Flutamide Fluticasone Fluvoxamine Fondaparinux Food and Drug Administration (FDA) Formoterol Fosamprenavir Fosaprepitant Foscarnet Fosphenytoin Fresh frozen plasma (FFP) “ Fruity breath” Full agonist Furosemide

G GABA. See Gamma-aminobutyric acid (GABA) Gabapentin Gamma-aminobutyric acid (GABA) Ganciclovir Ganglionic blocking agents Garlic Gastric ulcer Gastrin Gastroesophageal reflux disease (GERD), agents for Gastrointestinal agents adrenal steroids constipation diabetes insipidus diabetes mellitus diarrhea GERD IBD nausea and vomiting PUD thyroid disorders Gastrointestinal motility, ACh effects on Gastrointestinal secretions, ACh effects on Gemfibrozil General anesthetics Generalized anxiety disorder Generalized tonic-clonic seizures GERD. See Gastroesophageal reflux disease (GERD) GFR. See Glomerular filtration rate (GFR)

Gingival hyperplasia Gingko biloba Glargine insulin Glaucoma, acute-angle-closure Glimepiride Glipizide Glomerular filtration rate (GFR) Glucagon β(l-3)-d-Glucan Glucocorticoid(s) adverse reactions to arachidonic acid production decreased by cyclosporine A with in fetal lung maturation inhaled for rheumatoid arthritis thyroid-binding globulin levels decreased by from zona fasciculata α-Glucosidase inhibitors Glutathione Glyburide Glycerin suppository P-Glycoprotein Glycoprotein IIb/IIIa receptor Glycosides, digitalis Glycylcycline antibiotics GnRH agonist Gold poisoning Gold salts adverse reactions to for rheumatoid arthritis Gonadotropin-releasing hormone (GnRH) agonist Goose bumps Gout, agents for G-proteins, inhibitory Graded dose-response curve Grand mal seizures Granisetron Griseofulvin Growth fraction Guaifenesin Gynecomastia

H Half-life (t1/2) of platelets T3 T4 Hallucinogenic drugs Halofantrine Haloperidol Halothane HCT Z. See Hydrochlorothiazide (HCT Z) Headache(s), migraine drugs for herbal medication for Heart rate ACh effects on decrease in increase in Helicobacter pylori infection Hemophilia A Hemophilia B Heparin(s) low molecular weight poisoning by Hepatitis herbal medication for Herbal medications Heroin Heroin withdrawal Hexamethonium HGPRT (hypoxanthine-guanine phosphoribosyl transferase) Hiccups, intractable Histamine Homovanillic acid (HVA) Hot flushes, menopause-related, herbal medication for

HPA axis. See Hypothalamic-pituitary-adrenal (HPA) axis 5-HT . See Serotonin (5-HT ) HVA. See Homovanillic acid (HVA) Hydralazine Hydrochlorothiazide (HCT Z) Hydrocodone Hydrocortisone Hydromorphone β-Hydroxybutyric acid Hydroxychloroquine adverse reactions to for rheumatoid arthritis Hydroxyzine Hyperbilirubinemia Hypercholesterolemia, herbal medication for Hyperemesis gravidarum Hyperemia Hyperkalemia Hyperplasia, gingival Hypersensitivity ASA-induced Hypersensitivity reactions Hypertension essential stage I stage II Hypertensive crisis Hyperthermia ASA-induced malignant Hyperthyroidism Hypervitaminosis A Hypnotic agents Hypoglycemia Hypoglycemic agents, oral Hypokalemia Hypotension orthostatic postural Hypothalamic-pituitary-adrenal (HPA) axis Hypothyroidism Hypoxanthine-guanine phosphoribosyl transferase (HGPRT )

I IBD. See Inflammatory bowel disease (IBD) Ibutilide Idarubicin Ifosfamide Imipenem Imipramine Immunosuppressive agents Incretins IND. See Investigational new drug (IND) Indinavir Indomethacin INF-α Infections. See specific types, e.g., MRSA infections Inflammatory bowel disease (IBD), agents for Infliximab adverse reactions to for Crohn disease for rheumatoid arthritis Influenza A Infusion rate (k0) INH. See Isoniazid (INH) Inhaled anesthetics Inhaled anticholinergics Inhaled corticosteroids in asthma in COPD Inhaled glucocorticoids Inhibitory G-proteins Inotropes INR, of patient stabilized on warfarin, azole antifungal effects on Insulin types of Interferon necrosis factor-α (INF-α) International normalized ratio (INR), of patient stabilized on warfarin, azole antifungal effects on

Intranasal calcitonin Intranasal corticosteroids, in allergic rhinitis Intranasal cromolyn Intranasal decongestants Intravenous (IV) anesthetics Intravenous (IV) naloxone Intrinsic sympathomimetic activity (ISA) Investigational new drug (IND) Iodide Ion(s). See specific types, e.g., Potassium ion Ipratropium Irbesartan Irinotecan Iron poisoning Irritants ISA. See Intrinsic sympathomimetic activity (ISA) Isocarboxazid Isoflurane Isoflurophate Isoleucine Isoniazid (INH) Isoproterenol Isotretinoin Isradipine Itraconazole IV. See Intravenous (IV) anesthetics; Intravenous (IV) naloxone Ivermectin

J “ Jesuit Bark” “ Jet lag,” herbal medication for

K K+ channel blockers k0 Kaolin Kava kava Ketamine Ketoacidosis, diabetic Ketoconazole Ketones Kidney disease, chronic, blood pressure with Kinetics, first-order Kininase. See Angiotensin-converting enzyme (ACE)

L Labetalol Lacrimation, ACh effects on α-Lactam(s) β-Lactam(s), carbapenem Lactulose Lamivudine Lamotrigine Lansoprazole Latrodectus spp. Laxatives, osmotic LD50 Lead poisoning Leflunomide adverse reactions to for rheumatoid arthritis Leishmaniasis Lente insulin Lethal dose (LD50) Letrozole Leucine Leucovorin Leukotrienes (LT s) Leuprolide Levetiracetam Levodopa (L-dopa) Levonorgestrel Levothyroxine Lidocaine

Linezolid Lipopeptide antibiotics, cyclic 5-Lipoxygenase 5-Lipoxygenase inhibitors Lispro insulin Lithium Livedo reticularis Liver failure LMWH. See Low molecular weight heparin (LMWH) Loading dose (LD) Local anesthetics Loop diuretics Loop of Henle Loperamide Lopinavir Loratadine Lorazepam Losartan Low molecular weight heparin (LMWH) LSD. See Lysergic acid diethylamide (LSD) LT s. See Leukotrienes (LT s) Lubiprostone Lumefantrine Lung surfactant Lupus, drug-induced Lysergic acid diethylamide (LSD)

M MAC. See Minimum alveolar concentration (MAC) Macrolide antibiotics Magnesium Magnesium hydroxide Maintenance dose (MD) Malaria Malathion Malignant hyperthermia Manic-depression, agents for Mannitol MAO(s) MAOIs MAOs, MAOB Marijuana Marijuana withdrawal Mast cell stabilizers Maximal response (Vmsx)), of agonist MD. See Maintenance dose (MD) MDMA. See Methylenedioxymethamphetamine (MDMA) Mebendazole Mecamylamine Meclizine Mefloquine Meglitinides Melarsoprol Melatonin Membrane transport Menopause, hot flushes in, herbal medication for Meperidine 6-Mercaptopurine Mercury poisoning Mesalamine Mescaline Mesna Mestranol Metabolic alkalosis Metabolism Metanephrine Metaproterenol Metaraminol Metformin Methadone Methamphetamine Methemoglobin formation Methicillin-resistant Staphylococcus aureus (MRSA) infections Methimazole Methotrexate adverse effects of for IBD

in immunosuppression adverse reactions to for rheumatoid arthritis Methylcellulose α-Methyldopa Methylenedioxymethamphetamine (MDMA) Methylnorepinephrine Methylprednisolone Metoclopramide Metoprolol Metronidazole Mexiletine Midazolam Mifepristone (RU 486) Miglitol Migraine drugs for herbal medication for Milk thistle Milk-alkali syndrome Milrinone Mineral oil Mineralocorticoids Minimum alveolar concentration (MAC) Minoxidil Miosis ACh effects on opioid-induced of pupils Mirtazapine Misoprostol Mitomycin Mitotic inhibitors, anticancer Mivacurium Mixed action adrenergic agonists Monoamine oxidase (MAO) MAOB Monoamine oxidase inhibitors (MAOIs) MAOB inhibitor Monobactams Montelukast Moricizine Morphine Morphine-3-glucuronide Morphine-6-glucuronide Mosquirix Motion sickness MRSA infections Muromonab-CD3 (OKT 3) Muscarine Muscarinic agonists Muscarinic receptors Mycophenolate mofetil Mycoplasma Myoclonic seizures

N Na+ channel, voltage-gated Na+ channel blockers Nabilone Na+/K+-AT Pase Naloxone IV Naltrexone NAPQI. See N-Acetyl-benzoquinoneimine (NAPQI) Naratriptan Narrow therapeutic index (NT I) Nasal congestion Nasal decongestants Natalizumab Nateglinide Nausea and vomiting, agents for NDA. See New drug application (NDA) Nedocromil Nefazodone Nematodes Neonatal respiratory distress syndrome

Neostigmine Nephrogenic diabetes insipidus Nerve fibers, local anesthetics effects on Nesiritide Neurogenic diabetes insipidus Neurokinin receptor antagonists, for nausea and vomiting Neuroleptic malignant syndrome (NMS) Neuromuscular blocking agents (NMBs) nondepolarizing Neurotransmitters, SNS Nevirapine New drug application (NDA) Niacin Nicardipine Niclosamide Nicotinic acid Nicotinic agonists Nicotinic receptors location of Nifedipine Nifurtimox Nimodipine Nitrates Nitric oxide Nitroglycerin (NT G), sublingual Nitrous oxide Nizatidine NMBs. See Neuromuscular blocking agents (NMBs) NMS. See Neuroleptic malignant syndrome (NMS) NNRT Is Noncompetitive antagonist Nondepolarizing NMBs Nonergot DA agonists Nonnucleoside reverse transcriptase inhibitors (NNRT Is) Norepinephrine Norethindrone Norgestrel Normetanephrine Nortriptyline NPH insulin NRT Is NSAIDs. See Anti-inflammatory drugs, nonsteroidal NT G. See Nitroglycerin (NT G) NT I (narrow therapeutic index) Nucleoside reverse transcriptase inhibitors (NRT Is) Nystatin

O Obesity, agents for Obsessive-compulsive disorder (OCD) OCD. See Obsessive-compulsive disorder (OCD) Octreotide OKT 3 Olanzapine Omega-3 fatty acids Omeprazole Onchocerca volvulus Onchocerciasis Ondansetron “ On-off” phenomenon Onychomycosis Opioid(s) in cough suppression poisoning by Opioid analgesics and antagonists Opioid withdrawal Oral hypoglycemic agents Organophosphate(s) poisoning by Orlistat Orthostatic hypotension Oseltamivir Osmotic laxatives Osteoporosis agents for OT C. See Over-the-counter (OT C) Over-the-counter (OT C) sleep aids

Oxazepam Oxazolidinone antibiotics Oxygen Oxymetazoline

P “ Pacemaker current” Paclitaxel 2-PAM. See Pralidoxime (2-PAM) Pamidronate Parasympathetic nervous system (PNS) Parasympathetic preganglionic fibers, location of Parathion Parkinson disease agents for Parkinsonian-like symptoms Paroxetine Partial agonist Partial seizures Partial thromboplastin time (PT T ) Passive diffusion Patent ductus arteriosus PCOS. See Polycystic ovary syndrome (PCOS) PCP. See Phencyclidine (PCP) PDEIs. See Phosphodiesterase inhibitors (PDEIs) Pectin Penciclovir Penicillamine adverse reactions to for rheumatoid arthritis Penicillin(s) aminoglycosides and Pentamidine Pentazocine Pentobarbital Peptic ulcer disease (PUD), agents for Pergolide Periwinkle plant Perphenazine Petit mal seizures PGF1 PGF2α analogs PGs. See Prostaglandins (PGs) Pharmacodynamics Pharmacokinetics Pharmacologic antagonism Phase I, in drug development Phase I metabolism Phase II, in drug development Phase II metabolism Phase III, in drug development Phase IV, in drug development Phenacetin Phencyclidine (PCP) Phenelzine Phenobarbital Phenothiazines, for nausea and vomiting Phenoxybenzamine Phentermine Phentolamine D-Phenylalamine derivatives Phenylephrine Phenytoin Phosphodiesterase inhibitors (PDEIs) Phospholipase A2 Physiologic antagonism Physiologic stress, extreme Physostigmine Pilocarpine Piloerection Pimozide Pindolol Pinworm Pioglitazone Plasma Plasma proteins Plasmodium spp.

Platelet(s) Platelet aggregation Plicamycin Pneumocystis carinii PNS. See Parasympathetic nervous system (PNS) Poisoning. See also specific agents Polycarbophil Polycystic ovary syndrome (PCOS) Polyene antifungals POMC. See Proopiomelanocortin (POMC) Postural hypotension Potassium Potassium ion Potassium-sparing diuretics Potency Potentiation PPIs. See Proton pump inhibitors (PPIs) Pralidoxime (2-PAM) Pramipexole Pramlintide Praziquantel Prazosin Prednisone Pregabalin Preganglionic fibers, location of Pregnenolone Prehypertension Preload Premarin Priapism Prilocaine Primaquine Prinzmetal (variant) angina Probenecid Procainamide Procarbazine Prochlorperazine Progesterone Progestins Prolactinemia Promethazine Proopiomelanocortin (POMC) Propafenone Propofol Propoxyphene Propranolol for cardiac arrhythmias cimetidine with for migraine noncardiovascular uses of for thyrotoxicosis-related cardiovascular issues Propylthiouracil Prostacyclin Prostaglandins (PGs) Protamine Protamine sulfate Protease inhibitors Protein(s) C plasma Prothrombin Proton pump inhibitors (PPIs) Pseudoephedrine Psilocybin Psyllium PT T . See Partial thromboplastin time (PT T ) PUD. See Peptic ulcer disease (PUD) Pulmonary agents Pulmonary fibrosis Pupils, miosis of Purine metabolism Pyrantel Pyrazinamide Pyridostigmine Pyridoxine Pyrimethamine Pyrimethamine/sulfadoxine

Q

QT c. See Corrected QT interval (QT c) Quantal (cumulative) dose-response curve Quaternary ammonium compounds Quetiapine Quinidine Quinine Quinolone antibiotics

R Radioactive iodine (131I) Raloxifene Ranitidine α-Receptor(s) α1 activation of α2 α2-receptor agonists α2-receptor antagonists β-Receptor(s) β β2 H1Receptor(s) H2-Receptor antagonists 5-HT 3-Receptor antagonists, for nausea and vomiting D1-Receptors D2-Receptors “ Red man” syndrome 5-Reductase Renal disease, blood pressure with Renal failure, NSAID-induced Renin Repaglinide Reserpine Respiratory depression during opioid overdose opioid-induced Respiratory distress syndrome, agents for 13-cz’s-Retinoic acid Retinoids Reye syndrome RF. See Rheumatoid factor (RF) Rheumatoid arthritis, agents for Rheumatoid factor (RF) Rhinitis, allergic, agents for Ribavirin Rifampin Risedronate Risperidone Ritonavir River blindness Rocuronium Rofecoxib “ Roofies” Ropinirole Rosiglitazone Rosuvastatin Roundworms RT S-S/ASO2A RU 486

S Salicylism Salivation, ACh effects on Salmeterol Salt(s), gold adverse reactions to for rheumatoid arthritis Sarin Saw palmetto Scalp, dermatophytosis of Schizophrenia Scopolamine Secobarbital Sedative-hypnotic agents Seizures. See also specific types

Selective estrogen receptor modulators (SERMs) Selective serotonin and norepinephrine reuptake inhibitor (SSNRI) Selective serotonin reuptake inhibitor(s) (SSRIs) poisoning by Selegiline Senna SERMs. See Selective estrogen receptor modulators (SERMs) Serotonin (5-HT ) Serotonin syndrome Sertraline Sesquiterpene lactones Sevoflurane “ Shake and bake” reaction SIADH. See Syndrome of inappropriate antidiuretic hormone (SIADH) Sialorrhea Sibutramine Sildenafil Simvastatin Sirolimus Sleep aids OT C Slow-releasing substances of anaphylaxis (SRS-A) SLUDGE SNS. See Sympathetic nervous system (SNS) Sodium Sodium ion Sodium nitroprusside Somogyi effect Sorbitol Sotalol Spasm(s), bronchiolar Spider(s), venom of Spironolactone Spontaneous coronary vasospasm SRS-A. See Slow-releasing substances of anaphylaxis (SRS-A) SSNRI. See Selective serotonin and norepinephrine reuptake inhibitor (SSNRI) SSRIs. See Selective serotonin reuptake inhibitors (SSRIs) St. John’s wort Stage I hypertension Stage II hypertension Statin(s) Status asthmaticus Status epilepticus Stavudine Steady state (Css) Steroids. See also specific types adrenal Stevens-Johnson syndrome Stibogluconate Stimulants Stomach ulcers Stool softeners Streptogramin antibiotics Streptokinase Streptokinase-plasminogen complex Streptomycin Stress, extreme physiologic Sublingual nitroglycerin (NT G) Substance abuse and tolerance Substituted benzamides, for nausea and vomiting Succimer Succinylcholine Sucralfate Sulfadiazine Sulfasalazine Sulfinpyrazone Sulfonamide antibiotics allergy to Sulfonamides Sulfonylureas Sumatriptan Suppository(ies), glycerin Surfactant replacement therapy Sweating, ACh effects on Sympathetic nervous system (SNS) neurotransmitters of Syndrome of inappropriate antidiuretic hormone (SIADH)

T

tl/2. See Half-life (t1/2) T 3, half-life T 4, half-life T acrolimus T adalafil T amoxifen T amsulosin T apeworms T ardive dyskinesia (T D) T BW. See T otal body water (T BW) T CAs. See T ricyclic antidepressant(s) (T CAs) T D. See T ardive dyskinesia (T D) T D50 T elithromycin T emazepam T eniposide T enofovir T erazosin T erbinafine T erbutaline T eriparatide T estosterone T etracycline antibiotics T heophylline poisoning by T herapeutic index (T I) T hiabendazole T hiazide diuretics T hiazolidinediones (T ZDs) 6-T hioguanine T hionamides T hiopental T hioridazine T hrombin T hrombolytic agents poisoning by T hromboxane A T hromboxanes (T XAs) T hrush T hyroid disorders, agents for T hyroid storm T hyroid-stimulating hormone (T SH) T hyrotoxicosis T hyrotropin-releasing hormone (T RH) T hyroxine (T 4) T I (therapeutic index) T iclopidine T igecycline T imolol T inea capitis T iotropium T ipranavir T irofiban T issue plasminogen activator (tPA) T ocainide T olazamide T olbutamide T olcapone T onic-clonic seizures T opiramate T opotecan T orsades de pointes T orsemide T otal body water (T BW) T ourette syndrome T oxic dose (T D50) T oxicology T oxin(s), Botulinum tPA. See T issue plasminogen activator (tPA) T ranexamic acid T ranylcypromine T razodone T rematodes T remors T retinoin T RH. See T hyrotropin-releasing hormone (T RH) T riamcinolone T riamterene

T riazolam T ricyclic antidepressants (T CAs) poisoning by T rihexyphenidyl T riiodothyronine (T 3) T rimethaphan T rimethoprim T rimethoprim-sulfamethoxazole “ T riptans” Trypanosoma spp. T. brucei gambiense T. brucei rhodesiense T. cruzi T. gondii-45 T rypanosomiasis African American T SH. See T hyroid-stimulating hormone (T SH) T ubocurarine T XAs. See T hromboxanes (T XAs) T yramine T yrosine T ZDs. See T hiazolidinediones (T ZDs)

U Udenafil Ulcer(s) duodenal gastric stomach Ulcerative colitis Ultralente insulin Unstable angina Uric acid elimination Uricosuric agents Urinary retention, opioid-induced Urine acidification of alkalinization of Urokinase

V V1 receptor V2 receptor Valacyclovir Valdecoxib Valganciclovir Valproic acid (VPA) Vancomycin Vanillylmandelic acid (VMA) Vardenafil Vasoactive intestinal peptide secreting tumors (VIPomas) Vasodilation Vasodilators Vasopressin Vaughn-Williams classification Vd Venlafaxine Verapamil Vinblastine Vinca alkaloids Vincristine VIPomas Vitamin(s) A B3 B6 D fat-soluble K VMA. See Vanillylmandelic acid (VMA) Vmax of agonist Voltage-gated Na+ channel Volume of distribution (Vd) Vomiting, agents for Voriconazole

VPA. See Valproic acid (VPA)

W Warfarin ASA and cimetidine and herbal supplement with INR of patient stabilized on, azole antifungal effects on monitoring of poisoning by Wilson disease Withdrawal, drug. See specific drug

X Xanthine oxidase inhibitor Xerostomia

Y Yohimbine Zafirlukast

Z Zaleplon Zanamivir Zero-order elimination Zidovudine (AZT ) Zileuton Zollinger-Ellison syndrome Zolpidem Zona fasciculata Zona glomerulosa Zona reticularis

View more...

Comments

Copyright ©2017 KUPDF Inc.
SUPPORT KUPDF